Sunteți pe pagina 1din 175

Clinical Scenarios in

Oto-rhino-laryngology
A Problem Oriented Approach

First Edition
ISBN: 978-969-9340-01-7

PROF. DR. IQBAL HUSSAIN UDAIPURWALA


MBBS., DLO., FCPS.
Professor and Head of the ENT Department
Bahria University Medical & Dental College, Karachi.
Fellow and Examiner, College of Physicians & Surgeons Pakistan.
Editor, Pakistan Journal of Otolaryngology and Head & Neck Surgery.
Visiting ENT Surgeon, Liaquat National Hospital, Habib Medical Centre
and Zubaida Medical Centre, Karachi.

Clinical scenarios in oto-rhino-laryngology

Great care has been taken to maintain information contained in the volume. However,
in view of the ongoing research and changes in the government rules and regulation and a
constant inflow of information, the author can not be held responsible for errors or for any
consequences arising from the use of the information contained herein.

Copyright Reserved
All rights are reserved with the publisher. No part of this publication may be
reproduced, stored in a retrieval system, or transmitted, in any form or by any means,
electronic, mechanical, photocopying, recording or otherwise, without the prior permission
of the publisher.

ISBN: 978-969-9340-01-7

First Edition, 2011

II

Clinical scenarios in oto-rhino-laryngology

Preface to the first edition


It is a moment of great pleasure for me to present first edition of this book. Clinical
subjects are always difficult for the medical students because most of the text books are written
according to the diseases or systems but when they deal with the patients, who always come
with some complaints. To correlate these complaints of the patients with the disease and to
make diagnosis is always difficult for them. This book is an endeavor to improve clinical
acumen and interpretation of a medical student.
All common diseases of oto-rhino-laryngology and head & neck are presented in the
form of case discussions. A clinical scenario is presented first and then important points in
history taking and clinical examination are described with clinical, provisional or differential
diagnosis. How this patient will be investigated and how the diagnosis will be reached is then
given. At the end important points related with the diseases is discussed briefly.
This book could not have been accomplished without the help and assistance of many
people. I would like to express my sincerest gratitude to all my teachers and colleagues who
gave me valuable guidance and suggestions in writing this textbook. My special thanks goes
to Dr. Muhammad Shuja Farrukh, assistant professor of ENT, Dow University of Health
Sciences, Karachi who gave me his utmost support and full time assistance in proof reading of
the manuscript several times, as indeed in seeing it in the form of a print. I am thankful to Mr.
Rehan Ahmed Khan and Mr. Rehan Zia of Hamdard University Hospital for their technical
services and support in title designing and computer work. In the last but not least, I am grateful
to my wife Azra Iqbal and my daughters, Fatima, Saba and Zahra for their enormous support
and untiring efforts at every step of this work, without them it would not be possible to make
this idea into a reality.
I hope medical students will find it very useful in preparation of their final examination
in ENT as well as in their clinical practice. By no means it is perfect and there may be some
ambiguity in the text. Your suggestions and criticism are always welcome to improve the
standard of this book.

Karachi, 2011.

IQBAL HUSSAIN UDAIPURWALA

III

Clinical scenarios in oto-rhino-laryngology

Foreward
What is special about this book written by Prof. Iqbal Hussain Udaipurwala, who is
already author of few books on oto-rhino-laryngology. Real time clinical scenario based book
is a smart scientific attempt for teaching and learning, which makes it interesting and exciting.
Most traditional textbooks of oto-rhino-laryngology contain basic clinical and scientific facts
that forms the foundation of the speciality. While these text can provide an essential cornerstone
for the practice of oto-rhino-laryngology, applying this information to a clinical setting relies
on sound judgement, presence of mind and clinical experience. Problem based learning for
under-graduate and post-graduate teaching is very rewarding and practical. Based on real time
clinical cases, which you can witness by actual clinical case, patient pathology and photographs,
you get a confidence of trueness. This helps confidence building regarding diagnosis and
management.
I must commend the author, Prof. Iqbal Hussain Udaipurwala for contributing this
scientific book for learners, this is bound to enrich the readers mind and skill beyond his
expectations.

PROF. SYED TIPU SULTAN


MBBS, DA (London), FFARCSI, FCPS (Hon.)
Professor of Anaesthesiology, critical care and pain management,
Principal, Bahria University Medical & Dental College,
Council Member, College of Physicians & Surgeons, Pakistan,
President, Pakistan Medical Association (centre).

IV

Clinical scenarios in oto-rhino-laryngology

CONTENTS
SECTION I: EAR
Case 1- Boil in the ear
Case 2- Foreign body of the ear
Case 3- Wax in the ear
Case 4- Pre-auricular sinus
Case 5- Otomycosis
Case 6- Maggots in the external auditory canal
Case 7- Acute suppurative otitis media
Case 8- CSOM with aural polyp
Case 9- CSOM with cholesteatoma
Case 10- CSOM with facial paralysis
Case 11- Dry perforation of ear drum
Case 12- Otitis media with effusion
Case 13- Otosclerosis
Case 14- Presbyacusis
Case 15- Noise induced hearing loss
Case 16- Meneires disease
Case 17- Benign paroxysmal positional vertigo
Case 18- Congenital deafness

3
6
9
12
14
16
18
21
24
28
31
34
38
42
45
48
51
54

SECTION II: NOSE & PARANASAL SINUSES


Case 19- Deviated nasal septum
Case 20- Nasal trauma with fracture
Case 21- Antro-choanal polyp
Case 22- Septal haematoma
Case 23- Ethmoidal polypi
Case 24- Septal adhesion
Case 25- Allergic rhinitis
Case 26- Foreign body of the nose
Case 27- Boil in the nose

59
62
66
70
73
76
78
81
84

Clinical scenarios in oto-rhino-laryngology

Case 28- Epistaxis


Case 29- Chronic rhino-sinusitis
Case 30- Nasopharyngeal angiofibroma

86
89
92

SECTION III: ORAL CAVITY & PHARYNX


Case 31- Chronic tonsillitis
Case 32- Post-tonsillectomy haemorrhage
Case 33- Quinsy
Case 34- Enlarged adenoids
Case 35- Papilloma of the cheek
Case 36- Carcinoma of the cheek
Case 37- Carcinoma of the tongue
Case 38- Foreign body of the throat
Case 39- Ranula

99
102
104
107
110
113
117
120
123

SECTION IV: LARYNX & TRACHEA


Case 40- Vocal nodules
Case 41- Tracheostomy
Case 42- Carcinoma of the larynx
Case 43- Foreign body of the bronchus

127
130
133
138

SECTION V: HEAD & NECK


Case 44- Ludwigs angina
Case 45- Branchial cyst
Case 46- Retro-pharyngeal abscess
Case 47- Sub-mandibular salivary calculus
Case 48- Parotid gland pleomorphic adenoma
Case 49- Multinodular goiter (MNG)
Case 50- Tuberculous cervical lymphadenopathy

143
146
149
152
155
158
162

VI

Clinical scenarios in oto-rhino-laryngology

Section I
EAR

Case 1Case 2Case 3Case 4Case 5Case 6Case 7Case 8Case 9Case 10Case 11Case 12Case 13Case 14Case 15Case 16Case 17Case 18-

Boil in the ear


Foreign body of the ear
Wax in the ear
Pre-auricular sinus
Otomycosis
Maggots in the external auditory canal
Acute suppurative otitis media
CSOM with aural polyp
CSOM with cholesteatoma
CSOM with facial paralysis
Dry perforation of ear drum
Otitis media with effusion
Otosclerosis
Presbyacusis
Noise induced hearing loss
Meneires disease
Benign paroxysmal positional vertigo
Congenital deafness

3
6
9
12
14
16
18
21
24
28
31
34
38
42
45
48
51
54

Clinical scenarios in oto-rhino-laryngology

Case 1
Clinical Scenario
A 28 years male patient came in the OPD with complaint of pain in the left ear
for last 2 days. Initially pain was mild to moderate but next day it increased and became
severe. He also had deafness and swelling around the ear canal with some pussy discharge.
On examination his left pinna was very tender especially over the tragus with yellowish
pus coming out. On retraction of the pinna a rounded, soft and fluctuant swelling was
visible at the external auditory meatus (fig. 1.1). It was not possible to examine deeper
part of the canal and tympanic membrane because of pain.

Fig. 1.1
Rounded, soft, tender and fluctuant
swelling in the external auditory
meatus

Important points in history taking:


1- Any previous history of discharge from the ear. In this case there was no
such previous history.
2- History of diabetes mellitus or other immuno-compromised states. In
this case no such history.
3- Habit of scratching the ears with any sharp object. There was no such
history.
4- Any history of swimming especially in dirty water. There was no such
history.
Important points in clinical examination:
1- Palpation of tragus, pinna and mastoid area for tenderness. Tragus and
pinna were tender but mastoid area was not tender in this case.
2- Probe test of the swelling. Swelling was very tender and soft. It was not
possible to move the probe all around the swelling i.e swelling is arising
from the canal wall in its outer cartilagenous part.
3- Tuning fork tests showed conductive type of deafness. Rinnes test was
negative in the left ear and positive in the right ear, Webers test was
3

Clinical scenarios in oto-rhino-laryngology

lateralized towards the left and and Schwabachs test was equal to the
examiner in both ears.
Diagnosis:
The most probable diagnosis of this case was Boil ear.
Differential Diagnosis:
The differential diagnosis in this case could be:
1- Aural polyp: In aural polyp there is history of chronic discharging ear
for a long time. In addition probe test will differentiate a polyp from
swelling arising from the external auditory canal.
2- Osteoma: This is a benign tumour of bony origin and is situated in the
deeper bony part of the external auditory canal. It is hard and usually
non-tender.
Investigations:
No investigation was done in this case.
Treatment:
This was a case of large boil where pus was present along with
fluctuation and patient was already taking antibiotic without any relieve. So
incision and drainage was planned under local anaesthesia. A longitudinal
incision was given parallel to the external auditory canal. Pus was drained
and sent for culture and sensitivity. The external auditory canal was packed
with antiseptic ointment. Antibiotic against staphylococcus aureus was given
parenterally (amoxicillin with clavulanic acid).
Pus drained after incision & drainage was sent for culture and
sensitivity, which showed heavy growth of staphylococcus aureus. The
organisms were found to be sensitive to amoxicillin with clavulanic acid, so
the same antibiotic was continued for 7 days.
Discussion:
Boil or furuncle is the acute infection of the hair follicle by
staphylococci. In the external auditory canal hair follicles are only present
in the outer one-third part. Boil in the ear is usually single but multiple boils
can occur sometimes. The predisposing factors are diabetes mellitus, general
debilitating diseases, scratching of the external auditory canal, swimming
and poor hygiene.
Following points are important to remember in case of boil in the
ear:
1- Recurrent boil is common in patients having diabetes mellitus. Thus if
any patient comes with recurrent boil, diabetes mellitus should be excluded.
2- During incision and drainage of the boil ear, incision is always given
parallel to external auditory canal. The circumferential incision at the
external auditory meatus may lead to stenosis later on, so it should be
avoided.
4

Clinical scenarios in oto-rhino-laryngology

3- Boil in the ear is a very painful condition because the skin is tightly
adherent to the underlying cartilage.

TEST YOURSELF

Read the clinical scenario given at the beginning and answers the following questions
12345-

What is the most likely diagnosis in this case?


What are the differential diagnosis in this case?
How will you investigate this case?
How will you manage this case?
What are the important points to remember in a case of boil in the ear?

Clinical scenarios in oto-rhino-laryngology

Case 2
Clinical Scenario
A mother brought her 4 years old son with the complaint that he had inserted
something in his right ear 3 hours back. She tried to remove it with a forceps, which
resulted in further pushing of the foreign body deeper in the canal. He was also
complaining of mild pain in his right ear. Otoscopy showed foreign body (a bead)
impacted deeper in his external auditory canal (fig. 2.1)

Fig. 2.1
Otoscopic finding showing a foreign
body in the external auditory canal

Important points in history taking:


1- Inquire about nature of the foreign body, whether it was vegetative or
non-vegetative, metallic or non-metallic, smooth or sharp, rounded or
irregular etc. In this case nature of the foreign body was not known.
2- Duration of foreign body insertion. In this case it was three hours.
3- Any attempt of removal by family member or family doctor. Unskilled
attempt for removal may cause further pushing of foreign body deeper
and trauma to the surrounding structures. In this case there was history
of removal of the foreign body by patients mother herself.
4- Any bleeding from the ear.
5- Pain in the ear. Pain is caused by pressure of the foreign body or trauma
to external auditory canal or ear drum.
Important points in clinical examination:
1- Assess the site of impact.
2- Confirm the nature of foreign body.
3- Any trauma or bleeding present in the external auditory canal.
4- General condition of the child, whether he is anxious or co-operative.
Removal of foreign body in an anxious and unco-operative child may
6

Clinical scenarios in oto-rhino-laryngology

cause more trauma. So it is always better to remove foreign body under


general anaesthesia.
Diagnosis:
This was a case of impacted foreign body in right external auditory
canal.
Investigations:
No investigation is required in otherwise healthy child in such an
emergency situation.
Treatment:
Patient was admitted for removal of foreign body under general
anaesthesia, as he was crying and very anxious, even not allowing proper
examination. Under general anaesthesia foreign body was removed by passing
a ring probe beyond the foreign body and pulling it out (fig. 2.2).

Fig. 2.2
Method of removal of a rounded
foreign body by ring probe

Discussion:
Foreign bodies in the ear may be animate such as insects or inanimate.
Inanimate foreign bodies are usually introduced by children and mentally
retarded persons by themselves. Inanimate foreign body may be hygroscopic
or vegetative like seeds or non-hygroscopic or non-vegetative like metals,
plastic and other materials. A hygroscopic foreign body absorbs water and
moisture present in the canal and swells up and gets impacted in the canal.
Isthmus of the external auditory canal is the narrowest part and most of the
foreign bodies are impacted at isthmus.
Method of removal depends upon the size, site of impaction and type
of foreign body. Removal under general anaesthesia is essential in children
and sensitive adults. Smooth and rounded foreign body is removed by a ring
probe. Forceps must not be used in such foreign bodies as it can push the
foreign body further in.
7

Clinical scenarios in oto-rhino-laryngology

TEST YOURSELF

Read the clinical scenario given at the beginning and answers the following questions
1234-

How will you manage this patient?


What are the options for removal of foreign body from the external auditory canal?
Classify foreign bodies of the external auditory canal.
What is the narrowest part of external auditory canal?

Clinical scenarios in oto-rhino-laryngology

Case 3
Clinical Scenario
A 27 years old male patient came with the complaints of blockage of the right
ear after swimming in the pool on a picnic 2 days back, which was continous and same.
He also had mild pain and discomfort in his right ear.
Important points in history taking:
1- Any discharge from the ear. No discharge was present in this case.
2- History of cold or sore throat before going to swimming. There was no
such history.
3- Itching in the ear. Slight itching was present in this case.
4- Use of ear plugs during swimming. He had not used ear plugs during
swimming.
5- Any history of bleeding from the ear. There was no history of bleeding
from the ear.

Fig. 3.1
Otoscopic finding of the right ear

Important points in clinical examination:


1- Ear examination with the speculum and otoscope. Showing accumulation
of dark brown wax in the external auditory canal (fig 3.1).
2- Suction cleaning of the ear and inspection of the tympanic membrane.
Wax was not possible to be removed by suction as it was impacted and
hard.
3- Tenderness over pinna, tragus and mastoid area. There was no tenderness
on any area in this case.
4- Tuning fork tests. Rinnes test was negative in right and positive in left
ear, Webers test was lateralized to the right and Schwabachs test was
equal to the examiner on both the sides.
9

Clinical scenarios in oto-rhino-laryngology

5- Examination of the nose and throat for any pathology. No positive finding
was present in these areas.
Diagnosis:
The diagnosis in this case was impacted wax in the right ear.
Differential Diagnosis:
1- Otomycosis, where wet newspaper like mass is seen in the canal.
2- Traumatic perforation of the ear drum.
3- Otitis externa or boil. It is very painful and tenderness is present over
tragus and pinna.
Investigations:
No laboratory investigation was done in this case as the diagnosis
was clear. In case of suspected otomycosis, removed debris from the ear
should be sent for fungal smear.
Treatment:
Wax was first softened by instilling a softening agent like 2% soda
glycerine ear drops thrice daily for 2 days and suction cleaning was done
later on (fig 3.2). After suction cleaning of the ear, tympanic membrane and
external auditory canal were found to be normal and patient had normal
hearing.

Fig. 3.2
Suction cleaning of the wax

Discussion:
Wax or cerumen is the mixture of the secretions of ceruminous and
pilo-sebaceous glands. These glands are present only in the cartilagenous
portion of external auditory canal.The proportions in the mixture determine
the consistency of the wax. When secreted it is thick and golden brown in
colour, which becomes darker and hard on drying. Normally it is expelled
from the canal in flakes, aided by the movement of jaw. Plug formation is
encouraged by excessive formation of wax and its retention by stiff hairs,
10

Clinical scenarios in oto-rhino-laryngology

exostosis, desquamation and other stenosing conditions. The options for the
removal of soft wax are:
1- Syringing (fig. 3.3)
2- Suction cleaning (fig. 3.2)

Fig. 3.3
Syringing of the ear in another patient

TEST YOURSELF
Read the clinical scenario given at the beginning and answers the following questions
12345-

What are the differential diagnosis in this case?


How will you manage this case?
What is wax and how it is formed?
What are the signs and symptoms of a patient having impacted wax in the ear?
What are the different options for removal of impacted wax in the ear?

11

Clinical scenarios in oto-rhino-laryngology

Case 4
Clinical Scenario
A mother brought her 10 years old daughter with the complaint that she had a
small hole in front of her ear on both the sides since birth (fig 4.1). There was history
of repeated discharge often white or yellow in colour from these openings, which settled
down with medication from local general practitioner. Discharge was often associated
with pain and redness in front of the ears.

Fig. 4.1
Left ear showing a small hole or
opening with redness and swelling in
front of the crus helix

Important points in history taking:


1- Whether it was present at birth. In this case it was present at birth.
2- Any history of discharge from the opening or redness and pain of the
surrounding area. In this case there was history of occassional discharge
with redness and pain around the opening for which she takes medicines.
3- Unilateral or bilateral. In this case it was bilateral.
Important points in clinical examination:
1- Assess whether it is infected or not. At time of presentation there was no
sign of infection except skin was slightly red and congested.
2- Any other opening in external auditory canal. There was no other opening.
3- Assess for any other congenital abnormality of the ear. All other areas
were within normal limits.
Diagnosis:
This was a case of Pre-auricular sinus.
Investigations:
1- Pus or discharge for C/S. This patient when presented has a dry opening
so C/S was not done.
12

Clinical scenarios in oto-rhino-laryngology

2- Other baseline investigations for general anaesthesia when planned for


surgery like blood complete picture, ESR, random blood sugar, urine D/R
and X-ray chest (PA view).
3- Sinogram: It is done to delineate the whole sinus and its tract. In routine
cases it is not indicated, so it was not done in this case.
Treatment:
Surgery was planned after investigation. Under general anaesthesia
an elliptical incision was given (fig. 4.2). Whole tract and opening of the
sinus was excised and wound was closed in two layers.

Fig. 4.2
Elliptical incision was marked before
surgery

Discussion:
Pre-auricular sinus is a congenital condition and is due to failure of
complete fusion between the first and second branchial arch elements in the
auricle. External opening of pre-auricular sinus is situated between the tragus
and crus helix.

TEST YOURSELF

Read the clinical scenario given at the beginning and answers the following questions
1234-

What is your diagnosis in this case?


How will you manage this case?
What is a pre-auricular sinus and how is it formed?
What is the location of external opening of pre-auricular sinus?

13

Clinical scenarios in oto-rhino-laryngology

Case 5
Clinical Scenario
A 24 years old male patient came with the complaints of severe itching and
discharge from his right ear for last 10 to 12 days. On clinical examination, otoscopic
findings are shown in fig. 5.1.

Fig. 5.1
Otoscopic findings of the right ear

Important points in history taking:


1- Detailed history regarding itching and discharge. Itching was severe and
continuous present all the time. Discharge was scanty, creamish to yellow
in colour, thick and often contained blackish spots. It was never blood
stained.
2- Previous history of discharge. There was no history of discharge in the
past.
3- History of swimming. He went to a picnic spot where he did swimming
in a small swimming pool about three weeks back. He had not used ear
plugs during swimming.
4- Any history of pain. There was no pain in the ear.
5- Any history of deafness or blockage of the ear. According to him his right
ear was blocked since the start of these symptoms.
6- Any history of diabetes mellitus or any immuno-compromised state.
There was no such history.
7- Habit of scratching the ears with different objects. There was no such
history in this case.
Important points in clinical examination:
1- Examination of the external auditory canal and tympanic membrane.
External auditory canal was full of creamish yellow debris with brownish
black spot (fig. 5.1). Tympanic membrane was not visible.
14

Clinical scenarios in oto-rhino-laryngology

2- Voice test. Mild deafness was present in the right ear.


3- Tuning fork tests. Rinnes test was negative in the right ear and positive
in the left ear. Webers test was lateralized towards the right side and
Schwabachs test was equal to the examiner on both sides.
4- Suction cleaning of the debris done and examination of the external
auditory canal and tympanic membrane was done, which were both
oedematous and congested.
Differential Diagnosis:
1- Otomycosis
2- Impacted wax in the ear
3- Other types of otitis externa
Investigations:
1- Debris removed from the external auditory canal was sent for fungal
smear, which showed presence of fungal hyphae, confirming the diagnosis.
Diagnosis:
This was a case of otomycosis or fungal infection of the external
auditory canal.
Treatment:
Suction cleaning of the external auditory canal was done completely.
Clotrimazole lotion (anti-fungal drops) was given for topical application in
the right ear, three times a day. Patient was advised for dry mopping of the
external auditory canal before instilling lotion in the ear. Regular follow-up
was done and once again suction cleaning was done after 4 days. Patients
ear became normal and dry within ten days.
Discussion:
Otomycosis is the fungal infection of the lining skin of external
auditory canal. Swimming in the dirty water or continuous discharge due to
otitis media, are important predisposing factors. Aspergillus is the most
common type of fungus causing this condition but in some cases candida
albicans is the causative organism. Secondary bacterial infection is very
common which causes pain. On examination the external auditory canal is
filled with a wet news paper or blotting paper like mass and its colour depends
upon the type of fungus.

TEST YOURSELF

Read the clinical scenario given at the beginning and answers the following questions
1- What is your diagnosis in this case?
2- How will you manage this patient?
3- What are the different varieties of fungus causing otomycosis?
15

Clinical scenarios in oto-rhino-laryngology

Case 6
Clinical Scenario
A father brought his 2 years old son with the complaints of severe pain, discharge
and bleeding from the right ear for last 2 to 3 days (fig. 6.1). He took medicines from
his family physician but there was no relief and the condition was worsening.

Fig. 6.1
Patient with severe pain, discharge and
bleeding from the right ear

Important points in history taking:


1- Detailed history regarding his symptoms. According to his father, patient
was completely alright 3 days back, when he complained of pain in his
right ear. Next day he also had some discharge from his right ear along
with pain. He took some medicines from his family physician and pain
subsided. Next day patient again had severe pain and discharge along
with blood.
2- Previous history of discharge. There was no history of discharge in the
past.
3- Any history of trauma, scratching or foreign body insertion. There was
no such history.
4- Any history of cold, fever, sore throat or respiratory tract infection. There
was no such history.
5- Socio-economic and hygienic condition. They lived in a village with very
poor hygienic condition with lots of house flies and mosquitos.
Important points in clinical examination:
1- Examination of the right ear. There was discharge and blood coming out
from the right external auditory canal (fig. 6.1). Patient had severe
tenderness over the pinna and surrrounding area. All discharge and blood
cleaned from the external ear and external auditory canal. There were
many maggots present in the external auditory canal (fig. 6.2).
16

Clinical scenarios in oto-rhino-laryngology

Fig. 6.2
Maggots removed from the patients
right ear

Diagnosis:
This was a case of Maggots in the ear.
Treatment:
Patient was admitted in the hospital. Few drops of maggot oil were
instilled in the right ear and maggots removed. All debris and discharge also
cleaned from the external auditory canal and pack soaked in antiseptic
ointment was applied. Broad spectrum parenteral antibiotic was started along
with analgesic. Daily dressing and cleaning of the external auditory canal
was done. Subsequent recovery was uneventful.
Discussion:
Maggots are the larvae of housefly. These flies are attracted by the
foul smelling discharge present in the ears or nose and lay their eggs into
the external auditory canal or nasal cavity. Within 24 hours these eggs hatched
out into larvae or maggots. Maggots produces severe pain, irritation, swelling,
foul smelling and blood stained discharge. On examination maggots are
visible crawling in the external auditory canal. They may cause extensive
soft tissue necrosis.
Treatment consist of removal of all the maggots with forceps but
they are usually firmly attached to the meatal wall. Maggot oil (turpentine
oil) or chloroform water is instilled which causes asphyxia and killing of
maggots thus facilitating their removal.

TEST YOURSELF

Read the clinical scenario given at the beginning and answers the following questions
1- How will you manage this patient?
2- What are maggots?
3- Outline clinical features of maggots in the ear.

17

Clinical scenarios in oto-rhino-laryngology

Case 7
Clinical Scenario
A mother brought her 7 years old son with the complaint that his son was not
looking well since yesterday, when he went to bed early. At midnight he woke up with
the complaint that he has severe pain in his right ear. She gave him syrup paracetamol
and put some eardrops in his right ear, after that he slept again. Next morning when he
woke up he was again complaining of severe pain in his right ear.
Important points in history taking:
1- Detailed history regarding pain like character, nature, site, severity,
radiation, aggravating and relieving factors etc.
2- Any history of fever. In this case there was history of fever since yesterday.
3- Any history of present or previous discharge from the ear. There was no
such history in this case.
4- Any history of deafness or hearing impairment. On inquiry child mentioned
about the blockage or hearing impairment in his right ear.
5- Any history of sore throat, cold, nasal obstruction, nasal discharge, postnasal dripping etc. In this case there was history of common cold for last
two days.
6- Any history of scratching of the external auditory canal, foreign body
insertion, swimming or entry of water during bathing. In this case there
was no such history.
Important points in clinical examination:
1- Inspection of the pinna, external auditory canal, mastoid area, pre-auricular
region along with palpation for tenderness in these areas. All were within
normal limits.
2- Examination of tympanic membrane with speculum and otoscope. In this
case tympanic membrane was congested and bulging, more prominently

Fig. 7.1
Otoscopic picture of tympanic
membrane on right side showing
congested and bulging tympanic
membrane

18

Clinical scenarios in oto-rhino-laryngology

in its posterior half (fig. 7.1).


3- Tuning fork tests. It was not done in this case because of severe pain and
anxiety.
4- Examination of the nose and throat. In this case both nasal cavity and
throat were congested with secretions in both nasal cavities.
Diagnosis:
This was a typical case of acute suppurative otitis media. Pain usually
starts in the night during sleeping, when the ear is in dependent position
along with venous stasis and reduced eustachian tube opening.
Differential Diagnosis:
In a child of 7 years following conditions have to be differentiated
with acute suppurative otitis media:
1- Diffuse otitis externa and boil in the ear. These two conditions may
present with acute pain but the pain is not deep seated as in case of acute
suppurative otitis media. In addition there will be tenderness on the tragus
and pinna with oedema or swelling in the external auditory canal. Tympanic
membrane will be normal with no hearing loss.
2- Referred earache. In such cases tympanic membrane and external auditory
canal are normal with no deafness or discharge. Look the other areas for
referred earache like oral cavity, tonsils, pharynx, teeth, nose and neck.
Investigations:
No investigation was done in this case as the diagnosis was clear.
Pus is sent for culture and sensitivity in cases of tympanic membrane
perforation with discharge or in cases where myringotomy is done for
evacuation of pus.
Treatment:
The patient was planned for medical treatment first. Following
treatment was given and the patient was followed up for improvement.
1- Antibiotic (amoxicillin with clavulanic acid) in suspension form was
given according to body weight of the patient.
2- Syrup ibuprofen with psuedoephedrine.
3- Steam inhalation, twice daily for 10 minutes each.
Patient responded well on the above mentioned treatment and there
was no need for myringotomy.
Discussion:
Acute suppurative otitis media is the acute inflammation of the lining
mucous membrane of the middle ear cleft. Clinically it is divided into the
following four stages:
1- Stage of tubo-tympanitis
2- Stage of catarrhal inflammation
3- Stage of suppuration
19

Clinical scenarios in oto-rhino-laryngology

4- Stage of resolution or complication


This patient presented in the stage of suppuration where frank pus
was present in the middle ear with bulging of the tympanic membrane.
Myringotomy is often needed in this stage to evacuate the pus from the
middle ear when bulging of tympanic membrane is more or in cases where
no improvement occurs with medical treatment. The common micro-organism
responsible for acute suppurative otitis media are:
1- Streptococci
2- Pneumococci
3- Haemophilus influenzae
4- Morexella catarrhalis
The final outcome or sequelae of acute suppurative otitis media may
be:
1- Infection may halt at any stage with complete resolution.
2- Ruptured tympanic membrane may heal with return of normal hearing.
3- Ruptured tympanic membrane may heal with thin paper like membrane
with scarring and some residual hearing loss.
4- Ruptured tympanic membrane may not heal and residual dry perforation
remains with conductive hearing loss.
5- Acute inflammation may change into chronic suppurative otitis media
(tubo-tympanic type).
6- If the condition is not treated properly, complication may arise due to
spread of infection to other sites.

TEST YOURSELF

Read the clinical scenario given at the beginning and answers the following questions
1234567-

What are the important points in history taking and clinical examination in this patient?
What is the most likely diagnosis in this case?
What are the differential diagnosis in this case?
How will you manage this case?
What are the different stages of acute suppurative otitis media?
What are the common micro-organisms responsible for acute suppurative otitis media?
What are the possible outcomes or sequelae of acute suppurative otitis media?

20

Clinical scenarios in oto-rhino-laryngology

Case 8
Clinical Scenario
A 16 years old boy came in the OPD with complaint of some mass in the left
ear for last few months. On inquiry he said that there was history of discharge from the
left ear for last many years. Now he also had marked deafness from his left ear. On
clinical examination a reddish mass was seen coming out from the left external auditory
canal along with profuse purulent discharge (fig. 8.1).

Fig. 8.1
A reddish mass coming out from the
external auditory canal along with
purulent discharge

Important points in history taking:


1- Detailed history about discharge from the ear i.e. onset, continuous or
intermittent, amount, colour, foul smelling, blood stained, aggravating
and relieving factors etc. In this patient, discharge was present for the
last many years. It was almost continuous, profuse, yellow in colour and
foul smelling. Discharge reduces in amount whenever he takes medicine
from the general practitioner for few days and then after becomes the
same.
2- Detailed history about the mass like its onset and progression. He told
that few months back he noticed heaviness and something in his left ear
canal. Gradually that mass increased in size and later it came out of the
ear canal upto its present size. There was occassional bleeding from the
mass whenever he tried to clean the mass.
3- Detailed history regarding deafness and pain. Deafness was present since
the start of discharge but initially it was mild and it increased gradually
and now he has marked hearing loss. Regarding pain, it occurred off and
on and relieved by taking medicines.
4- Any history of fever, headache, altered conciousness, vomiting, neck
stiffness or any other neurological symptom. In this case there was no
such history.
21

Clinical scenarios in oto-rhino-laryngology

Important points in clinical examination:


1- General physical examination. It showed that the patient was a young,
average built boy sitting comfortably and fully oriented in time, space
and person.
2- Inspection of the external ear. It showed a reddish, irregular, smooth
surface, shiny mass filling the external auditory meatus and coming out
from the canal. There was also yellowish muco-purulent discharge around
the mass and adjacent pinna.
3- Examination of the post-aural region. There was no significant finding
and this region was normal.
4- Probe test of the mass. It showed that the mass is soft, mobile and appears
that it was not attached to the external auditory canal and pedicle was
deep seated. It did not bleed on touch.
5- Examination of the discharge. It was present around the mass in the
external auditory canal and adjacent pinna. Pus was profuse, yellow in
colour, mixed with mucous, foul smelling but not blood stained. Pus was
collected on a sterile swab and sent for culture and sensitivity.
6- Examination of the tympanic membrane. It was not visible because of
the mass.
7- Voice test, showed moderate degree of hearing loss.
8- Tuning fork tests. Rinnes test was negative in the left ear and positive
in right ear. Webers test was lateralized towards the left side and
Schwabachs test was equal to the examiner in both ears.
9- Fistula test. It was negative in both the ears.
10- Vestibular function tests. All appeared to be within normal limits.
11- Examination of the facial nerve. It was found to be intact.
12- Examination of the nose and throat. These regions were within normal
limits.
Investigations:
1- Pus for culture and sensitivity. It showed mixed growth of pseudomonas
aeroginosa and staphylococcus aureus.
2- Pure tone audiogram. It showed moderate to severe conductive deafness.

Fig. 8.2
X-ray mastoid (Laws view)

22

Clinical scenarios in oto-rhino-laryngology

3- Plain X-ray mastoid (Laws view). It showed haziness or opacification


in the mastoid region along with a soft tissue mass in the external auditory
canal (fig. 8.2).
4- CT scan of the mastoid bone. This was not done because of un-affordibility
by the patient.
Diagnosis:
This was a case of aural polyp, a complication of chronic suppurative
otitis media most likely tubo-tympanic type.
Treatment:
The patient was planned for aural polypectomy and mastoid exploration
under general anaesthesia. The pedicle of the polyp was lying deep, it was
hold and cut by a crocodile forceps and the polyp removed completely.
Tympanic membrane was found to have a large sized central perforation.
Through a post-aural approach mastoid antrum was opened and cortical
mastoidectomy was done. Disease was cleared from the mastoid antrum and
mastoid air cells. Myringoplasty was also done at the same time by using
temporalis fascia graft. Post-operative recovery was uneventful.
Discussion:
Chronic suppurative otitis media is the chronic inflammation of mucosa
of the middle ear cleft. This is conventionally divided into two main clinical
types:
1- Tubo-tympanic type
2- Attico-antral type
Tubo-tympanic type is virtually always a complication of acute
suppurative otitis media. It is the safe variety and relatively more common
than the attico-antral type. Serious complications are rare in tubo-tympanic
type. With prolonged discharge a polyp may form in the middle ear and
comes out through the perforation. Some times polyp is so large that it come
out through the external auditory meatus like in this case. Polyp is formed
because of extensive oedema in the mucous membrane as a result of chronic
inflammation.

TEST YOURSELF

Read the clinical scenario given at the beginning and answers the following questions
123455-

What is the most likely diagnosis in this case?


How will you manage this case?
See fig. 8.1 and describe its findings.
See X-ray in fig. 8.2 and describe its findings.
What are the different types of chronic suppurative otitis media?
What is an aural polyp? and how it is formed?
23

Clinical scenarios in oto-rhino-laryngology

Case 9
Clinical Scenario
A 24 years old male patient came in OPD with the complaint of thick, foul
smelling and occasional blood stained discharge from his right ear for last one year. He
also had marked hearing impairment and for last few days he had moderate to severe
pain in his right ear.
Important points in history taking:
1- Detailed history about discharge from the ear i.e. onset, continuous or
intermittent, amount, colour, foul smelling, blood stained, aggravating
and relieving factors etc.
2- Detailed history regarding deafness and pain in the ear. Deafness was
present since the start of discharge but initially it was mild and it increased
gradually and now he has marked hearing loss. Regarding pain, it occurred
off and on and relieved by taking medicines but now for last few days
it is continuous and moderate to severe which was not relieved by taking
analgesic drugs.
3- Any history of fever, headache, altered conciousness, vomiting, neck
stiffness or any other neurological symptom. In this case there was no
such history.
Important points in clinical examination:
1- General physical examination. It showed that the patient was a young,
average built person sitting comfortably and fully oriented in time, space
and person.
2- Examination of the discharge. It was present in the external auditory
canal, scanty in amount, thick yellowish in colour, purulent, foul smelling
and blood stained. Pus was cleaned completely from external auditory
canal and sent for culture and sensitivity.
3- Examination of the tympanic membrane. It showed postero-superior
marginal perforation with whitish material (most likely cholesteatoma).
Granulation tissues were also seen anterior to it. There was also
involvement of the attic region or pars flaccida (fig. 9.1)
4- Examination of the post-aural sulcus and mastoid region for any swelling,
redness, sinus or tenderness etc. These areas were appeared to be within
normal limits.
5- Voice test. It showed moderate degree of hearing loss.
6- Tuning fork tests. Rinnes test was negative in right ear and positive in
left ear. Webers test was lateralized towards the right side and Schwabachs
test was equal to the examiner in both ears.
7- Fistula test. It was negative in both the ears.
24

Clinical scenarios in oto-rhino-laryngology

Fig. 9.1
Tympanic membrane showing posterosuperior marginal perforation with
cholesteatoma and granulation tissues

8- Vestibular function tests. All appeared to be within normal limits.


9- Examination of the facial nerve. It was found to be intact.
Investigations:
1- Pus for culture and sensitivity. It showed heavy growth of pseudomonas
aeroginosa.
2- Pure tone audiogram. It showed moderate conductive hearing loss (about
40-50 dB) on the right side (fig. 9.2)
3- Plain X-ray mastoid (Laws view). Showed a lytic lesion (most likely
cholesteatoma) in the mastoid bone (fig. 9.3).
4- CT scan of the temporal bone and brain. This was not done because of
un-affordibility by the patient.
Diagnosis:
This was a case of chronic suppurative otitis media most likely with
cholesteatoma (attico-antral type), causing bone and ossicular erosion.
Differential Diagnosis:
1- Tubo-tympanic type of chronic suppurative otitis media.
2- Chronic suppurative otitis media with complication.

Fig. 9.2
Pure tone audiogram showing moderate
conductive hearing loss

25

Clinical scenarios in oto-rhino-laryngology

Fig. 9.3
Plain X-ray mastoid (Laws view)
showing cholesteatoma

Treatment:
The patient was planned for mastoid exploration under general
anaesthesia. Through post-aural approach mastoid antrum was opened.
Extensive cholesteatoma was found involving middle ear, additus, mastoid
antrum and mastoid air cells. Modified radical mastoidectomy (canal wall
down) was done and all the disease was removed. Long process of incus was
found to be necrosed by the cholesteatoma but other ossicles were found
intact. Tympanoplasty was also done at the same time. Post-operative recovery
was un-eventful.
Discussion:
Chronic suppurative otitis media is the chronic inflammation of mucosa
of the middle ear cleft. This is conventionally divided into two main clinical
types.:
1- Tubo-tympanic type
2- Attico-antral type
Tubo-tympanic type is virtually always a complication of acute
suppurative otitis media. It is the safe variety and relatively more common
than the attico-antral type. Attico-antral type is considred as dangerous variety
because of its aggresive nature and presence of cholesteatoma.
Cholesteatoma is a bag of stratified squamous epithelium which
contains keratin debris, shed epithelium and bacteria. It has tendency of
expansion and causing necrosis of the neighbouring structures and bones.
There are four theories for the formation of cholesteatoma:
1- Congenital cell rest theory
2- Metaplastic theory
3- In-growth of squamous epithelium theory
4- Retraction pocket theory
Cholesteatoma if not treated may give rise to following complications,
which are broadly classified into:
1- Extra-cranial complications:
a- Mastoiditis
b- Labyrinthitis
26

Clinical scenarios in oto-rhino-laryngology

cdefg-

Facial nerve paralysis


Petrositis
Otitis externa
Thrombosis of internal jugular vein
Chronic adhesive otitis media

2- Intra-cranial complications:
a- Extra-dural abscess
b- Sub-dural abscess
c- Brain abscess
d- Meningitis
e- Sigmoid sinus thrombosis
f- Otitic hydrocephalus

TEST YOURSELF

Read the clinical scenario given at the beginning and answers the following questions
12345678-

What is the most likely diagnosis in this case?


What are the differential diagnosis in this case?
How will you investigate this case?
How will you manage this case?
See fig. 9.1 and describe its findings.
See pure tone audiogram in fig. 9.2 and describe its findings.
See X-ray in fig. 9.3 and describe its findings.
What is cholesteatoma? and how it is formed?

27

Clinical scenarios in oto-rhino-laryngology

Case 10
Clinical Scenario
A 33 years old male patient came in OPD with complaint of asymmetry of his
face for the last 2 weeks. On examination he had facial nerve paralysis involving the
whole left side (fig. 10.1). On inquiry he also complained of discharge from the left ear
for last one year.

Fig. 10.1
Patient with left sided facial nerve
paralysis

Important points in history taking:


1- Detailed history regarding onset, progression, associated factors,
aggravating and relieving factors etc. Facial paralysis started insidiously
about two weeks back. Initially it was very slight and the patient ignored
it but gradually it increased to its present condition. There was no
aggravating and relieving factor with no significant associated factor.
2- Any history of ear disease or discharge from the ear. He had history of
discharge and deafness from his left ear for the last one year. Discharge
was usually scanty, thick, foul smelling and often blood stained. He has
never consulted to a doctor for his ear complaints.
3- Any history of trauma or head injury. There was no such history.
4- Any history of pain in the ear or headache. There was no history of pain
or headache in this patient.
5- Any history of change in taste sensations on the tongue. He has not
noticed any change.
6- Any history of vertigo. No history of vertigo.
7- Motor or sensory weakness in any other part of the body or limbs. There
was no such problem.
Important points in clinical examination:
1- General physical examination. It showed that the patient was a young
adult of average built, sitting comfortably and fully oriented in time,
28

Clinical scenarios in oto-rhino-laryngology

space and person. His vital signs were all within normal limits.
2- Examination of motor part of the facial nerve. There was whole left sided
facial paralysis (fig. 10.1).
3- Examination of the ears. There was scanty, thick and foul smelling
purulent discharge in the left external auditory canal. After cleaning the
pus, tympanic membrane was examined. It showed a large posterosuperior marginal perforation with some whitish material in the middle
ear (probably cholesteatoma). Right ear was within normal limits.
4- Examination of the post-aural region. There was no significant finding
and this region was normal.
5- Voice test, showed moderate degree of hearing loss of the left side.
6- Tuning fork tests. Rinnes test was negative in the left ear and positive
in right ear. Webers test was lateralized towards the left side and
Schwabachs test was equal to the examiner in both ears.
7- Fistula test. It was negative in both the ears.
8- Vestibular function tests. All appeared to be within normal limits.
9- Taste sensations on the tongue. It showed a decrease in taste sensations
on the left anterior two-third of the tongue.
10- Examination of the nose and throat. These regions were within normal
limits.
Investigations:
1- Pus for culture and sensitivity from the left external auditory canal. It
showed heavy growth of pseudomonas aeroginosa.
2- Pure tone audiogram. It showed moderate to severe conductive deafness
on the left side with normal hearing threshold on the right side.
3- Plain X-ray mastoid (Laws view). It showed haziness in the mastoid
region along with bony erosion and cavity formation (finding consistant
with a cholesteatoma).
4- CT scan of the mastoid bone. This was not done because of un-affordibility
by the patient.
5- Electro-diagnostic tests for facial nerve like minimal nerve excitability
test, electromyography and electroneuronography. These tests were
not done because of un-affordibility by the patient.
Diagnosis:
This was a case of facial nerve paralysis as a complication of atticoantral type of chronic suppurative otitis media (CSOM with cholesteatoma).
Treatment:
The patient was planned for mastoid exploration and facial nerve
decompression under general anaesthesia. Through a post-aural approach
mastoid antrum was opened. Extensive cholesteatoma was present in the
mastoid antrum and mastoid air cells. All the cholesteatoma was cleared
from these area. The bridge was lowered and modified radical mastoidectomy
was done. Cholesteatoma was found to be causing facial canal erosion and
29

Clinical scenarios in oto-rhino-laryngology

compression of the facial nerve at its second genu. All the disease cleared
from this area and the facial canal was opened around the site of erosion.
Nerve was found to be intact so it was covered with a temporalis fascia graft.
Post-operative recovery was un-eventful and facial nerve showed improvement.
Discussion:
The facial nerve is the seventh cranial nerve and it is a mixed nerve
contains motor, sensory and secretomotor fibres. The cause of facial nerve
paralysis is classified into:
1- Supra-nuclear paralysis: In this type, only the lower half of the face is
affected while the upper half escapes paralysis. It is because of the reason
that facial nucleus receives fibres from both side of the cortex controlling
upper half of the face.
2- Nuclear paralysis: The facial motor nucleus is affected and the clinical
picture is similar to that of infra-nuclear type.
3- Infra-nuclear paralysis: The whole side of the face is affected along with
other structures supplied by the facial nerve. According to the site of
involvement it is further classified into:
i- Intra-cranial
ii- Intra-temporal
iii- Extra-temporal or Extra-cranial
The facial nerve is intimately related with the ear, so many ear diseases
can cause facial nerve paralysis. Cholesteatoma because of its bone erosion
capability, is one of the important cause of facial nerve paralysis.

TEST YOURSELF

Read the clinical scenario given at the beginning and answers the following questions
1234-

Look at fig. 10.1 and state facial nerve paralysis is present on which side of the face?
How will you manage this case?
Give the classification of facial nerve paralysis.
What are the important causes of otogenic facial nerve paralysis?

30

Clinical scenarios in oto-rhino-laryngology

Case 11
Clinical Scenario
A 24 years adult male patient came in the OPD with complaint of recurrent
discharge from his right ear for many years. He was fond of swimming and developed
recurrent ear discharge whenever water went into his right ear during swimming. He
has to take antibiotic for few days to make his ear dry. At the time of presentation his
ear was dry and otoscopic picture of his right ear is shown in fig. 11.1.

Fig. 11.1
Dry small central perforation involving
antero-inferior quadrant of pars tensa

Important points in history taking:


1- Detailed history about the first onset of symptoms i.e how it started? In
this patient he did not remember about the first episode of discharge from
the ear and how it started.
2- Detailed history regarding discharge i.e. how often, amount, colour, foul
smelling, blood stained etc. In this patient ear remained dry for most of
the time and discharge usually started after swimming and becames dry
again after taking medicines for few days.
3- Detailed history about the hearing impairment and other ear symptoms.
In this case he has slight feeling of hearing impairment and occassional
tinnitus on right side.
4- Any history of nasal or throat problem/symptom. In this case there was
no symptom related with nose or throat.
Important points in clinical examination:
1- Assessment of perforation. In this patient, perforation was small, rounded,
central and dry involving antero-inferior quadrant of pars tensa (fig.
11.1). There was no discharge.
2- Voice test. He heared whisper in both ears i.e. hearing appears to be
within normal limits.
31

Clinical scenarios in oto-rhino-laryngology

3- Tuning fork tests. Rinnes test was positive in both ears, Webers test
was lateralized towards the right side and Schwabachs test was equal
to the examiner on both sides.
4- Any signs of middle ear infection. In this patient ear was completely dry
with no congestion of tympanic membrane. Middle ear mucosa as seen
through the perforation also appeared normal.
5- Any pathology in the nose and throat. In this case both nose and throat
were within normal limits with no significant finding.
Diagnosis:
This was a case of small and dry central perforation of pars tensa.
Investigations:
1- Pure tone audiometry. It showed a conductive loss of 15 to 20 dB especially
in the lower frequencies (fig 11.2).
2- Pus for C/S is done when the discharge is present. This patient when
presented has a dry ear so C/S was not done.
3- Other baseline investigations for general anaesthesia like blood complete
picture, ESR, random blood sugar, urine D/R and X-ray chest (PA view).
All were within normal limits in this patient.

Fig. 11.2
Pure tone audiogram of the right ear

Treatment:
Patient was planned for myringoplasty under general anaesthesia.
As this was a small perforation, fat-plug myringoplasty was planned. Margins
of the perforation were freshened by a needle and fat plug taken from the
lobule was tucked in the perforation. Spongestone was applied all around
and external auditory canal was packed with ribbon guaze soaked in BIPP.
Post-operative recovery was uneventful and perforation healed completely
(fig. 11.3).
Discussion:
The causes of dry perforation of pars tensa includes:
1- Previous acute otitis media with perforation where healing is incomplete
resulting in persistant small perforation.
32

Clinical scenarios in oto-rhino-laryngology

Fig. 11.3
Tympanic membrane at 6 weeks after
myringoplasty

2- Persistant perforation after myringotomy and grommet insertion.


3- Tubo-tympanic type of chronic suppurative otitis media where infection
had been settled with medical or surgical treatment but perforation remains
there.
4- Traumatic perforation where healing is incomplete.
Many patients live with small tympanic membrane perforation that
is entirely without symptoms. Symptoms of small and dry perforation include
audible whistling sounds during sneezing and nose blowing, decreased
hearing and a tendency to infection during cold or when water goes into the
ear. Hearing loss in small perforation is usually mild and may be unnoticeable
by the patients. In swimmers, divers and other water sports enthusiasts,
recurrent ear infection may occur. Medical therapy for small pars tensa
perforation is directed at controlling ear infection when discharge is present
by giving appropriate systemic antibiotics, aural toilet and antibiotic ear
drops. Surgical treatment is repair of the tympanic membrane (myringoplasty).
The options for myringoplasty in such a case are:
1- Overlay or underlay myringoplasty with graft material like temporalis
fascia. But these two techniques of myringoplasty are more suitable for
medium size to large perforations.
2- Fat-plug myringoplasty is very suitable for small perforations like this.
3- Repeated cauterization of margins of the perforation with chemicals.

TEST YOURSELF

Read the clinical scenario given at the beginning and answers the following questions
12345-

What are the important points in history taking and clinical examination in this patient?
What investigations will you order in such a case?
How will you manage this patient?
What are the different options for myringoplasty?
What are the advantages of fat-plug myringoplasty?
33

Clinical scenarios in oto-rhino-laryngology

Case 12
Clinical Scenario
A 17 years old male patient came with the complaint of impaired hearing from
the left ear for last 7 to 8 months. It was often associated with crackling and bubbling
noises with sensation of fluid in his left ear especially on changing head position. There
was no history of discharge or pain in the ear. On examination, otoscopic findings of
his left ear is shown in fig 12.1.

Fig. 12.1
Tympanic membrane of left side
showing fluid in the middle ear with
fluid level

Important points in history taking:


1- Detailed history regarding deafness. In this patient, deafness was gradual
in onset, continuous but fluctuating in severity and sound appeared quieter
but not distorted.
2- Any history of discharge or pain. In this case there was no history of
discharge or pain in his left ear.
3- Any symptom related with nose or throat. He had history of recurrent
nasal discharge and post-nasal dripping, which was usually relieved by
taking medicines for few days.
Important points in clinical examination:
1- Examination of the tympanic membrane. It showed fluid in the middle
ear with fluid level. Tympanic membrane appeared retracted and somewhat
congested with absence of cone of light (fig. 12.1).
2- Valsalvas manuever. There was no affect of Valsalvas manuever on the
tympanic membrane of left side i.e. eustachian tube patency was absent.
3- Siegels pneumatic otoscopy. It showed no movement of tympanic
membrane.
4- Voice test. It showed moderate degree of deafness on the left side.
5- Tuning fork tests. Rinnes test was negative on the left side and positive
34

Clinical scenarios in oto-rhino-laryngology

on the right side. Webers test was lateralized towards the left side and
Schwabachs test was equal to the examiner on both sides.
6- Examination of the nose and throat. It did not showed any significant
finding.
Investigations:
1- Pure tone audiogram. It showed moderate conductive hearing loss (3040 dB in the left ear (fig. 12.2). Right ear showed hearing within normal
limits.

Fig. 12.2
Pure tone audiogram of the left ear
showing moderate conductive hearing
loss

2- Speech audiogram. It showed 100% speech discrimination score on both


the side.
3- Tympanogram. It showed type B graph on the left side (fig. 12.3) and
type A graph on the right side.
4- Plain X-ray soft tissue nasopharynx (lateral view) and X-ray PNS (waters
view) for any pathology in the nasopharynx, nose and PNS and these
were within normal limits.
5- Other baseline investigations for general anaesthesia. All were within
normal limits.
Diagnosis:
This was a case of otitis media with effusion in the left ear.

Fig. 12.3
Tympanogram of the left side showing
type B graph

35

Clinical scenarios in oto-rhino-laryngology

Treatment:
Patient was planned for myringotomy and grommet insertion under
general anaesthesia. Radial incision was given with myringotomy knife in
the antero-inferior quadrant of the pars tensa. Mucoid fluid came out, which
was removed by suction and the grommet inserted (fig. 12.4). Post-operatively
antibiotic, analgesic/NSAID and systemic decongestant were given for 2
weeks. Patients symptoms improved markedly. Grommet was removed after
3 months.

Fig. 12.4
Tympanic membrane after
myringotomy and grommet insertion

Discussion:
The term otitis media with effusion or non-suppurative otitis media
is applied to the clinical condition characterized by the presence of nonpurulent fluid in the middle ear cleft. The fluid may be serous, mucoid or
sometimes hemorrhagic in nature.
Acute and chronic forms can sometimes be distinguished according
to the mode of onset or by duration, but the distinction may not always be
clear and the condition is often recurrent. In acute form of otitis media with
effusion, duration of the disease is less than 3 weeks and in chronic form
duration is longer than 3 months.
From time to time different terminologies have been proposed for
this disease. In 1886 Politzer first described the term otitis media catarrhalis.
After the end of the second world war the term glue ear was first introduced
by Jordan. The other commonly used synonyms for this condition are secretory
otitis media, serous otitis media, mucinous otitis media, catarrhal otitis media,
exudative otitis media and glue ear. It is considered as the most important
cause of deafness in the children world over.
The exact aetiology of this condition is unknown. The following
factors are described as the aetiological factors for this condition.
1- Occlusion of the eustachian tube
2- Allergy
3- Viral infection
4- Unresolved acute otitis media
5- Cleft palate
36

Clinical scenarios in oto-rhino-laryngology

TEST YOURSELF

Read the clinical scenario given at the beginning and answers the following questions
12345678-

What is your diagnosis in this case?


What investigations will you order in this case?
How will you treat this case?
Briefly outline the pathology of otitis media with effusion.
Describe the findings of tympanic membrane as shown in fig. 12.1.
Describe the findings of pure tone audiogram in fig. 12.2.
Which type of tympanogram graph is seen in fig. 12.3.
What is the site of incision for myringotomy in cases of otitis media with effusion?

37

Clinical scenarios in oto-rhino-laryngology

Case 13
Clinical Scenario
A 28 years old female patient presented with the complaint of impaired hearing
from both ears for the last 10 to 12 years. She had more problem in hearing in calm and
quiet environment as compared to noisy places. There was no history of pain or discharge
from the ears. On examination her external auditory canal and tympanic membrane
were appeared normal in both ears.
Important points in history taking:
1- Detailed history about deafness i.e. onset, progression, associated factors,
aggravating and relieving factors. According to patient onset was insidious,
initially hearing impairment was mild but it increased gradually to the
current situation. Deafness was continuous with no fluctuation in severity,
with no agrravating or relieving factor. Occasionally it was associated
with ringing sensations in both ears.
2- Hears better in quiet room or in noisy places. In this patient she had better
hearing in noisy places (paracussis Wallisi).
3- Age of onset. In this patient complaint had started when she was about
16 to 17 years of age.
4- Effect of pregnancy on hearing. She had two childrens and according to
her, deafness had increased during pregnancy in both the times.
5- History of deafness in the family. According to patient her mother and
her elder sister had also similar complaints.
6- Any history of pain or discharge from the ears. In this case there was no
history of pain or discharge from the ears.
7- Any complaint related with the nose or throat. In this patient there was
no such complaint.
Important points in clinical examination:
1- General physical examination. She is a young, fair coloured lady of
average built and fully oriented.
2- Examination of the external ear, external auditory canal and tympanic
membrane, all appear to be within normal limits.
3- Voice test, showing moderate degree of hearing loss.
4- Tuning fork tests. Rinnes test was negative in both the ears, Webers test
was centralized and Schwabachs test was equal to the examiner in both
ears.
5- Valsalvas maneuver and Siegels pneumatic otoscopy. It showed patent
eustachian tube and tympanic membrane was mobile.
6- Vestibular function tests, all were within normal limits.
7- Examination of the nose and throat. No significant finding on it.
38

Clinical scenarios in oto-rhino-laryngology

Investigations:
1- Pure tone audiogram. It showed moderate degree of conductive deafness
in both ears. There was a dip in bone conduction at 2000 Hz (Carharts
notch) in both ears (fig. 13.1 and 13.2).
2- Speech audiogram. Speech discrimination score was 98% in right ear
and 96% in the left ear.
3- Tympanogram, showing type As graph in both the ears (fig. 13.3 and
13.4).

Fig. 13.1
Pure tone audiogram of right ear

Fig. 13.2
Pure tone audiogram of left ear

Fig. 13.3
Tympanogram of right ear

39

Clinical scenarios in oto-rhino-laryngology

Fig. 13.4
Tympanogram of left ear

Diagnosis:
This was a case of otosclerosis involving both ears.
Differential Diagnosis:
The other causes of conductive deafness due to pathology in the
middle ear, should be considered in differential diagnosis, like:
1- Congenital fixation of the footplate of stapes. Deafness is present since
birth in this condition.
2- Adhesive otitis media. Tympanic membrane will not appear normal
looking and mobile.
3- Tympanosclerosis. White chalky patches are seen on tympanic membrane.
4- Ossicular dislocation. Type AD tympanogram will be seen in this condition.
Treatment:
The patient was planned for stapedotomy and teflon piston insertion
under general anaesthesia on the right side first. Rosens incision was given
and tympanomeatal flap raised. Chorda tympani nerve was reflected
downwards and postero-superior bony buttress removed. Tendon of stapedius
muscle was cut and supra-structure of stapes removed after fracture. A hole
is made in the footplate of stapes and teflon piston (Sheas piston) was

Fig. 13.5
Stapedotomy operation with insertion
of teflon piston

40

Clinical scenarios in oto-rhino-laryngology

inserted (fig. 13.5). Post-operative recovery was uneventful with improvement


in hearing.
Discussion:
Otosclerosis or Otospongiosis is a localized disease of the otic capsule.
There is formation of new spongy bone, which causes ankylosis of the footplate
of stapes to the margins of the oval window or may invade to involve the
cochlea. This process occurs in the endochondral layer of the bony otic capsule.
These bony changes occur at one or more constant sites of otic capsule. The
commonest site is anterior to the oval window (fissula antefenestrum) causing
ankylosis of the footplate of stapes to the margins of oval window. Abnormal
bone may be present at other sites of otic capsule but usually causes no clinical
manifestation.

TEST YOURSELF

Read the clinical scenario given at the beginning and answers the following questions
12345678-

What is the most likely diagnosis in this case?


What are the differential diagnosis in this case?
How will you investigate this case?
How will you treat this case?
What are the findings on pure tone audiogram in fig 13.1 and 13.2?
What are the findings on tympanogram in fig. 13.3 and 13.4?
Outline the steps of stapedotomy operation.
What is the pathology of otosclerosis?

41

Clinical scenarios in oto-rhino-laryngology

Case 14
Clinical Scenario
A 68 years old retired bank manager came with the complaint of progressive
bilateral hearing impairment for the last few years. He had marked difficulty in
understanding speech especially in the presence of background noises. On examination
his external ears and tympanic membranes appeared to be normal.
Important points in history taking:
1- Detailed history about deafness i.e. its onset, continuous or intermittent,
progressive or not, unilateral or bilateral, hears better in noisy places or
in quiet room, associated factors. In this case patient had bilateral,
progressive hearing loss and difficulty in understanding speech in the
presence of background noises (this is typically seen in cases of sensorineural deafness).
2- Any history of discharge from the ears. In this case there was no history
of discharge from the ears.
3- Any history of diabetes mellitus. In this case no history of diabetes
mellitus.
4- Occupational history and exposure to loud sound. In this case he was a
retired bank manager and there was no history of any occupational noise
exposure or sudden sound like bomb blast or firearm.
5- Any history of using any ototoxic drug. In this case there was no history
of using any drug having such affects.
Important points in clinical examination:
1- Examination of the external ear, external auditory canal and tympanic
membrane. All appeared to be within normal limits.
2- Voice test. It showed moderate degree of hearing loss.
3- Tuning fork tests. Rinnes test was positive in both the ears, Webers test
was centralized and Schwabachs test was less than the examiner in both
ear (it means patient has bilateral and equal sensori-neural loss).
4- Vestibular function tests. All were within normal limits.
5- Examination of the nose and throat. No significant finding on it.
Investigations:
1- Pure tone audiogram. It showed moderate to severe slopping type (i.e.
more pronounced in higher frequencies) sensori-neural deafness in both
ears (fig. 14.1 and 14.2).
2- Speech audiogram. Speech discrimination score was 69% in right ear
and 72% in the left ear.
3- Tympanogram, showed type A graph in both the ears.
42

Clinical scenarios in oto-rhino-laryngology

Fig. 14.1
Pure tone audiogram of right ear

Fig. 14.2
Pure tone audiogram of left ear

Diagnosis:
This was a case of bilateral sensori-neural hearing loss, most probably
senile deafness or presbyacusis.
Differential Diagnosis:
The other causes of sensori-neural deafness should be considered in
differential diagnosis.
Treatment:
This was a case of bilateral moderate to severe sensori-neural hearing
loss because of ageing, so following treatment was offered to the patient:
1- Hearing aid. Patient was fitted with behind the ear type (BTE), digital
programmable hearing aids in both the ears (fig. 14.3).
2- Lip reading and auditory training.
Discussion:
The term presbyacusis or senile deafness is used to described hearing
loss resulting from degenerative changes due to ageing. Deafness is
characteristically bilateral, symmetrical and slowly progressive. It affects both
43

Clinical scenarios in oto-rhino-laryngology

Fig. 14.3
Behind the ear (BTE) type hearing aid

the sexes equally. Degenerative changes occur as a result of vascular


insufficiency due to sclerosis, thrombosis or atherosclerosis and involves:
1- Hair cells in the organ of Corti
2- Neural tissues in the spiral ganglion
3- Stria vascularis
4- Basal membrane

TEST YOURSELF

Read the clinical scenario given at the beginning and answers the following questions
12345-

What is the most likely diagnosis in this case?


What are the differential diagnosis in this case?
How will you investigate this case?
How will you manage this case?
What is senile deafness or presbyacusis?

44

Clinical scenarios in oto-rhino-laryngology

Case 15
Clinical Scenario
A 48 years old textile factory worker came with the complaint of impaired
hearing from both ears for the past many years. He had marked difficulty in understanding
speech especially in noisy places. There was no history of pain or discharge from the
ears but occassionally he also had ringing sounds in both ears. On clinical examination
pinna, external auditory canal and tympanic membrane were all normal.
Important points in history taking:
1- Detailed history about deafness i.e. its onset, continuous or intermittent,
progressive or not, unilateral or bilateral, hears better in noisy places or
in quiet room, associated factors. In this case, deafness was insidious in
onset, continuous, progressive, bilateral and equal in both ears.
2- Occupational history and exposure to loud sound. He was a textile factory
worker and had been working on heavy machines with loud sound 8 to
10 hours a day for the past 30 years. There was no facility of any sort
for protection of the ears from loud sound in his factory.
3- Any history of diabetes mellitus. In this case no history of diabetes
mellitus.
4- Any history of trauma to the ears or head. There was no such history.
5- Any history of using any ototoxic drug. In this case there was no history
of using any drug having such affects.
Important points in clinical examination:
1- Examination of the external ear, external auditory canal and tympanic
membrane. All appeared to be within normal limits.
2- Voice test, showed moderate degree of hearing loss.
3- Tuning fork tests. Rinnes test was positive in both the ears, Webers test
was centralized and Schwabachs test was less than the examiner in both
ears (it means patient has bilateral and equal sensori-neural loss).
4- Vestibular function tests. All were within normal limits.
5- Examination of the nose and throat. No significant finding on it.
Investigations:
1- Pure tone audiogram. It showed moderate to severe slopping type (i.e.
more pronounced in higher frequencies) sensori-neural deafness in both
ears (fig. 15.1 and 15.2).
2- Speech audiogram. Speech discrimination score was 70% in right ear
and 74% in the left ear.
3- Tympanogram, showed type A graph in both the ears.
45

Clinical scenarios in oto-rhino-laryngology

Fig. 15.1
Pure tone audiogram of right ear

Fig. 15.2
Pure tone audiogram of left ear

Diagnosis:
This was a case of bilateral sensori-neural hearing loss, probably
noise induced hearing loss (NIHL).
Differential Diagnosis:
The other causes of sensori-neural deafness should be considered in
differential diagnosis.
Treatment:
This was a case of bilateral moderate to severe noise induced hearing
loss, so following treatment was offered to the patient.
1- Hearing aid. Patient was fitted with behind the ear type (BTE), digital
programmable hearing aids in both the ears.
2- Lip reading and auditory training.
3- Protection of the ears from loud sound during working, to prevent further
noise induced hearing loss.
Discussion:
Noise induced deafness or hearing loss (NIHL) is caused by prolong
exposure to loud sound. The degree of deafness is proportional to sound intensity
46

Clinical scenarios in oto-rhino-laryngology

and duration of daily exposure, though there is a marked variation in individual


susceptibility. OSHA (Occupational Safety and Health Administration, USA)
standard for the safe exposure to different sound intensity levels per day for five
days a week is listed below. These levels are referred to as the permissible
exposure level (PEL).
- 16 hours
85 dB
- 8 hours
90 dB
- 6 hours
92 dB
- 4 hours
95 dB
- 3 hours
97 dB
- 2 hours
100 dB
- 1.5 hours
102 dB
- 1.0 hour
105 dB
- 30 minutes
110 dB
- 15 minutes
115 dB

TEST YOURSELF

Read the clinical scenario given at the beginning and answers the following questions
123456-

What is the most likely diagnosis in this case?


What are the differential diagnosis in this case?
How will you investigate this case?
How will you manage this case?
What are the causes of noise induced hearing loss?
What is permissible exposure level (PEL) of sound?

47

Clinical scenarios in oto-rhino-laryngology

Case 16
Clinical Scenario
A 48 years old male patient came in OPD with the complaints of severe vertigo
for last one day. Along with vertigo he has ringing sounds and hearing loss in his right
ear. He had history of similar attacks, three or four times before during the last three
years, which improved by medication and he remained normal between the attacks.
Important points in history taking:
1- Detailed history about the vertigo. According to the patient it started
suddenly yesterday and since then it was continuous. There was no affect
of posture i.e. it remain same on standing, sitting or lying down. Patient
was feeling that all things around him was rotating. There was no specific
aggravating or relieving factor. It was associated with nausea and vomitting
as well.
2- Detailed history about hearing loss and tinnitus. According to him, both
symptoms started immediately after start of vertigo. Hearing loss was in
right ear, mild to moderate, continuous with no aggravating and relieving
factors. He has ringing sounds in his right ear, which was constant and
same with no aggravating or relieving factors.
3- History of similar attacks in the past. According to patient during the last
three years he had three to four similar attacks before which lasted for
a weak or so and completely improved by medication. There was no
complaint or symptom in between the attacks.
4- Any history of head trauma, headache or any other neurological symptom.
There was no such history.
5- Any history of using any vestibulo-toxic drug. In this case there was no
history of using any drug having such affects.
6- Any history of allergy. Patient has allergy to multiple things.
Important points in clinical examination:
1- General physical examination. The patient was a middle aged man of
average height and obese built, looking anxious but well oriented in time,
space and person. His vital signs were within normal range.
2- Examination of the external ear, external auditory canal and tympanic
membrane. All appeared to be within normal limits.
3- Voice test, showed moderate degree of hearing loss on the right side.
4- Tuning fork tests. Rinnes test was positive in both the ears, Webers test
was lateralized towards the left ear and Schwabachs test was less than
the examiner in right ear and equal to the examiner in left ear.
5- Fistula test. It was negative in both the ears.
6- Facial nerve examination. It was intact.
48

Clinical scenarios in oto-rhino-laryngology

7- Spontaneous nystagmus. It was positive with the fast component towards


the left side.
8- Gait. Patient deviated towards the right side when he was walking with
eyes closed.
9- Rombergs test. Patient swayed on the right side when he was standing
with eyes closed.
10- Cerebellar functions tests like finger nose test, rapid alternating movements
(dysdiadochokinesia) and rebound phenomenon, were all within normal
limits.
11- Examination of other cranial nerves. All were intact.
12- Examination of the nose and throat. No significant finding on it.
Investigations:
1- Pure tone audiogram. Showing moderate sensori-neural type of deafness,
equal in all frequencies on the right side and normal hearing on the left
side (fig. 16.1).
2- Speech audiogram. Speech discrimination score was 70% in right ear
and 96% in the left ear.
3- Recruitment test. It was positive on the right side, showing a cochlear
lesion.
4- Caloric test. It was not done because of acute attack of severe vertigo.
Diagnosis:
This was a case of Meneires disease.
Differential Diagnosis:
The other causes of vertigo should be considered in differential
diagnosis, like:
1- Vestibular neuronitis. It usually follows an attack of influenza and hearing
remains normal in this condition.
2- Labyrinthitis. It is mostly a complication of chronic suppurative otitis
media or ear surgery. Sometimes labyrinthitis of viral origin can occur.

Fig. 16.1
Pure tone audiogram of right ear

49

Clinical scenarios in oto-rhino-laryngology

3- Benign paroxysmal positional vertigo (BPPV). In this condition vertigo


only occurs in certain specific head positions.
4- Central causes of vertigo.
Treatment:
During the acute attack of vertigo, patient was advised strict bed rest
and antivertiginous drugs or labyrinthine sedatives were prescribed. In
addition patient was advised for:
1- Low salt diet
2- Avoid excessive intake of water
3- Quit smoking
4- Avoid stress and bring a change in life style
5- Avoid things that are causing allergic reaction
The patient became symptom free in ten days. Regular follow-up
was advised and after taking above mentioned precautions, there was no
further attack of vertigo in the follow-up period of last one year.
Discussion:
Menieres disease is the disorder of endolymphatic labyrinth. It is
characterized by sudden paroxysmal attacks of vertigo, deafness and tinnitus. It
involves both the cochlear and the vestibular components of the inner ear.
The most consistent histological finding in Menieres disease is
dilatation of the endolymphatic compartment of the inner ear. The basic
defect is either over production or diminished absorption of endolymph. As
a result of this scala media is distended with endolymph. Due to over distension
of scala media, rupture of Reissners membrane occurs. This leads to mixing
of endolymph and perilymph, which disturbs the cochlear microphonics and
action potentials of the nerves. This occurs because endolymph is a potassium
rich fluid and perilymph is a sodium rich fluid. The attack continues till the
ruptured membrane is healed and local biochemistry is corrected. The same
type of attack occurs again when the scala media is over distended.

TEST YOURSELF

Read the clinical scenario given at the beginning and answers the following questions
12345-

What is the most likely diagnosis in this case?


What are the differential diagnosis in this case?
How will you investigate this case?
How will you treat this patient?
What is the pathology of Meneires disease and why it occurs in paroxysmal attacks?

50

Clinical scenarios in oto-rhino-laryngology

Case 17
Clinical Scenario
A 46 years old female patient came with the complaints of repeated vertigo,
nausea and occasional vomiting, lasting for 2 to 3 minutes for last 3 months. The vertigo
was exacerbated whenever she turned her head to the left while lying in bed or rising
from the bed. She has no history of deafness and tinnitus.
Important points in history taking:
1- Detailed history regarding vertigo. According to the patient, her symptoms
started after a road traffic accident when she got minor head injury. She
has feeling of rotation whenever she turned her head to the left while
lying down or rising from the bed. There was no problem on standing,
sitting or walking. She also has feeling of nausea and occasionally
vomiting during the attack of vertigo which remained there for few
minutes and she became normal after few minutes. The symptoms were
constant and same since starting with no aggravating or relieving factors
except head movement.
2- Any history of discharge, deafness, tinnitus or any symptom related with
the ear. There was no other complaint.
3- Any history of neck pain or stiffness. There was no such history but
occasionally she felt dizziness on rapid head movement.
4- Any history of trauma or head injury. There was a definite history of road
traffic accident and head injury and all the symptoms started after that.
5- Past medical history for diseases like hypertension, diabetes mellitus,
syphillis etc. There was no history of such diseases.
6- Any other neurologic complaint. There was no such history.
7- Drug history. She has used different medicines for these complaints with
no relief and she did not know their names. There was no history of using
any drug before the start of these symptoms.
8- Any history of psychiatric illness. There was no such history.
Important points in clinical examination:
1- General physical examination. The patient was a middle aged woman of
average height and built, sitting comfortably and fully oriented. Her vital
signs were within normal limits and no other positive finding on general
physical examination.
2- Examination of the ears. Pinna, external auditory canal and tympanic
membrane were all normal on both the sides.
3- Voice test and tuning fork tests. All were within normal limits.
4- Examination of all cranial nerves. All were intact.
5- Examination for spontaneous nystagmus. It was absent.
51

Clinical scenarios in oto-rhino-laryngology

6- Gait without and with closed eyes. It was within normal limit.
7- Cerebellar function tests. All were within normal limits.
8- Dix-Hallpike test. There was nystagmus and feeling of vertigo when the
patient was lowered with head turned towards the left. The nystagmus
was delayed in onset and showed fatigability. There was no nystagmus
with head turned towards the right side.
Differential Diagnosis:
The first step in the differential diagnosis of a vestibular disorder is
to delineate between a peripheral or central lesion. The following disorders
should be included in the differential diagnosis of this patient:
1- Benign paroxysmal positional vertigo
2- Traumatic injury to the labyrinth
3- Vestibular neuronitis
4- Labyrinthitis
5- Menieres disease
6- Perilymph fistula
7- Acoustic neuroma
8- Postural hypotension
9- Multiple sclerosis
10- Psychiatric illness or use of drugs like tranqulizers
Investigations:
1- Baseline investigations like complete blood picture, blood sugar, serum
electrolytes, renal function tests. All were within normal limits.
2- Pure tone and speech audiogram. Both were within normal limits.
3- Tympanogram. Type A graph was seen on both the sides.
4- Caloric test. It was within normal limits on both sides with cold and
warm water.
Diagnosis:
This was a case of benign paroxysmal positional vertigo (BPPV),
affected ear being the left side.
Treatment:
After all clinical workup, patient was planned for particle repositioning
procedure or Epleys maneuver. It involves sequential movement of the
head into different positions, staying in each position for roughly 30 seconds
(fig. 17.1). Initially patient was advised to sit on a couch (position 1). As the
affected ear was left, patients head was rotated to 45o on the left and then
head was lowered (position 2). Then keeping the patient supine, her head
was rotated 90o towards the right (position 3). Again after 30 seconds,
patients whole body was rolled or rotated towards right, so that her face
was 45o towards the ground (position 4). Patient was then advised to sit
again from this position (position 5). Patient responded well after this
maneuver and became symptom free.
52

Clinical scenarios in oto-rhino-laryngology

Fig. 17.1
Method of performing
Epleys maneuver

Discussion:
Benign paroxysmal positional vertigo (BPPV) is a common cause of
dizziness and about 20% of all cases of dizziness are due to this. This
incidence increases with age and in older people upto 50% of the cases of
dizzines are due to BPPV. In this condition there are recurrent, paroxysmal
and short-lived attacks of vertigo in certain head positions. Exact aetiology
of this condition is unknown but head injury is supposed to be an important
factor. Due to head injury otoconia or calcium crystals present in the utricle
are dislodged and settle in the posterior semi-circular canal. When the head
is reoriented relative to gravity, the gravity-dependent movement of the
heavier otoconial debris within the affected semicircular canal causes abnormal
endolymph displacement and a resultant sensation of vertigo.
The head position, which provokes the vertigo, should be avoided
and it is often all that is required. Role of antivertiginous drug is controversial
and may get relief in some patients. Many of the cases may settle spontaneously
within a few months but some may persist for years. There are two office
procedures to treat BPPV, both are intended to move otoconial debris from
the posterior semi-circular canal to some less sensitive areas. These include
Epleys maneuver and Semont maneuver. Surgical treatment is indicated
in cases of persistant disease for years not responding to any other treatment.
It include canal plugging, singulare nerve section, vestibular nerve section
and labyrinthectomy.

TEST YOURSELF

Read the clinical scenario given at the beginning and answers the following questions
12345-

What are the important points in history taking and clinical examination in this case?
How will you investigate this case?
How will you manage this case?
What is the pathophysiology of benign paroxysmal positional vertigo?
Describe the method of Epleys maneuver.
53

Clinical scenarios in oto-rhino-laryngology

Case 18
Clinical Scenario
A mother brought her one year old son with the complaint that he did not pay
attention to any kind of sound and his condition was same since birth. She thought that
he was deaf and she came here for assessment and consultation.
Important points in history taking:
1- Detailed history regarding presenting complaints. According to his mother
he did not respond to any sound stimuli and she thought that he was
completely deaf.
2- History about the development of the child and other milestones. All
other milestones were normal and otherwise he was perfectly normal.
3- History during pregnancy for any disease, use of drugs, viral infections,
exposure to radiation etc. In this case there was no significant history
during pregnancy.
4- Detailed history about his birth like type of delivery, time duration of
delivery, hypoxia of the child during or after delivery, prematurity of the
child, birth weight of the child etc.
5- Detailed history of events after child birth like, jaundice, Rh factor
incompatibility, hypothyroidism etc.
6- Consanguinity, whether his mother and father are cousins or close relatives.
In this case his mother and father were first cousins.
7- Condition of the other siblings. He has one elder sister and one brother
and both were absolutely normal.
8- History of congenital deafness in the other family members and close
relatives of both maternal and paternal side. In this case there was history
of congenital deafness in many persons among close relatives of both
maternal and paternal side.
Important points in clinical examination:
1- General physical examination. The child was active and otherwise normal
looking according to his age.
2- Inspection of the pinna, external auditory canal and tympanic membrane.
All were normal.
3- Distraction test. There was no response of the child in distraction test,
means he was not listening to the sound stimuli.
4- Other ENT examination. It was within normal limits.
5- Other systemic examination for any other abnormality in the body. In
this case no other abnormality was detected.
Investigations:
1- Brainstem evoked response audiometry (BERA). No reproducible wave
54

Clinical scenarios in oto-rhino-laryngology

form pattern was obtained even on maximum sound stimuli, showing


profound deafness in both the ears.
2- CT scan of the whole ear and mastoid area of both sides. No significant
positive finding was seen on CT scan.
Diagnosis:
The patient was a case of congenital deafness which was profound
and bilateral. The most probable cause may be genetic because of consanguinity
and similar cases in the family.
Treatment:
After all clinical workup, patient was planned for cochlear implant.
Surgery was done successfully and post-operative recovery was uneventful.
After cochlear implant auditory rehabilitation was started and patient showed
good improvement.
Discussion:
Around the globe, deafness is a common public health problem and
it is estimated that 200 million people on earth are suffering from this problem.
Majority of them belong to the developing countries. The causes of congenital
deafness are classified into genetic and non-genetic or environmental.
Worldwide severe to profound deafness affect 1 in 2,000 newborns and half
of them have genetic etiology. Inherited deafness is more common in
population where consanguineous marriages are common like in Pakistan.
The only option for treatment in severe to profound bilateral deafness
is cochlear implant. Cochlear implant is the device which performs the
function of cochlea i.e. it converts sound energy into electrical impulses and
stimulates the cochlear nerve fibres. At present cochlear implant is indicated
in a patient having bilateral profound sensori-neural deafness with no
measurable speech discrimination and even the most powerful hearing aid
is not effective. It is most useful in post-lingually deaf adults and children
i.e. where hearing loss occur after acquisition of speech. Pre-lingually deaf
adults may also get benefit of cochlear implant. Congenitally deaf children
get minimum benefit from cochlear implant.

TEST YOURSELF

Read the clinical scenario given at the beginning and answers the following questions
123456-

What are the important points in history taking in this case?


What are the important points in clinical examination in this case?
How will you investigate this case?
How will you manage this case?
What is a cochlear implant and how it works?
Which type of patient can get the maximum benefit from cochlear implant?
55

Clinical scenarios in oto-rhino-laryngology

Section II

NOSE & PARANASAL SINUSES


Case 19Case 20Case 21Case 22Case 23Case 24Case 25Case 26Case 27Case 28Case 29Case 30-

Deviated nasal septum


Nasal trauma with fracture
Antro-choanal polyp
Septal haematoma
Ethmoidal polypi
Septal adhesion
Allergic rhinitis
Foreign body of the nose
Boil in the nose
Epistaxis
Chronic rhino-sinusitis
Nasopharyngeal angiofibroma

57

59
62
66
70
73
76
78
81
84
86
89
92

Clinical scenarios in oto-rhino-laryngology

Case 19
Clinical Scenario
A 32 years old male patient came in the OPD with the complaint of nasal
obstruction for the last 3 years. It was associated with post-nasal dripping and thick
nasal discharge off and on. Nasal obstruction was almost continuous and more on the
left side. Anterior rhinoscopy with nasal speculum showed deviated nasal septum on
the left side (fig 19.1).

Fig. 19.1
Anterior rhinoscopy with nasal
speculum showing deviated nasal
septum

Important points in history taking:


1- Effect of posture on nasal obstruction. On lying down dependent nasal
cavity was blocked and patient was unable to lie on his right lateral
position.
2- History of excessive sneezing and watery rhinorrhoea. These were absent
in this case.
3- Facial pain and headache. These were absent in this case.
4- Other symptoms of allergy like itching or watering of the eyes, skin
rashes etc. All were absent.
5- Aggravating and relieving factors. Nasal obstruction and post-nasal
dripping were aggravated by exposure to cold and relieved by taking
medicines from local general practitioner.
6- Previous history of trauma to nose. There was a history of fall from the
stairs and trauma on the nose many years back.
Important points in clinical examination:
1- External inspection of the nose for deformity. Slight deviation of tip of
the nose was present towards the left side.
2- Anterior rhinoscopy. It showed a deviated nasal septum with convexity
59

Clinical scenarios in oto-rhino-laryngology

on the left side and a sharp spur on the left side (fig. 19.1).
3- Nasal patency test. It was reduced on the right side and absent on the
left side.
4- Palpation of the sinuses for tenderness. Not tender in this case.
5- Posterior rhinoscopy. Nothing significant found on this examination in
this case.
6- Examination of the throat and ears for any pathology related with his
nasal disease.
7- Rigid nasal endoscopy under local anaesthesia in the OPD. It showed
gross deviation of the nasal septum on left side with a spur. There is also
hypertrophy of the inferior turbinate on the right side. At the time of this
examination no muco-purulent or purulent secretion seen in nasal cavity
or in any meati.
Diagnosis:
This was a case of deviated nasal septum. History was also suggestive
of recurrent rhino-sinusitis, but at the time of presentation there were no
symptoms or signs of infection.
Investigations:
1- Blood CP: For total and differential leucocyte count, TLC was 13,000/mm3,
neutrophils were 70% and eosinophils were 3%.
2- X-ray PNS (Waters view or occipito-mental view): It showed that all
the sinuses were clear. Nasal septum deviation on the left with a spur
was clearly visible (fig. 19.2)
3- Serum IgE level: It was within normal limits.
4- CT scan of nose and PNS in both axial and coronal views. It was not
done in this case because of its cost.
5- Other baseline investigations for G/A fitness like X-ray chest (PA view),
blood sugar, ECG, urine D/R. All were within normal limits in this case.
Treatment:
After all investigations, patient was planned for septal surgery

Fig. 19.2
X-ray PNS (Waters view) showing
DNS on the left side with a spur

60

Clinical scenarios in oto-rhino-laryngology

(septoplasty) under general anaesthesia.


Discussion:
Symptomatic deviated nasal septum requires surgery. Two surgical
procedures are described for deviated nasal septum, sub-mucous resection
(SMR) and septoplasty. SMR was the classical operation for DNS in adults
at one time but now a days septoplasty is the preferred choice. Septoplasty
can be performed in children having deviated nasal septum producing
symptoms.

TEST YOURSELF

Read the clinical scenario given at the beginning and answers the following questions
1234-

How will you investigate this case?


What are the surgical options for treating this patient?
Compare differences between SMR and septoplasty.
Enlist complications of septal surgery.

61

Clinical scenarios in oto-rhino-laryngology

Case 20
Clinical Scenario
A 16 years old boy came with the complaint that he fell down from the stairs
about five days before and got trauma on his face and nose, which resulted in nasal
deformity. There was bleeding from the nose at that time which stopped spontaneously
after applying ice packs. Now he has marked nasal obstruction mainly on the right side.
Important points in history taking:
1- Exact nature of the trauma.
2- Injuries at other parts of the body. There was no injury to other body
parts.
3- History of unconsciousness at the time of fall. This patient remained
conscious after fall.
4- Amount of blood loss. Only small amount of blood was lost in this patient.
5- Shape of the nose before trauma. If any previous photograph is available,
then shape of the nose can be compared before and after the trauma.
6- Any previous history of nasal obstruction or other nasal symptom before
trauma. There was no problem related with nose before trauma.
Important points in clinical examination:
1- General physical examination regarding level of consciousness, vital
signs, anaemia and trauma to the other parts of the body.
2- Examination for shape of the nose from front, sides and above by standing
behind the patient. In this patient there was deformity of the external
nose lateral displacement of the nasal bones on right side and deviation
of tip of the nose on left side (fig. 20.1).
3- Anterior rhinoscopy and examination of the nasal vestibule. It showed
grossly deviated nasal septum on the right side almost touching the lateral
wall (fig. 20.2).

Fig. 20.1
Patient with trauma to the nose causing
external deformity with deviation of
the nasal bridge to right side

62

Clinical scenarios in oto-rhino-laryngology

Fig. 20.2
Nasal septum is grossly deviated
towards the right side

4- Nasal patency test: It was almost absent on the right side.


5- Any other active bleeding site, clot or mucosal tears in the nasal cavity.
6- Other injuries around the nose, eyes and face. There are few ecchymosis
below left eye and on the face (fig 20.1).
Investigations:
1- X-ray nasal bone (lateral view): This X-ray has a great medico-legal
importance especially in cases of assault. In this case multiple fractures
were present in nasal bones of both sides (fig. 20.3).
2- X-ray PNS (Waters view): This view is important to assess condition
of the nasal septum especially bony septum and the sinuses.
3- CT scan: It is helpful in cases where multiple bone fractures and soft
tissue injuries of the head and face are present. It was not done in this
case.
4- Baseline investigations for general anaesthesia like blood CP, prothrombin
time, activated partial thromboplastin time and urine D/R etc.
Diagnosis:
This was a case of class II nasal bone fracture with fracture and
deviation of the nasal septum.

Fig. 20.3
X-ray nasal bone (lateral view)
showing fracture of nasal bones

63

Clinical scenarios in oto-rhino-laryngology

Treatment:
The patient was planned for surgery of correction of nasal bone
fracture along with correction of the nasal septum (septo-rhinoplasty) under
general anaesthesia. Displaced nasal bone fracture of short duration (like
five days in this case) can be corrected by manipulation with the Walshams
forceps. When the patient comes late after healing of the displaced fracture,
osteotomies are required for correction of nasal deformity. In this case
correction of the displaced nasal bone was done with Walshams forceps and
for septal deviation septoplasty was done (fig 20.4).

Fig. 20.4
Post-operative photograph of the
patient after three weeks

Discussion:
Fracture of the nasal bone is very common and it is often associated
with fractures of other bones of the face and base of skull. It is classified into
following three types:
1- Class I fracture: In this type distal thin part of the nasal bone is fractured,
which may be depressed or displaced. This type of fracture occurs because
of trauma from the front and is associated with vertical fracture of the
nasal septum.
2- Class II fracture: In this type along with the fracture of nasal bone, fracture
of the frontal process of maxilla also occurs. This type of fracture is
caused by medium velocity trauma from the lateral side. It is associated
with horizontal or C shaped fracture of the nasal septum along with
perpendicular plate of ethmoid.
3- Class III fracture: In this type there is also involvement and fracture of
the ethmoid labyrinth. There is marked depression of the nasal bones
which are pushed under the frontal bones and there is an apparent widening
of space between the two eyes (telecanthus).

64

Clinical scenarios in oto-rhino-laryngology

TEST YOURSELF

Read the clinical scenario given at the beginning and answers the following questions

1234-

What are the important points in history taking and clinical examination in this case?
How will you investigate this patient?
How will you manage this patient?
What are the different types of nasal bone fracture?

65

Clinical scenarios in oto-rhino-laryngology

Case 21
Clinical Scenario
A 36 years old male patient presented with the complaints of nasal obstruction
which was mainly on the left side for last one year. It was often associated with left
sided facial pain, frontal headache and thick nasal discharge. For these complaints he
repeatedly visited his family physician who gave him medications and his symptoms
were relieved upto some extent for a short time period. Anterior rhinoscopy showed a
smooth, soft and pale mass in the left nasal cavity (fig. 21.1).
Important points in history taking:
1- Detailed history about the nasal obstruction including onset, duration,
progression, unilateral or bilateral, continuous or intermittent, aggravating
and relieving factors etc. In this case the onset was insidious. Nasal
obstruction was almost continuous, mostly on the left side and progressive
with no specific relieving or aggravating factors.
2- Any history of blood stained discharge or epistaxis. In this case there
was no such history.
3- Any history of nasal allergy or asthma like excessive sneezing, watery
rhinorrhoea, dyspnoea etc. In this case no such history.
4- Watering of the eyes. In this case there was occasional watering from the
left eye.
5- Any history of previous nasal surgery. In this case there was no history
of previous surgery.
Important points in clinical examination:
1- Anterior rhinoscopy. It revealed a mass in the left nasal cavity, completely
filling it (fig. 21.1). There was a slight deviated nasal septum on the right
side.

Fig. 21.1
Patient with mass in the left nasal cavity

66

Clinical scenarios in oto-rhino-laryngology

2- Probe test: It showed that the mass was soft, mobile, polypoidal, insensitive
to touch and did not bled on touch.
3- Nasal patency test: It was almost absent on the left side.
4- Posterior rhinoscopy. It was difficult to perform posterior rhinoscopy on
this patient but the mass was not visible.
Investigations:
1- X-ray PNS (Waters view). It showed haziness or opacification in the
left maxillary sinus with soft tissue in the left nasal cavity (fig. 21.2).
2- CT scan of nose and PNS in both axial and coronal views. It showed a
soft tissue mass arising from the left maxillary sinus and involving left
nasal cavity and nasopharynx (fig. 21.3 and 21.4). The other paranasal
sinuses were clear.
3- Baseline investigations for general anaesthesia like blood CP, prothrombin
time, activated partial thromboplastin time and urine D/R etc. All were
within normal limits.
Diagnosis:
This was a case of antro-choanal polyp involving left maxillary sinus,
nasal cavity and nasopharynx.

Fig. 21.2
X-ray PNS (Waters view) showing
haziness in left maxillary sinus and
nasal cavity

Fig. 21.3
CT scan (axial view) showing polyp
in left maxillary sinus, nasal cavity and
nasopharynx

67

Clinical scenarios in oto-rhino-laryngology

Fig. 21.4
CT scan (coronal view) showing polyp
in left maxillary sinus and nasal cavity

Differential Diagnosis:
1- Inverted papilloma: It is more common in old age patient and appears
more solid or firm than a polyp.
2- Hypertrophied inferior turbinate. Grossly enlarged inferior turbinate may
have the appearance similar to a polyp. Probe test can differentiate a
polyp with the enlarged inferior turbinate.
Treatment:
The patient was planned for surgery and offered the options of
conventional intranasal polypectomy or functional endoscopic sinus surgery.
Patient preferred the option of conventional intranasal polypectomy because
of cost. Under general anaesthesia complete removal of the polyp was done.
Antral part was also removed intranasally (fig 21.5). Post-operatively nasal
cavity was packed for 48 hours with ribbon gauze soaked in BIPP. Removed
polyp was sent for histopathology which confirmed the diagnosis.
Discussion:
Antro-choanal polyp arises from the maxillary antrum and prolapsed
through the ostium of the sinus in the middle meatus. Initially it hangs in the
nasal cavity and grows towards the choana thus it has three parts, antral,

Fig. 21.5
Antro-choanal polyp after removal

68

Clinical scenarios in oto-rhino-laryngology

choanal and nasal. The antral part is present in the maxillary antrum and
attached with the other two parts through a thin stalk. The choanal part of
the polyp may be seen in the oropharynx where it pushes the soft palate
downward and forward. The aetiology of antro-choanal polyp is exactly
unknown but it is supposed to be due to sinus infection. Antro-choanal polypi
are much less common than the ethmoidal polypi. They are more common
in males and can occur at any age but mostly before 40 years. It is mostly
unilateral but rarely may be bilateral. Histologically polyp is covered with
normal respiratory epithelium. The submucosa is grossly oedematous and
similar in appearance with the ethmoidal polyp except that there is no
eosinophilia.

TEST YOURSELF

Read the clinical scenario given at the beginning and answers the following questions
123456-

What is the most likely diagnosis in this case?


What are the differential diagnosis in this case?
What are the important points in history taking and clinical examination in this case?
How will you investigate this patient?
How will you manage this patients?
What are nasal polyp and what are its types?

69

Clinical scenarios in oto-rhino-laryngology

Case 22
Clinical Scenario
A 42 years old female patient came in OPD with complaints of swelling over
the nose with pain and bilateral nasal obstruction for one day (fig 22.1). She also had
history of trauma over her face and nose by a childs head. Local examination of the
nose showed bilateral bulging on the nasal septum, which was soft, fluctuant and tender
(fig 22.2).

Fig. 22.1
Patient showing swelling over the nose

Fig. 22.2
Bilateral bulging on the nasal septum

Important points in history taking:


1- Detailed history regarding the nature and severity of trauma.
2- Detailed history regarding the pain. In this case pain was mild to moderate
and more on touching the nose. It was continuous and localized with no
other specific aggravating or relieving factors.
3- Any history of bleeding or clotting disorders. In this patient there was
70

Clinical scenarios in oto-rhino-laryngology

no such history.
4- History of fever. There was no history of fever.
5- History of any nasal problem before the trauma. In this case there was
no such previous history.
Important points in clinical examination:
1- General physical examination with vital signs especially temperature.
There was no fever.
2- Examination and palpation of the external nose. Swelling was tender.
Nasal bone palpation was normal.
3- Examination of the nasal cavity by elevating tip of the nose. It showed
a bilateral, reddish, swelling or bulge on the nasal septum.
4- Palpation of the swelling with probe (probe test). The swelling was soft
and fluctuant.
5- Anterior rhinoscopy. It was not possible to perform anterior rhinoscopy
because of swelling and tenderness.
6- Examination of the eyes. There was no significant positive finding.
7- Examination of the throat especially for post-nasal bleeding. Throat was
clear.
8- Palpation of the draining lymph nodes. No palpable lymph node found.
Diagnosis:
The most probable diagnosis in this case was septal haematoma.
Differential Diagnosis:
1- Septal abscess: Sometimes clinically distinction between the septal
haematoma and abscess is not evident clearly. Septal abscess is more
painful and tender and fever is usually present. The colour of well formed
abscess may be pale as compared to haematoma which is usually red or
bluish. Tender and palpable draining lymph nodes suggest septal abscess.
2- Associated nasal bone fracture: Because of trauma to the nose fracture
of the nasal bone may be present along with septal haematoma.
3- Nasal boil with surrounding cellulitis.
Investigations:
1- Blood complete picture. It was within normal limits.
2- Investigations for bleeding or clotting disorders like bleeding time, clotting
time, prothrombin time and activated partial thromboplastin time. All
were within normal limits.
3- Plain X-ray nasal bone (lateral view) to exclude nasal bone fracture.
Treatment:
The patient was admitted in hospital and planned for incision and
drainage under local anaesthesia. 4% xylocaine pack was applied in the nasal
cavity for 15 to 20 minutes. After all aseptic measures incision was given on
one side of the swelling, clotted blood came out, all blood and necrotic tissues
71

Clinical scenarios in oto-rhino-laryngology

were evacuated. A corrugated drain was put and both nasal cavities were
packed with polyfax soaked ribbon gauze. Prophylactic antibiotic was given
post-operatively and the pack was removed after 48 hours.
Discussion:
Septal haematoma is collection of the blood under muco-perichondrium
or muco-periosteum of the nasal septum. In most of the cases it results from
trauma to the nasal septum but sometimes it may occur spontaneously. The
trauma to the nasal septum may be accidental e.g. road traffic accident, blow
or fall on the nose and surgical trauma e.g. after SMR and septoplasty
operation. The spontaneous haematoma formation may occur in bleeding
disorders like haemophilia, purpura, leukaemia etc.
Organization of the septal haematoma may occur if it is not drained,
leading to permanent thickening of the nasal septum. Infection usually follows
the septal haematoma leading to septal abscess formation. Larger haematoma
and septal abscess causes necrosis of the septal cartilage because nutrition
of the cartilage is affected due to the separation of muco-perichondrium.
Necrosis of the septal cartilage may result in supra tip depression or saddle
nose deformity.

TEST YOURSELF

Read the clinical scenario given at the beginning and answers the following questions
123456-

What is the likely diagnosis in this case?


What are the differential diagnosis in this case?
How will you manage this case?
What is septal haematoma?
What are the causes of septal haematoma?
What are the complications of septal haematoma if it is not treated appropriately?

72

Clinical scenarios in oto-rhino-laryngology

Case 23
Clinical Scenario
A 28 years old female patient came with the complaints of bilateral nasal
obstruction, excessive sneezing and watery rhinorrhoea for past 8 to 10 years. Now
nasal obstruction has increased markedly to became almost continuous and she can not
breath through her nose. On clinical examination of the nose, pale, multiple and bilateral
polypi were present in the nasal cavities (fig. 23.1).

Fig. 23.1
Patient with multiple polypi in both
nasal cavities

Important points in history taking:


1- Detailed history about the nasal obstruction including onset, duration,
progression, unilateral or bilateral, continuous or intermittent, aggravating
and relieving factors etc. In this case the onset was insidious. Nasal
obstruction was almost continuous, bilateral and progressive with no
specific relieving or aggravating factors.
2- Any history of nasal allergy or asthma like excessive sneezing, watery
rhinorrhoea, dyspnoea etc. All were present. Patient was allergic to
multiple things but allergy was severe with house dust.
3- Watering of the eyes. In this case there was watering from both the eyes.
4- Any history of previous nasal surgery. In this case there was no history
of previous surgery.
5- About vision and other eye problems. There was no other complaint
related with the eyes except watering of eyes and vision was normal.
6- Any history of blood stained discharge or epistaxis. In this case there was
no such history.
Important points in clinical examination:
1- External examination of the nose, face and eyes. There was no positive
findings on examination of these structures.
73

Clinical scenarios in oto-rhino-laryngology

1- Anterior rhinoscopy. It revealed multiple, pale, smooth and shiny grape


like polypi completely filling both nasal cavities (fig. 23.1).
2- Probe test: It showed that the polypi were soft, mobile, insensitive to
touch and did not bleed on touch.
3- Nasal patency test: It was almost absent on both the side.
4- Posterior rhinoscopy. Nasopharynx was clear on posterior rhinoscopy.
5- Examination of the eyes for vision, proptosis and intercanthal distance.
All were within normal limits.
Investigations:
1- CT scan of nose and PNS in both axial and coronal views. It showed
presence of polypi in both nasal cavities with involvement of both
maxillary sinuses, all ethmoidal air cells and sphenoid sinus (fig. 23.2
and 23.3).
2- Investigations for allergy including peripheral eosinophil count and total
serum IgE level, both were increased.
3- Baseline investigations for general anaesthesia like blood CP, prothrombin
time, activated partial thromboplastin time and urine D/R etc. All were
within normal limits.

Fig. 23.2
CT scan nose and paranasal sinuses
(axial view)

Fig. 23.3
CT scan nose and paranasal sinuses
(coronal view)

74

Clinical scenarios in oto-rhino-laryngology

Diagnosis:
This was a case of bilateral ethmoidal nasal polypi.
Differential Diagnosis:
1- Allergic fungal sinusitis. In this condition there is characteristic presence
of allergic mucin and double density shadows on CT scan.
Treatment:
The patient was planned for functional endoscopic sinus surgery
under general anaesthesia. Clearance of the polypi was done from all the
sinuses and nasal cavity using microdebrider. Removed polypi pieces were
sent in two separate bottles (one in formalin and other in normal saline) for
both histopathology and fungal smear and culture. Histopathology confirmed
the diagnosis of nasal polypi while fungal smear and culture did not showed
presence/growth of fungus.
Discussion:
Nasal polyp is a pedunculated swelling arising from the sinus and
nasal mucosa. Two distinctive varieties are described:
1- Ethmoidal polypi
2- Antro-choanal polyp
Ethmoidal polypi are more common and occurs in more than 70%
of cases. Although it is a disease of the ethmoidal sinuses, mucosal changes
extend further into the nose and other paranasal sinuses. The maxillary sinuses
are affected more than the frontal and sphenoid sinuses. The polyp may arise
from uncinate process, bulla ethmoidalis, ostium of the sinuses and medial
surface of the middle turbinate.
Ethmoidal polypi are mostly multiple, bilateral, pale and whitish in
colour. On naked eye examination they are soft, smooth and grape like
structures, which moves on probing and insensitive to touch. On microscopic
examination it is covered with ciliated columnar epithelium and submucosa
shows large intercellular spaces filled with serous fluid. There is marked
infiltration of eosinophils and round cells. The covering epithelium may
undergo metaplastic changes to transitional and squamous type, when exposed
to atmosphere.

TEST YOURSELF

Read the clinical scenario given at the beginning and answers the following questions
12345-

What is the most likely diagnosis in this case?


How will you investigate this patient?
How will you manage this patient?
What are nasal polyp and what are its types?
What are the features of ethmoidal polypi?
75

Clinical scenarios in oto-rhino-laryngology

Case 24
Clinical Scenario
A 31 years old male patient had undergone septal surgery 8 months back. After
surgery he lost for follow-up as he went to his village far away. He has reported again
with the complaint of left sided nasal obstruction for last few months. Nasal obstruction
was very marked and constant on left side only. On clinical examination, findings on
anterior rhinoscopy are shown in fig. 24.1.

Fig. 24.1
Anterior rhinoscopy showing nasal
adhesion between the septum and
inferior turbinate

Important points in history taking:


1- Detailed history about the nasal obstruction including onset, duration,
progression, unilateral or bilateral, continuous or intermittent, aggravating
and relieving factors etc. In this case the onset was insidious. Nasal
obstruction was almost continuous, mostly on the left side and progressive
with no specific relieving or aggravating factors.
2- Any history of nasal discharge or post-nasal dripping. In this case there
was no such history.
3- Any complaint related with the throat or ears. In this case there was no
complaint.
Important points in clinical examination:
1- External examination of the nose for shape and any pathology like saddle
nose deformity, columellar retraction or other external deformity.
2- Anterior rhinoscopy. It revealed adhesion formation between the nasal
septum and anterior end of inferior turbinate on the left side (fig. 24.1).
3- Nasal patency test. It was almost absent on the left side and normal on
the right side.
4- Posterior rhinoscopy. It was within normal limits.
5- Examination of the throat for any pathology like post-nasal dripping and
signs of inflammation.
76

Clinical scenarios in oto-rhino-laryngology

Diagnosis:
This was a case of nasal adhesion formation between the nasal
septum and anterior end of the inferior turbinate after septal surgery.
Investigations:
Baseline investigations for general anaesthesia like blood complete
picture, prothrombin time, activated partial thromboplastin time and urine
D/R etc. All were within normal limits.
Treatment:
The patient was planned for excision of the nasal adhesion. Patient
was not willing for procedure to be done under local anaesthesia, so surgery
was planned under general anaesthesia. Fibrous tissues were cut with
diathermy completely. Splint was applied to prevent further adhesion formation
and nasal cavity was packed for 24 hours. Splint was removed after 14 days.
Patient was symptom free after that and was followed up for some time.
Discussion:
The causes of nasal obstruction after septal surgery can be either due
to persistent septal deviation or due to complication like nasal adhesions,
septal perforation etc. Nasal adhesions are fibrous bands that are formed
between the nasal septum and lateral wall of the nose especially inferior
turbinate. Chances of adhesion formation are increased when surgery on the
lateral wall is also performed with septoplasty or SMR, like turbinectomy,
sub-mucous diathermy or electric cautery of the inferior turbinate. Nasal
adhesion is treated by excision of the fibrous band and keeping splint between
the nasal septum and inferior turbinate for some time, so that healing of the
overlying mucosa is complete.

TEST YOURSELF

Read the clinical scenario given at the beginning and answers the following questions

1234-

What is your diagnosis in this case?


How will you manage this patients?
How the nasal adhesions are formed after septal surgery?
How can adhesion formation be prevented after septal surgery?

77

Clinical scenarios in oto-rhino-laryngology

Case 25
Clinical Scenario
A 28 years old female patient came in OPD with the complaints of excessive
sneezing, watery rhinorrhoea and nasal obstruction off and on for last 10 to 12 years.
On clinical examination, findings on anterior rhinoscopy are shown in fig. 25.1.

Fig. 25.1
Anterior rhinoscopy showing pale and
enlarged inferior turbinate

Important points in history taking:


1- Onset of symptoms. It was insidious in onset and she did not know when
she was completely alright.
2- Frequency and severity of symptoms. According to her, frequency and
severity of all symptoms was mild initially but it increased gradually
with time. Then sneezing and rhinorrhoea occurred almost daily throughout
the year. There was no seasonal variation.
3- Aggravating and relieving factors. Sneezing and rhinorrhoea started
whenever she had exposure to dust, smoke and sometime perfumes.
Symptoms usually improved whenever she took anti-allergy drugs by
herself but relief was only temporary and started again after stopping the
medicine.
4- Detailed history about the nasal obstruction. Initially it was mild, intermittent
and usually on one side at one time but then it increased to became almost
continuous and one nose was alternatively blocked all the time.
5- Any symptom related with the eyes. She also had irritation and watering
from both eyes off and on.
6- Any history of skin or other allergy. There was no such history.
7- Any history of asthma or aspirin sensitivity. There was no such history.
Important points in clinical examination:
1- Anterior rhinoscopy. The findings are shown in fig. 25.1. Nasal mucosa
78

Clinical scenarios in oto-rhino-laryngology

2345-

was oedematous and pale with hypertrophy of inferior turbinates on both


sides. Thin watery secretions were also present in the nasal cavity.
Nasal patency test. It was markedly reduced on both the sides.
Posterior rhinoscopy. It was within normal limits.
Probe test to differentiate between enlarged inferior turbinate and a polyp.
Examination of the throat for any pathology. It was within normal limits.

Investigations:
1- Blood complete picture, showed raised eosinophil count of 12% in the
peripheral blood.
2- Total serum IgE level. It was markedly raised.
3- X-ray PNS (Waters view). It showed mucosal thickening in the maxillary
sinuses and enlarged inferior turbinates on both sides (fig. 25.2).
4- Skin tests with different allergens. Patient was found to be allergic with
multiple allergens.
5- Other baseline investigations for general anaesthesia. All were within
normal limits.

Fig. 25.2
Plain X-ray PNS (Waters view)
showing soft tissues of enlarged
inferior turbinates and mucosal
thickening in maxillary sinuses

Diagnosis:
This was a case of allergic rhinitis with enlarged inferior turbinates.
Treatment:
The patient was planned for reducing the size of inferior turbinate
by using microdebrider under general anaesthesia. Xylocaine with adrenaline
was injected in the inferior turbinate. A small submucosal pocket was formed
by pushing the microdebrider at the anterior end of inferior turbinate.
Submucosal tissues were removed by microdebrider keeping the overlying
mucosa intact. Post-operatively nasal cavity was packed for 24 hours. Good
nasal patency was achieved after surgery. Patient was advised to keep her
nasal allergy in control by taking medicines and regular follow-up. Postoperatively following medicines were prescribed:
1- Oral fexofenadine 120 mg once daily.
79

Clinical scenarios in oto-rhino-laryngology

2- Oral montelukast 10 mg once daily.


3- Topical beclomethasone spray twice daily.
4- Prophylactic oral antibiotic for 7 days.
Discussion:
Allergy is an abnormal reaction of the tissue to certain substances.
It is mediated by immunoglobulin E (IgE) and is classified as a type-I
hypersensitivity reaction.
Two clinical forms of allergic rhinitis are well known, seasonal and
non-seasonal or perennial. In seasonal form, symptoms appear only during
specific season in the specific areas of the world for e.g. in pollen season.
In non-seasonal or perennial form the symptoms are usually not very marked
as in the seasonal variety but remains throughout the year.
Typically in allergic rhinitis, nasal mucosa appears to be pale, swollen,
and sometimes bluish in colour. Nasal cavity may be seen full of watery and
thin secretions and inferior turbinates are swollen and hypertrophied.
Sometimes nasal polypi may be present.
Hypertrophied inferior turbinate size can be reduced surgically by
the use of:
1- Submucosal diathermy
2- Electric cautery
3- CO2 Laser
4- Submucosal resection by microdebrider
5- Coblation

TEST YOURSELF

Read the clinical scenario given at the beginning and answers the following questions

1234-

What is your diagnosis in this case?


How will you manage this patient?
What is the pathophysiology of nasal allergy?
What are different surgical options available for reducing the size of inferior turbinate?

80

Clinical scenarios in oto-rhino-laryngology

Case 26
Clinical Scenario
A father brought his 4 years old son with the complaint that he had inserted
something in his nose during playing about 4 hours back. After insertion he was
complaining of pain in his nose along with some blood stained discharge from his right
nostril. Child was very anxious and non-cooperative, it was not possible to perform
anterior rhinoscopy properly and foreign body was not visible on clinical examination.
Important points in history taking:
1- Inquire about nature of the foreign body, whether it was vegetative or
non-vegetative, metallic or non-metallic, smooth or sharp, rounded or
irregular etc. In this case nature of foreign body was not known.
2- Duration of foreign body insertion. It was inserted about 4 hours back.
3- Any attempt of removal by family member or family doctor. Unskilled
attempt for its removal may cause further pushing of foreign body deeper
and trauma to the surrounding structures. In this case there was no attempt
for removal.
4- Any bleeding from the nose. Only blood stained discharge was present.
5- Pain in the nose. Patient was complaining of pain in the nose.
Important points in clinical examination:
1- General physical examination. Child was very anxious and crying. His
vital signs were within normal limits.
2- External examination of the nose, showed slight blood stained discharge
coming out from his right nostril.
3- Anterior rhinoscopy. It was not possible to perform anterior rhinoscopy
and assess the foreign body, its nature and site of impaction.
Investigations:
1- Plain X-ray nasal cavity (lateral view). It showed a rounded, radiopaque
foreign body in the nasal cavity (fig. 26.1).
Diagnosis:
This was a case of foreign body in right nasal cavity.
Treatment:
Patient was admitted for removal of foreign body under general
anaesthesia, as he was crying and very anxious, even not allowing proper
examination. Under general anaesthesia foreign body was removed by passing
a probe beyond the foreign body and pulling it out (fig. 26.2). The foreign
body was a lithium battery (fig. 26.3).
81

Clinical scenarios in oto-rhino-laryngology

Fig. 26.1
X-ray nasal cavity (lateral view)
showing a rounded radiopaque foreign
body

Fig. 26.2
Method for removal of rounded
foreign body from the nose

Fig. 26.3
Removed foreign body from the nose,
a lithium battery

Discussion:
Foreign bodies in the nose are much more common in children and
mentally retarded persons. It may enter through one of the following routes:
1- Through the anterior nares
2- Through the posterior nares
3- Through penetration of its walls
82

Clinical scenarios in oto-rhino-laryngology

Most of the foreign bodies are impacted near the floor between the
nasal septum and inferior turbinate. Plain X-ray of the nose in lateral and
antero-posterior view may show the presence of radiopaque foreign body.
Soft tissue shadow may be seen in cases of radiolucent foreign body.
If the foreign body is not removed and remains unnoticed for few
days it will give rise to local reaction, oedema with superadded infection.
This local reaction occurs much earlier and more intense in cases of lithium
battery because of leakage of chemicals. It may cause ulceration in the nasal
mucosa and necrosis of the nasal septum. Calcium deposition will occur over
the foreign body leading to rhinolith formation. Rarely foreign body of the
nose may slip into the nasopharynx spontaneously and gets impacted in
aerodigestive tract lower down.

TEST YOURSELF

Read the clinical scenario given at the beginning and answers the following questions

1234-

How will you manage this case?


What are the different routes for entry of foreign body in the nose?
What is the commonest site of foreign body impaction in the nasal cavity?
What complications can arise if the foreign body nose is not removed?

83

Clinical scenarios in oto-rhino-laryngology

Case 27
Clinical Scenario
A 30 years old female patient came in the OPD with complaint of pain and
swelling at right nostril for one day. When she woke up in the morning a day before,
she had mild pain in the right nostril and she noticed some redness there but soon pain
increased and she had severe pain along with marked redness and swelling. Examination
findings of the nose at the time of presentation are shown in fig. 27.1.

Fig. 27.1
Patient with swelling and redness at
the right nostril

Important points in history taking:


1- Any previous history of similar swelling on the nose. There was no such
history.
2- History of fever. There was no history of fever.
3- History of diabetes mellitus or other immuno-compromised states. In
this case no such history.
Important points in clinical examination:
1- External examination of the nose. It showed a small, localized swelling
present near the columella on right nostril (fig. 27.1). Swelling was red
in colour along with oedema and redness of the surrounding skin. A small
opening was present over its surface and thick pus was coming out from
the opening. It was very tender.
2- Anterior rhinoscopy. Difficult to perform on the right side but it showed
redness and oedema of the surrounding nasal mucosa. Left nasal cavity
was normal.
Diagnosis:
The diagnosis of this case was Boil in the nose.
84

Clinical scenarios in oto-rhino-laryngology

Investigations:
No investigation was done in this case.
Treatment:
This was a case of a boil in the nose with spontaneous rupture. Pus
was thick and the opening was small, so the opening was made larger with
an artery forceps and all the pus drained out. Pus was sent for culture and
sensitivity. Antibiotic against staphylococcus aureus was given orally (amoxicillin
with clavulanic acid) along with analgesics. Pus C/S report showed heavy
growth of staphylococcus aureus. The organisms were found to be sensitive
to amoxicillin with clavulanic acid, so the same antibiotic was continued for
7 days. The patients recovery was uneventful.
Discussion:
Nasal boil or furuncle is the staphylococcal infection of a hair follicle.
In the nose hair follicles are present in the vestibule of the nasal cavity. Nasal
vestibule is present in the dangerous area of the face and its inflammation
may spread to cause intracranial complications. Boil in the nose is usually
single but multiple boils may occur. Like boil in the ear, nasal boil is also
very painful as skin of the nasal vestibule is tightly adherent to the underline
structures. Recurrence of nasal boil is common. The predisposing factors are
diabetes mellitus, general debilitating diseases, scratching and nose picking.

TEST YOURSELF

Read the clinical scenario given at the beginning and answers the following questions
1234-

What is the most likely diagnosis in this case?


How will you manage this case?
What are the predisposing factors of nasal boil?
Why nasal boil can cause intracranial extension of infection?

85

Clinical scenarios in oto-rhino-laryngology

Case 28
Clinical Scenario
A 47 years old male patient came in emergency department with complaint of
severe bleeding from the nose, which started suddenly half an hour back while he was
in his office. He tried to stop bleeding by pinching his nose and applying ice packs on
forehead and nose, but all measures failed.
Immediate management:
1- Examination of the nose showed profuse bleeding from the right nostril.
2- Anterior nasal packing was done immediately with a ribbon gauze soaked
with antibiotic ointment (polyfax) and xylocaine jelly in both the nasal
cavities.
3- Bleeding stopped after anterior nasal packing.
4- Throat was examined for any posterior bleeding. There was no posterior
bleeding and throat was clear after nasal packing.
5- Vital signs were checked. His pulse was 90/minute, blood pressure was
180/110 mm. of Hg., respiratory rate was 20/minute and temperature was
98.8oF.
6- Sublingual anti-hypertensive agent (captopril 25 mg.) was given immediately
to lower blood pressure.
7- Intravenous line was maintained with 5% dextrose water.
8- Prophylactic antibiotic was started parenterally.
9- Intravenous tranexamic acid was started.
10- Oral analgesics and anxiolytic agents were started.
11- Patient was admitted for further management.
Important points in history taking:
1- Any history of epistaxis before. According to the patient he had previous
history of recurrent epistaxis but it was always mild and stopped spontaneously
by pinching of the nose for 10 to 15 minutes.
2- Any history of bleeding from other site. There was no such history.
3- Any history of bleeding or clotting disorder. There was no such history.
4- Any history of fever. There was no history of fever.
5- Any history of trauma to nose. There was no such history.
6- History of hypertension. He was a known hypertensive patient for last
7 to 8 years and has been taking anti-hypertensive drugs irregularly. Blood
pressure often remained elevated and at such time bleeding also occurred
from the nose.
7- History of using of anticoagulant or platelet aggregation inhibitor drugs
like aspirin. There was no such history.

86

Clinical scenario in oto-rhino-laryngology

Important points in clinical examination:


1- General physical examination. The patient was a middle aged person of
obese built and average height. He was fully oriented in time, space and
person.
2- Assess the amount of blood loss and check for any signs of shock,
hypovolumia or anaemia.
Investigations:
1- Blood complete picture with platelet count. Haemoglobin was 11.2 gm/dl.
while all other were within normal limits.
2- Bleeding and clotting profile. All were within normal limits.
3- Tests for blood sugar, liver and renal functions. All were within normal
limits.
Diagnosis:
This was a case of severe epistaxis, most probably due to hypertension.
Further management:
1- Patient was shifted to the ward.
2- Vital signs monitoring including blood pressure. His blood pressure
became stable after few hours.
3- Physicians opinion for control of hypertension.
4- Anterior nasal packing was removed after 48 hours in the operation
theatre and nasal cavity was examined for any local pathology. There
was no bleeding after removal of the pack and no significant finding was
noted on examination of the nasal cavities.
5- Patient was kept for observation in the hospital for one day more and
discharged next day.
Discussion:
Epistaxis is a very common event which is often unpleasant for the
sufferer and can occasionally be fatal. Epistaxis is mentioned in the medical
literature dating back to very early times. Bleeding may occur from any site
but in 90% of cases epistaxis occurs from the Littles area. Littles area is
situated in the antero-inferior part of the nasal septum, where the anastomosis
of four blood vessels is present, called Kiesselbachs plexus. These arteries
include anterior ethmoidal, septal branch of superior labial, sphenopalatine
and greater palatine arteries. The other sites of bleeding include above the
middle turbinate from the ethmoidal vessels, below the middle turbinate from
branches of sphenopalatine artery and the nasopharynx. Venous bleeding
may occur especially from the retro-columellar vein which runs vertically
just behind the columella. Hypertension is a common cause of bleeding from
the nose in middle aged and old persons. The important step in the management
of epistaxis due to hypertension is the control of blood pressure. When the
blood pressure is properly controlled, epistaxis usually stops by 24 to 48
hours of nasal packing.
87

Clinical scenarios in oto-rhino-laryngology

TEST YOURSELF

Read the clinical scenario given at the beginning and answers the following questions

1234-

How will you manage this case?


What is the commonest site for epistaxis?
What is Kiesselbachs plexus?
What are the different causes of epistaxis?

88

Clinical scenarios in oto-rhino-laryngology

Case 29
Clinical Scenario
A 30 years old male patient came in the OPD with complaints of nasal discharge,
post-nasal dripping, facial pain and headache for last 8 to 10 months. He repeatedly
took medicines from his family physician for these complaints during this period but
there was no relief. On clinical examination, thick muco-purulent secretion was present
with hypertrophied inferior turbinates and congested mucosa in both nasal cavities.
Important points in history taking:
1- Detailed history regarding nasal discharge and post-nasal dripping.
According to the patient he was in his usual state of health about 10 months
back when he had an attack of common cold. He took medicines for this,
his fever was relieved but the nasal discharge and post-nasal dripping
continued. These symptoms were almost continuous but relieved to some
extent for a short period after taking medicines. Nasal discharge was thick
and yellowish to green in colour.
2- Detailed history regarding headache and facial pain. These symptoms
also started 10 months back along with the other complaints. Headache
was mainly in the frontal region or sometimes between the eyes. These
symptoms were intermittent and relieved by taking medicines but reappeared
again after stopping the treatment.
3- Any history of watery rhinorrhoea, excessive sneezing, irritation in the
nose or eyes or any other symptom of nasal allergy. There was no previous
history of nasal allergy.
4- Any history of nasal obstruction. According to him, he had often nasal
obstruction of one nostril or the other, especially on lying in lateral position
lower dependent nose was blocked.
5- Any history of repeated sore throat. Because of post-nasal dripping he
often had irritation in the throat and he had to clear his throat repeatedly.
6- Drug history. He did not know all the names of medicines which he had
used during this period, but he had repeatedly taken different antibiotics,
analgesics, decongestants, nasal sprays, steam inhalation etc.
7- Any history of asthma or skin allergies. There was no such history.
Important points in clinical examination:
1- General physical examination. There was nothing significant on general
physical examination.
2- External examination of the nose, face and eyes. All were normal.
3- Anterior rhinoscopy. It showed a slightly deviated S shaped nasal septum.
Muco-purulent secretion was present in both nasal cavities with congested
mucosa and hypertrophied inferior turbinates.
89

Clinical scenarios in oto-rhino-laryngology

4- Posterior rhinoscopy. It showed presence of post-nasal dripping.


5- Nasal endoscopy. It was done in OPD under local anaesthesia. All the
findings of anterior rhinoscopy were confirmed. There was oedema,
mucosal thickening and polypoidal mucosa in the area of osteo-meatal
complex in the middle meatus.
Investigations:
1- CT scan of the nose and PNS (both axial and coronal views). It showed
blockage in the osteo-meatal complex on both the sides, mucosal
thickening in the ethmoidal air cells and both maxillary sinuses. There
was retained secretions in the left maxillary sinus (fig. 29.1 and 29.2).
Frontal and sphenoid sinuses were within normal limits.
Diagnosis:
This was a case of chronic rhino-sinusitis.
Treatment:
As patient had already taken all the medicines with no relief, so
patient was admitted for functional endoscopic sinus surgery under general

Fig. 29.1
CT scan of nose & PNS in coronal
view

Fig. 29.2
CT scan of nose & PNS in axial view

90

Clinical scenarios in oto-rhino-laryngology

anaesthesia. All the polypoidal mucosa present in the middle meatus was
cleared. Uncinate process was removed and middle meatus antrostomy done.
All the ethmoidal air cells were cleared (ethmoidectomy done). Nasal cavity
was packed for 24 hours. Post-operatively patient was put on medical
treatment and patient showed good improvement in all his symptoms.
Discussion:
Chronic sinusitis is the chronic inflammation of the paranasal sinuses
which lasts for months and sometime years. Acute exacerbations are common
and in between the intervals, symptoms may be reduced. Most of the cases
of chronic sinusitis are due to failure of the acute infection to resolve. It may
follow single or repeated attacks of acute sinusitis. Oedema of the sinus
mucosa is present ranging from slight thickening to gross polyposis. Chronic
inflammatory cellular infiltration is present with glandular hypertrophy. The
surface epithelium may show desquamation, regeneration, ulceration or
metaplasia. The organisms are usually mixed and include streptococci,
pneumococci, B. proteus and anaerobes.

TEST YOURSELF

Read the clinical scenario given at the beginning and answers the following questions

12345-

What is the most likely diagnosis in this case?


What are the important points in history taking and clinical examination in this patient?
How will you investigate this patient?
How will you manage this patient?
Describe the pathology of chronic rhino-sinusitis.

91

Clinical scenarios in oto-rhino-laryngology

Case 30
Clinical Scenario
A 14 years old boy came with the complaints of left sided nasal obstruction,
recurrent epistaxis and hyposmia for last 6 to 7 months. He had also noticed a mass
coming out from his left nostril for last one month with more frequent nasal bleeding
(fig. 30.1).

Fig. 30.1
Patient with mass in the left nasal cavity

Important points in history taking:


1- Detailed history regarding his symptoms. According to the patient he was
in his usual state of health 6 to 7 months before, when he developed nasal
obstruction mainly on the left side. Initially it was intermittent but soon
it became continuous. Along with nasal obstruction he had decreased
sense of smell and recurrent bleeding from left side of the nose, amount
ranging from few drops to profuse bleeding and stopped spontaneously
most of the time. Last month he also noticed a mass coming out from his
left nostril which was increasing progressively.
2- Any history of nasal packing or admission in the hospital due to epistaxis.
He was never admitted in a hospital for nasal packing before.
3- Any history of blood transfusion. There was no such history.
4- Any history of rhinorrhoea, sneezing or nasal allergy. There was no such
history.
5- Any complaint related with the eyes. There was no complaint.
4- Any complaint related with the ear. According to the patient he had
noticed some blockage and occasional discomfort in his left ear.
Important points in clinical examination:
1- General physical examination. Patient was a young adolescent boy of
average height and built, fully conscious and oriented. His vital signs
92

Clinical scenarios in oto-rhino-laryngology

2-

2345-

were within normal limits. He was slightly anaemic with no other positive
finding on general physical examination.
Examination of the nose. It revealed a mass coming out from the left
nasal cavity, completely filling it and bleeding on touch (fig. 30.1). The
outer surface of the mass was irregular, ulcerated and red. There was
deviation of the nasal septum towards the right side due to the mass.
Probe test. It showed that the mass was soft to firm in consistency, mobile,
polypoidal, insensitive to touch and was bleeding on touch.
Nasal patency test. It was absent on the left side and decreased on the
right side.
Posterior rhinoscopy. It was difficult to perform posterior rhinoscopy on
this patient but a similar mass was visible in the nasopharynx.
Examination of the eyes. There was no apparent proptosis, displacement,
or telecanthus. Eye movements and vision were also normal.

Differential Diagnosis:
Following conditions must be considered in the differential diagnosis:
1- Nasopharyngeal angiofibroma.
2- Inverted papilloma. It is more common in old age patient.
3- Antrochoanal polyp. It is smooth, soft, pale and glistening.
4- Fungal rhino-sinusitis.
Investigations:
1- CT scan of nose and PNS in both axial and coronal views with contarst.
It showed a soft tissue mass filling the nasopharynx, left nasal cavity,
left maxillary sinus and anterior ethmoid air cell. It was pushing the nasal
septum on the right with remoulding of the surrounding bones. It appeared
to be vascular on contrast media (fig. 30.2 and 30.3).
2- Angiography with embolization. Four vessels angiography was performed
which showed that the mass was very vascular and recieving its blood
supply from the branches of left external carotid artery. Embolization of
the feeding vessels were done with gelfoam (fig. 30.4).
3- Baseline investigations for general anaesthesia like blood CP, prothrombin

Fig. 30.2
CT scan of nose & PNS in coronal
view

93

Clinical scenarios in oto-rhino-laryngology

Fig. 30.3
CT scan of the nose & PNS in axial
view

Fig. 30.4
Angiography of the left external carotid
artery before and after embolization

time (PT), activated partial thromboplastin time (APTT) and urine D/R
etc. All were within normal limits except his haemoglobin was 9.2 gm/dl.
He was transfused with two packed cells unit preoperatively.
Diagnosis:
This was a case of nasopharyngeal angiofibroma involving
nasopharynx, left nasal cavity, left maxillary sinus and left anterior ethmoidal
air cells.
Treatment:
The patient was planned for surgical removal of the angiofibroma.
Angiography with embolization was already done one day before surgery.
A combined lateral rhinotomy and sub-labial transantral approach was planned
(fig. 30.5). Complete excision of the tumour was done with this approach.
Tumour removed was sent for histopathology which confirmed the diagnosis
of nasopharyngeal angiofibroma. Post-operative recovery was uneventful
and during follow-up patient was alright with no recurrence.
Discussion:
Nasopharyngeal angiofibroma is a benign but locally aggressive
94

Clinical scenarios in oto-rhino-laryngology

Fig. 21.5
Tumour visible through sub-labial
transantral approach

tumour. It is the commonest benign tumour of the nasopharynx. It occurs


mostly in adolescent males so it is also called juvenile angiofibroma. The
usual age of onset is in the second decade. It occurs almost exclusively in
males. The exact aetiology of the tumour is unknown but it is thought to be
hormonal dependent (male sex hormones). Histologically angiofibroma is
composed of fibrous connective tissues interspersed with variable proportion
of endothelium lined blood spaces. The ratio of fibrous and vascular component
may vary. Mostly the vessels are just endothelium lined spaces with no
muscle coat. That is why severe bleeding may occur on taking biopsy and
surgical removal, as these vessels cannot contract to stop bleeding.
The tissue of origin remains unknown, although various theories have
been proposed. The site of origin of the tumour is also a matter of dispute.
Previously it was thought that it arises from the roof or posterior wall of the
nasopharynx. Now it is believed that it arises from the posterior part of lateral
wall of the nose close to sphenopalatine foramen. From here the tumour
extends into the nasal cavity, nasopharynx, paranasal sinuses and
pterygopalatine fossa.
Nasopharyngeal angiofibroma is a benign tumour but locally it is
invasive and destroys the adjoining structures. From the site of its origin near
sphenopalatine foramen the tumour grows in different directions. Anteriorly
it extends into the nasal cavity and may invade the paranasal sinuses including
maxillary, ethmoid and sphenoid sinuses. Laterally it may extend into the
pterygopalatine fossa and from there to infra-temporal fossa and cheek. It
may enter into the orbit through inferior and superior orbital fissures. It may
extend above into the middle and anterior cranial fossa. In middle cranial
fossa it enters through foramen lacerum or through sphenoid sinus. In anterior
cranial fossa it enters through ethmoidal sinuses or through cribriform plate.

95

Clinical scenarios in oto-rhino-laryngology

TEST YOURSELF

Read the clinical scenario given at the beginning and answers the following questions
1234567-

What is the most likely diagnosis in this case?


What are the differential diagnosis in this case?
What are the important points in history taking and clinical examination in this case?
How will you investigate this patient?
How will you manage this patients?
What is the site of origin of nasopharyngeal angiofibroma?
Why nasopharyngeal angiofibroma is common in adolescent males?

96

Clinical scenarios in oto-rhino-laryngology

Section III

ORAL CAVITY & PHARYNX


Case 31Case 32Case 33Case 34Case 35Case 36Case 37Case 38Case 39-

Chronic tonsillitis
Post-tonsillectomy haemorrhage
Quinsy
Enlarged adenoids
Papilloma of the cheek
Carcinoma of the cheek
Carcinoma of the tongue
Foreign body of the throat
Ranula

97

99
102
104
107
110
113
117
120
123

Clinical scenarios in oto-rhino-laryngology

Case 31
Clinical Scenario
A 23 years old female patient came in OPD with complaints of recurrent attacks
of sore throat, odynophagia and fever for the last 10 to 12 years. She used to take
medicines from her family physician and symptoms usually relieved within few days.
Initially these attacks were very infrequent but with the passage of time, frequency of
attacks had increased and it occured every one to two months.
Important points in history taking:
1- Detailed history regarding onset, duration, frequency, aggravating and
relieving factors of an attack of sore throat.
2- Fever, whether high grade or low grade. In this case fever was mostly in
the range of 101 to 102o F.
3- Any associated nasal complaints like nasal obstruction, post-nasal dripping,
rhinorrhoea etc. In this case there was no nasal complaint.
4- About gastro-oesophageal reflux disease (GERD). In adults it is one of
the common cause of recurrent or chronic sore throat. In this patient there
was no history of GERD.
5- About any symptom related with rheumatic fever, glomerulonephritis or
sub-acute bacterial endocarditis.
6- History of sleep apnoea or snoring. In this patient there was history of
snoring and occasional sleep apnoea.
7- Any history of bleeding or clotting disorder. This is important in patient
who is undergoing tonsillectomy.
Important points in clinical examination:
1- Assessment of the palatine tonsils and pillars. In this case tonsils were
congested, hypertrophied with prominent crypts and congested anterior
pillars (fig 31.1).

Fig. 31.1
Patient with congested and
hypertrophied tonsils and congested
anterior pillars

99

Clinical scenarios in oto-rhino-laryngology

2- Palpation of jugulo-digastric lymph nodes. These were palpable and


tender on both the sides in this patient.
3- Anterior and posterior rhinoscopy to rule out any nasal disease. In this
patient it was within normal limits.
Diagnosis:
This was a case of chronic or recurrent tonsillitis with hypertrophied
tonsils.
Investigations:
1- Blood complete picture with platelet count. It was within normal limits.
2- Bleeding and clotting profile including PT and APTT.
3- Anti-streptolysin O titre (ASO titre). In this case it was increased (300).
4- X-ray chest (PA view) for general anaesthesia.
5- Urine D/R.
6- Hepatitis B and C screening.
Treatment:
The patient was planned for tonsillectomy under general anaesthesia
and admitted in the hospital in morning with overnight nil per orally (NPO).
Tonsillectomy was done by diathermy method (fig 31.2) and patient was
discharged next day. Post-operatively she was given antibiotic and analgesics
for 7 days. Post-operative recovery was uneventful (fig 31.3).
Discussion:
In chronic tonsillitis there are small micro-abscesses within the core
of the tonsils and micro-organisms are present in these abscesses. These
abscesses are surrounded by fibrous tissues and antibiotics do not reach
within the abscesses in proper concentration. Thus eradication of microorganisms within the core of the tonsil is difficult. One way of eradication
is to remove the tonsil itself. That is why tonsillectomy is performed in cases
of chronic or recurrent tonsillitis. Three clinical forms of chronic tonsillitis
are recognized:

Fig. 31.2
Tonsillectomy by diathermy method
in Roses position

100

Clinical scenarios in oto-rhino-laryngology

Fig. 31.3
Post-operative photograph of the
patient after 5 days of tonsillectomy
showing slough in the tonsillar fossa

1- Chronic follicular tonsillitis


2- Chronic parenchymatous tonsillitis
3- Chronic fibroid tonsillitis
In cases of chronic or recurrent tonsillitis, tonsillectomy is indicated
when there are:
1- Seven attacks of tonsillitis per year for one year.
2- Four to six attacks of tonsillitis per year for two consecutive years.
3- Three attacks of tonsillitis per year for three consecutive years.
There are different methods for performing tonsillectomy including:
1- Dissection or cold steel method
2- Diathermy
3- Diode or CO2 laser
4- Harmonic scalpel
5- Coblation
6- Radiofrequency ablation
7- Microdebrider

TEST YOURSELF

Read the clinical scenario given at the beginning and answers the following questions
12345-

What is the most likely diagnosis in this case?


How will you manage this case?
What investigations are required in a patient undergoing tonsillectomy?
What are the indications for tonsillectomy other than chronic or recurrent tonsillitis?
What are the different methods for tonsillectomy?

101

Clinical scenarios in oto-rhino-laryngology

Case 32
Clinical Scenario
A father brought his 9 years old son in emergency whose tonsillectomy had
been performed 5 days back with the complaint that he had vomited out a large amount
of blood 1 hour back. After that fresh blood was coming out from the throat.
Important points in history taking:
1- Detailed history regarding the amount and nature of vomiting and bleeding.
The vomited material contained dark coloured blood and was about half
a litre in amount. After that he was continuously spitting fresh blood mixed
with saliva at regular interval.
2- Any history of fever. He had a spike of high grade fever last night which
was relieved by taking syrup paracetamol.
3- Was he taking antibiotic regularly? According to his father, he was not
taking the medicines regularly and he had missed many doses. In addition
he was also not using antiseptic mouth wash regularly.
Important points in clinical examination:
1- General physical examination. The child was looking lethargic and weak
but he was fully oriented in time, space and person. His pulse was
100/minute, blood pressure was 110/70 mm of Hg., respiratory rate was
20/minute and temperature was 100oF. Signs of dehydration were positive
showing mild to moderate dehydration.
2- Assess the amount of blood loss and check for any signs of shock,
hypovolumia or anaemia.
3- Examination of the oral cavity and throat. It showed a clot present in his
right tonsillar fossa. There was no active bleeding. Oral hygiene was very
poor.
Immediate management:
1- Patient was admitted in hospital and was advised not to take any thing
by mouth (NPO) till further orders.
2- Intravenous line was maintained with dextrose saline infusion.
3- Parenteral antibiotic (intravenous ceftrixone) was started according to the
body weight.
4- Blood was sent for blood complete picture, prothrombin time (PT) and
activated partial thromboplastin time (APTT).
Diagnosis:
This was a case of secondary post-tonsillectomy haemorrhage.
102

Clinical scenario in oto-rhino-laryngology

Further management:
1- Patient was admitted in the ward for further management and monitoring.
2- Vital signs monitoring. Patient was haemodynamically stable.
3- Maintain good oral hygiene with regular antiseptic gargles and mouth
wash.
4- Blood reports showed that his haemoglobin was 10.2gm/dl while PT and
APTT were within normal limits.
5- Regular examination of the throat. There was no further bleeding and
clot size started to regress and disappeared completely next day.
6- Oral syrup paracetamol started and fever settled down with no further
spike.
7- Soft diet allowed from next day.
8- Patient was kept in hospital for 2 days more for observation and parenteral
antibiotic and then discharged on oral antibiotic.
Discussion:
Haemorrhage is the most important complication after tonsillectomy
which can be fatal sometimes. It is conventionally divided into, primary,
reactionary and secondary haemorrhage. Primary haemorrhage occurs on
the operation table after removal of the tonsils. Reactionary haemorrhage
occurs within 24 hour after the operation. Secondary haemorrhage occurs
after 24 hours till 14 days, when the healing in tonsillar fossa is completed.
Usually it occurs on 5th post-operative day or afterwards. The cause of
secondary haemorrhage is infection of the tonsillar bed with sloughing and
opening of small blood vessels in its bed. The haemorrhage is usually not
profuse and is associated with fever. It is treated by giving proper antibiotics,
rest, sedation and observation in the hospital. In severe and profuse secondary
haemorrhage not responding to medical treatment, fossa is packed and the
pillars are stitched together under general anaesthesia and the pack is removed
after few days, when the condition is settled.

TEST YOURSELF

Read the clinical scenario given at the beginning and answers the following questions

12345-

How will you manage this case?


What are the different types of post-tonsillectomy haemorrhage?
What is the cause of secondary post-tonsillectomy haemorrhage?
What is the usual time for secondary post-tonsillectomy haemorrhage?
What is the treatment of secondary post-tonsillectomy haemorrhage?

103

Clinical scenarios in oto-rhino-laryngology

Case 33
Clinical Scenario
A 17 years old male patient came in the OPD with complaints of severe pain in
the throat on left side, difficulty in swallowing, high grade fever and restricted mouth
opening for last 2 days. Examination findings of his throat and oral cavity are shown
in fig. 33.1.

Fig. 33.1
Patient with marked bulging and
redness over the soft palate with
restricted mouth opening

Important points in history taking:


1- Onset and progression of the symptoms. The onset was sudden two days
back. Initially all the symptoms were mild in severity but increased rapidly.
2- Detailed history regarding pain especially unilateral or bilateral. Initially
pain was bilateral and mild but soon it became more on the left side. Then
it was severe, mainly on the left side, continuous, localized and throbbing
in nature. It was aggravated by swallowing and relieved to some extent
by taking analgesics.
3- Detailed history about the fever. It was sudden in onset, almost continuous,
high grade, not associated with chills and relieved to some extent by
taking antipyretic drugs.
4- Detailed history about the odynophagia. There was severe pain on
swallowing especially on left side of the throat. Pain was so severe that
he was unable to swallow his saliva as well.
5- Detailed history of restricted mouth opening or trismus. There was severe
pain on the left side on mouth opening along with trismus.
6- Any history of voice change. Voice has changed to became thick and
muffled.
7- Any history of respiratory obstruction or stridor. There was no such
history.
8- Any history of earache. There was mild earache on the left side but his
104

Clinical scenarios in oto-rhino-laryngology

hearing was unaffected.


9- Previous history of repeated sore throat and fever. He had recurrent attacks
of sore throat with fever since childhood. Usually it occurred after every
two to three months.
Important points in clinical examination:
1- General physical examination. Patient was a teenager of average built
and height, looking very ill and restless. His pulse was 110/minute, blood
pressure was 110/75 mm of Hg., respiratory rate was 20/minute and
temperature was 102oF. His voice was thick and muffled. Rest of the
general physical examination was unremarkable.
2- Examination of the oral cavity and throat. There was restricted mouth
opening with bad smell coming out and tongue was coated. There was
a bulge involving left side of the soft palate and tonsil which was red and
congested (fig. 33.1). The uvula was swollen and pushed towards the
right side.
3- Examination of the lymph nodes showed palpable and tender jugulodigastric lymph node on the left side.
4- Examination of the nose and ears. It was within normal limits.
Diagnosis:
The diagnosis in this case was peri-tonsillar abscess or quinsy.
Differential Diagnosis:
This condition has to be differentiated from:
1- Parapharyngeal abscess.
2- Neoplasia of the tonsils.
3- Retropharyngeal abscess.
Investigations:
No pre-operative investigation was required in this case.
Treatment:
The patient was admitted for incision and drainage. He was very cooperative so incision and drainage was decided to be done under local
anaesthesia. 4% xylocaine was used as gargles for local anaesthesia. In sitting
position incision was given on the bulging soft palate, at the junction of two
imaginary lines as shown in fig 33.2. Thick pus came out which was sent for
culture and sensitivity. Parenteral broad spectrum antibiotic was started along
with oral anti-inflammatory and analgesic drugs. Pus C/S report showed heavy
growth of mixed gram +ve organisms, sensitive to the given antibiotic. The
patients recovery was uneventful.
Discussion:
Quinsy or peritonsillar abscess is the collection of pus in the peritonsillar
space between the capsule of the tonsil and adjacent lateral pharyngeal wall.
105

Clinical scenarios in oto-rhino-laryngology

Fig. 33.2
Two imaginary line, horizontal along
the base of uvula and vertical along
the anterior pillar

It usually follows an acute attack of tonsillitis. The route of infection is


probably via a crypt, mostly crypta magna. Before development of the
abscess, inflammation is set up in the peritonsillar region outside the tonsillar
capsule and is called stage of peritonsillitis. As the inflammation increases,
frank pus is collected between the tonsillar capsule and superior constrictor
muscle. As the route of infection in majority of the cases is crypta magna,
the peritonsillar abscess lies mostly near the upper pole of the tonsils. In
majority of the cases, abscess is unilateral and most frequently affects young,
adult male but may occur at any age.
If the peri-tonsillar abscess or quinsy is not treated properly,
spontaneous rupture of the abscess can occur leading to aspiration pneumonia
and lung abscess. If incision and drainage of the abscess is planned under
general anaesthesia, great care should be taken during endotracheal intubation
as spontaneous rupture can occur during this procedure.

TEST YOURSELF

Read the clinical scenario given at the beginning and answers the following questions
1234567-

What is the most likely diagnosis in this case?


What are the important points in history taking in this patient?
What are the important points in clinical examination of this patient?
What are the differential diagnosis in this case?
How will you manage this case?
What is the pathology of peri-tonsillar abscess or quinsy?
What complications can occur if quinsy is not treated early and properly?

106

Clinical scenarios in oto-rhino-laryngology

Case 34
Clinical Scenario
A mother brought her 8 years old son with the complaints of mouth breathing,
snoring, bed wetting and recurrent earache for the last two to three years. All these
complaints were increased in severity during the last 6 months. He also had history of
recurrent fever off and on.

Fig. 34.1
Patient with open mouth and prominent
incisors

Important points in history taking:


1- Detailed history about the symptoms, whether these are continuous or
intermittent, its severity, aggravating and relieving factors etc. According
to the patients mother, these symptoms were developed about three years
back and were continuous and progressively increasing. There was no
specific aggravating or relieving factor.
2- Ask about the voice change. His voice had a nasal tone in voice.
3- Other ear symptoms like deafness and discharge. He often complained
of earache but he had no complaint regarding hearing.
4- Ask about the pain in the throat, odynophagia etc. He had recurrent
attacks of sore throat often associated with high grade fever.
5- History about any allergy. There was no history of asthma or any other
allergy.
Important points in clinical examination:
1- General physical examination. It showed open mouth, prominent incisors
and pinched nostrils (fig 34.1). Rest of the general physical examination
was unremarkable.
2- Examination of the nose for any pathology like deviated nasal septum,
nasal polyp, enlarged turbinates. In this patient nose was congested with
presence of muco-purulent secretions in the nose.
107

Clinical scenarios in oto-rhino-laryngology

3- Check for nasal patency. Nasal patency was almost absent on both the
sides.
4- Posterior rhinoscopy. It is usually very difficult to perform in a child.
Similarly this child also did not co-operate for posterior rhinoscopy.
5- Examination of the oral cavity and oropharynx especially palatine tonsils,
faucial pillars. Tonsils were congested with slight enlargement.
6- Examination of the ear especially condition of the tympanic membrane
and tuning fork tests and Valsalvas maneuver. No significant finding
was present at the time of examination.
7- Nasal and nasopharyngeal examination with rigid endoscope, if available.
8- Palpation of cervical lymph nodes especially jugulo-digastric lymph
nodes, which were not palpable.
Diagnosis:
The most probable diagnosis was enlarged adenoids in this case.
Differential Diagnosis:
All other cause of nasal obstruction in children should be included
in differential diagnosis like deviated nasal septum, nasal polyp, hypertrophied
inferior turbinate, foreign body or rhinolith, choanal atresia, etc.
Investigations:
1- X-ray soft tissue of the nasopharynx (lateral view). This is the most
important investigation for the diagnosis of enlarged adenoids.
Nasopharyngeal airway was assessed in this view, which showed grossly
enlarged adenoids with only slit like nasopharyngeal airway (Fig. 34.2).
2- Other baseline investigations for general anaesthesia when planned for
surgery like blood complete picture, ESR, random blood sugar, urine
D/R and X-ray chest (PA view).
Treatment:
Patient was planned for adenoidectomy under general anaesthesia.
Oral endotracheal intubation was done. In Roses position Boyles Davis

Fig. 34.2
X-ray soft tissue nasopharynx (lateral
view) showing enlarged adenoids with
airway narrowing

108

Clinical scenarios in oto-rhino-laryngology

mouth gag was applied and digital palpation of the nasopharynx was done
and adenoids enlargement was confirmed (fig 34.3). Adenoids removed by
adenoid curette.

Fig. 34.3
Patient in Roses position with Boyles
Davis mouth gag applied

Discussion:
The adenoids are enlarged and hypertrophied nasopharyngeal tonsils,
sufficient to produce symptoms. Nasopharyngeal tonsil is present in the
nasopharynx at the junction of its roof and posterior wall. This is composed
of vertical ridges of lymphoid tissues, separated by deep cleft and covered
by ciliated columnar epithelium. It is present at birth, show physiological
enlargement and tends to start atrophy at puberty. Hypertrophy sufficient to
produce symptoms occur most commonly between the ages of three to seven
years. Inflammatory changes occur in adenoids as a result of infection in it
alone or in association with rhinitis and tonsillitis. Allergy of the upper
respiratory tract may also contribute to the enlargement of adenoids. The
symptoms of adenoids enlargement are produced due to respiratory obstruction
and blockage of the eustachian tube. Signs and symptoms depend upon the
relative size of the adenoids with that of nasopharynx.

TEST YOURSELF

Read the clinical scenario given at the beginning and answers the following questions
12345-

What is the most likely diagnosis in this case?


What are the differential diagnosis in this case?
What investigations will you order in this case?
How will you treat this case?
Briefly outline the steps of adenoidectomy operation.

109

Clinical scenarios in oto-rhino-laryngology

Case 35
Clinical Scenario
A 58 years old male patient presented with a growth on the inner surface of his
right cheek for last 5 to 6 months (fig 35.1). Initially it was very small but it increased
in size rapidly and within this period it became of the present size.

Fig. 35.1
Patient with a growth on the inner
surface of right cheek

Important points in history taking:


1- Detailed history regarding the growth. According to the patient, it was
present for last 5 to 6 months and he did not know how it started. Initially
it was very small and he ignored it. Soon it started to increase in size till
its present form (fig. 35.1). There was no aggravating or relieving factor
known to him and there was also no associated factor.
2- Any history of pain in the swelling or burning sensation in the mouth.
There was no history of pain in the growth or surrounding area and no
history of burning sensation in the mouth.
3- Any history of trauma to the cheek especially due to sharp tooth etc.
There was no such history.
4- Any history of using beetle nut, pan, tobacco, smoking or alcohol. He
used to eat pan with some tobacco but in very less quantity, 2 to 3 pans
per day for the last 10 to 12 years. There was no history of consumption
of alcohol or smoking.
5- Any history of restricted mouth opening. According to him there was
some degree of restricted mouth opening especially after the start of
growth.
6- Any swelling in the neck or below mandible. There was no such history.
7- Any history of fever, anorexia or weight loss. There was no such history.
Important points in clinical examination:
1- General physical examination. Patient was a middle aged person of
110

Clinical scenarios in oto-rhino-laryngology

2-

345-

average height and slightly obese built, fully oriented in time, space and
person. His pulse was 80/minute, blood pressure was 130/85 mm of Hg.,
respiratory rate was 22/minute and temperature was 98.6oF.
Examination of the growth. There was a growth present on the inner
surface of the cheek, irregular in shape, 3x4 cms. in size on the right
side. The surface was formed by multiple small finger like projections,
partly white and partly pink in colour. The margins were irregular but
well defined and adjacent mucosa was slightly congested and red. On
palpation it was non-tender, firm in consistency with no induration of
the surrounding area or the base and there was no bleeding on touch.
Examination of the cervical lymph nodes. There was no palpable lymph
node in the neck on any side.
Examination of rest of the oral cavity and oropharynx. Oral hygiene was
satisfactory, teeth were stained with pan and mouth opening was slightly
restricted.
Examination of the hypopharynx and larynx. It was within normal limits.

Differential Diagnosis:
1- Benign tumours like squamous papilloma, fibroma, haemangioma or
pyogenic granuloma etc.
2- Malignant tumours like verrucous carcinoma, squamous cell carcinoma,
adnocarcinoma or adenoid cystic carcinoma etc.
Investigations:
1- Punch biopsy. It was done under surface anaesthesia, tissues taken from
multiple areas of the swelling and sent for histopathological examination.
The histopathology report showed fibrous stroma covered with stratified
squamous cells with some infiltration by lymphocytes. No features of
malignancy were seen. The diagnosis was squamous cell papilloma.
2- Orthopentomogram (OPG). It was within normal limits.
3- Baseline investigations for general anaesthesia. All were within normal
limits.
Diagnosis:
The histopathological diagnosis in this case was squamous cell
papilloma.
Treatment:
The patient was admitted and planned for excision of the swelling
under general anaesthesia. Complete excision of the swelling was done with
safe margins all around and sent for histopathology. The raw area formed
after the excision was around 5x5 cms., it was covered with a partial thickness
skin graft (fig. 35.2). The histopathological report of the specimen after
surgery confirmed the pre-operative diagnosis. Post-operative recovery of
the patient was uneventful. Healing of the surgical area occurred completely
with normal mucosa (fig. 35.3).
111

Clinical scenarios in oto-rhino-laryngology

Fig. 35.2
Partial thickness skin grafting after
excision of the swelling

Fig. 35.3
Complete healing of the surgical area
as seen after 4 weeks of surgery

Discussion:
A squamous cell papilloma is a benign tumour of epithelial origin.
It usually presents as a warty white or pink swelling with finger like processes.
Histologically it consists of stratified squamous cell epithelium supported
by a core of vascular connective tissues. It is treated by complete surgical
excision. Recurrence after complete surgical excision is very rare.

TEST YOURSELF

Read the clinical scenario given at the beginning and answers the following questions
1234-

What is the most likely diagnosis in this case?


What are the important points in history taking in this patient?
What are the important points in clinical examination of this patient?
How will you manage this case?

112

Clinical scenarios in oto-rhino-laryngology

Case 36
Clinical Scenario
A 51 years old male patient came in OPD with complaint of a non-healing
ulcer/growth on inner surface of the left cheek for last 3 to 4 months. He took medicines
from his family physician but with no relief, rather it was increasing day by day. There
was no history of pain and fever but he had lost some weight during this period.
Important points in history taking:
1- Detailed history regarding the ulcer/growth. According to the patient, it
was present for last 3 to 4 months. Initially it was very small and he
ignored it. Soon it started to increase in size till its present form (fig.
36.1). There was no aggravating or relieving factor known to him and
there was no associated factor.
2- Any history of pain in the swelling or burning sensation in the mouth.
There was no history of pain in the growth or surrounding area but he
was complaining of burning sensation in the mouth and he was unable
to take spicy food for a long time.
3- Any history of trauma to the cheek especially due to sharp tooth etc. There
was no such history.
4- Any history of using beetle nut, pan, tobacco, smoking or alcohol. He
used to eat pan with tobacco, about 14 to 15 pans per day for the last 35
years. He also used to smoke 8 to 10 cigarettes per day for about the same
period. There was no history of taking alcohol.
5- Any history of restricted mouth opening. According to him there was
some degree of restricted mouth opening especially after the start of
growth.
6- Any swelling in the neck or below mandible. There was no such history.
7- Any history of fever, anorexia or weight loss. There was no history of
fever but he had history of anorexia and weight loss.

Fig. 36.1
Patient with an ulcerative growth on
inner surface of the left cheek

113

Clinical scenarios in oto-rhino-laryngology

Important points in clinical examination:


1- General physical examination. Patient was a middle aged person of
average height and built, fully oriented in time, space and person. His
pulse was 78/minute, blood pressure was 140/85 mm of Hg., respiratory
rate was 22/minute and temperature was 98.8oF.
2- Examination of the growth. There was an ulcerative growth present on
the inner surface of the cheek, irregular in shape, about 5 cms. in size in
its maximum diameter on the left side, reaching upto the lower gingival
sulcus. Posteriorly it was reaching upto the posterior limit of the teeth.
The surface was irregular with ulceration and the margins were also
irregular and ill defined. The surrounding mucosa was showing marked
sub-mucous fibrosis and leucoplakia patches.
3- Palpation of the growth. On palpation it was non-tender, firm in consistency
with marked induration of the surrounding area and the base. There was
no bleeding on touch.
4- Examination of rest of the oral cavity. There was marked trismus due to
oral sub-mucous fibrosis. Oral hygiene was poor and the teeth were badly
stained with pan.
5- Examination of the cervical lymph nodes. There was no palpable lymph
node in the neck on any side.
6- Examination of the nose, hypopharynx and larynx. It was within normal
limits.
Differential Diagnosis:
1- Squamous cell carcinoma
2- Other malignant tumours like adenocarcinoma, adenoid cystic carcinoma
or sarcoma
3- Chronic granulomatous diseases like tuberculosis, syphilis.
4- Pyogenic granuloma
5- Benign tumours
Investigations:
1- Punch biopsy. It was done under surface anaesthesia, tissues taken from
multiple areas of the swelling and sent for histopathological examination.
The histopathology report showed moderately differentiated squamous
cell carcinoma.
2- Orthopentomogram (OPG). It was within normal limits with no
involvement of the mandible by the disease.
3- Investigations for staging the disease like X-ray chest (PA view),
ultrasonography of abdomen. These were within normal limits.
4- CT scan of the neck and oral cavity. It was not done because patient was
un-affording.
5 - Baseline investigations for general anaesthesia. All were within normal
limits.

114

Clinical scenarios in oto-rhino-laryngology

Diagnosis:
The diagnosis in this case was squamous cell carcinoma of the left
cheek. The clinical stage of the disease was T3N0M0.
Treatment:
The patient was admitted and planned for surgery. As the patient had
extensive T3 growth with marked mouth opening restriction so intra-oral
excision was not possible. Lower lip splitting incision was planned for excision
of the growth. Secondly there was no clinically palpable cervical lymph node
but the primary tumour was very extensive, so selective supra-omohyoid neck
dissection was also planned. Fig. 36.2 shows pre-operative marking for the
incision used. Complete excision of the primary tumour was done with a safe
margin of 1 cm all around, along with selective supra-omohyoid neck dissection
(removal of level I to III lymph nodes). Raw area of the oral cavity was
covered with partial split thickness skin graft taken from the thigh. Drain
applied and wound closed in layers (fig. 36.3). Specimen removed was sent
for histopathology, which re-confirmed the diagnosis of moderately differentiated
squamous cell carcinoma. All removed lymph nodes were free from malignancy.
Post-operative recovery was uneventful and patient was well and disease free
in the follow-up period.

Fig. 36.2
Patient with pre-operative marking for
the surgical incision

Fig. 36.3
Patient after completion of the surgery
with stitched wound

115

Clinical scenarios in oto-rhino-laryngology

Discussion:
Neoplasia of the oral cavity are classified as benign and malignant.
Majority of the tumours in the oral cavity are malignant. Among the malignant
tumours, squamous cell carcinoma accounts for about 95% of the cases. The
rest being salivary gland tumours, lymphoma and melanoma.
The incidence of oral squamous cell carcinoma varies worldwide, but
its incidence in our region is very high. Aetiological factors responsible for
this higher incidence are consumption of pan, chalia (beetle nut) and tobacco
in different combination. The other factors are smoking, dietary deficiencies
and alcohol use. The role of dietary deficiency is not exactly known but
deficiency of vitamin B complex, iron and other antioxidants may be
responsible. There is a definite male preponderance with a peak age incidence
of 50 to 60 years.
According to the UICC classification (Union Internationale Center
le Cancer), the T staging of oral cancer is according to the size of the
tumour:
T1 = tumour less than 2 cms in maximum diameter.
T2 = tumour between 2 and 4 cms.
T3 = tumour between 4 and 6 cms.
T4 = tumour more than 6 cms or tumour of any size extending to neighboring
structures like bone, muscles, nerves or skin.

TEST YOURSELF

Read the clinical scenario given at the beginning and answers the following questions
1234567-

What is the most likely diagnosis in this case?


What are the important points in history taking in this patient?
What are the important points in clinical examination of this patient?
What investigations are required in this case?
How will you treat this case?
What are the aetiological factors for squamous cell carcinoma of the oral cavity?
Outline TNM classification for oral carcinoma?

116

Clinical scenarios in oto-rhino-laryngology

Case 37
Clinical Scenario
A 42 years old male patient presented with a growth on the upper surface of the
tongue for last 3 months (fig 37.1). Initially it was very small but it increased in size
rapidly. He was a pan chewer and Hukka smoker for last 25 years.

Fig. 37.1
Patient with a growth on the tongue
involving dorsal surface, tip and lateral
margin on the right side

Important points in history taking:


1- Detailed history regarding the growth. According to the patient, it was
present for last 3 months and he did not know how it started. Initially it
was very small but soon it started to increase in size till its present form
(fig. 37.1). There was no aggravating or relieving factor known to him
and there was also no associated factor.
2- Any history of pain in the swelling or burning sensation in the mouth.
There was no history of pain in the growth or surrounding area but he
had history of burning sensation in the mouth.
3- Any history of using beetle nut, pan, tobacco, smoking or alcohol. He
used to eat pan with tobacco and he was a Hukka smoker for last 25
years. He used to take 8-10 pan per day. There was no history of consumption
of alcohol.
4- Any swelling in the neck or below mandible. There was no such history.
5- Any history of fever, anorexia or weight loss. There was no such history.
Important points in clinical examination:
1- General physical examination. Patient was a middle aged person of
average height and built, fully oriented in time, space and person. His
vital signs were within normal limits and no positive finding on general
physical examination.
2- Examination of the growth. There was a growth present on the tongue
117

Clinical scenarios in oto-rhino-laryngology

involving dorsal surface, tip and lateral margin on the right side and
going on the under surface as well. It was irregular in shape, 4x4 cms.
in size, surface was irregular with some small finger like projections.
The margins were also irregular but well defined and adjacent mucosa
was normal looking. On palpation it was non-tender, firm in consistency
with induration of the surrounding area and deeper structures and not
bleeding on touch.
3- Examination of the cervical lymph nodes. There was no palpable lymph
node in the neck on any side.
4- Examination of rest of the oral cavity and oropharynx. Oral hygiene was
poor, teeth were stained with pan and mouth opening was slightly
restricted.
5- Examination of the hypopharynx and larynx. It was within normal limits.
Differential Diagnosis:
1- Benign tumours like squamous papilloma.
2- Malignant tumours like verrucous carcinoma or squamous cell carcinoma.
Investigations:
1- Punch biopsy. It was done under surface anaesthesia, tissues taken from
multiple areas of the swelling and sent for histopathological examination.
The histopathology report showed verrucous carcinoma.
2- Baseline investigations for general anaesthesia. All were normal.
Diagnosis:
The histopathological diagnosis in this case was verrucous carcinoma
of the tongue. The clinical staging was T2N0M0.
Treatment:
The patient was admitted and planned for partial glossectomy under
general anaesthesia. Complete excision of the swelling was done with safe
margins all around and primary closure done. Specimen sent for histopathology
which confirmed the pre-operative diagnosis. Post-operative recovery of the
patient was uneventful. Patient was disease free in the follow-up period.
Discussion:
Squamous cell carcinoma is the commonest tumour of the oral cavity
and the lips. The incidence of squamous cell carcinoma according to different
sites of the oral cavity is as follows:
- Tongue
35%
Lateral border 31%
Tip
2%
Dorsum
2%
- Floor of the mouth
30%
Anterior
25%
Lateral
5%
118

Clinical scenarios in oto-rhino-laryngology

- Lower alveolus
- Buccal mucosa
- Upper alveolus
- Hard palate
- Retro-molar trigone

15%
10%
5%
3%
2%

Verrucous carcinoma is a variant of squamous cell carcinoma and


has a characteristic warty or papillary appearance usually grayish in colour.
It is a low grade malignant tumour which rarely metastasizes to regional
lymph nodes and never distantly. It has a very good prognosis and treated
mainly by surgical excision with safe margins.

TEST YOURSELF

Read the clinical scenario given at the beginning and answers the following questions
123456-

What is the most likely diagnosis in this case?


What are the important points in history taking in this patient?
What are the important points in clinical examination of this patient?
How will you manage this case?
What is verrucous carcinoma and what is its overall prognosis?
What is the incidence of squamous cell carcinoma at different sites of the oral cavity?

119

Clinical scenarios in oto-rhino-laryngology

Case 38
Clinical Scenario
A father brought his 4 years old child in emergency with the complaint that his
child had ingested a foreign body (coin) 2 hours back. After ingestion of coin he was
complaining of pain in the throat and he is unable to swallow any food.
Important points in history taking:
1- Any history of breathlessness or cyanosis. In this case there was no such
history.
2- Any history of dysphagia or odynophagia. In this case patient was unable
to swallow any food or liquid after ingestion of foreign body. Before this
incident the child had no symptoms.
Important points in clinical examination:
1- General physical examination. The child was fully conscious but slightly
anxious and breathing comfortably.
2- Vital signs examination. All were within normal limits.
3- Check for cyanosis or breathlessness. There was no cyanosis or
breathlessness.
4- Check for dehydration. Patients was not dehydrated.
5- Examination of the throat. There was pooling of saliva in the oral cavity,
otherwise there was no significant finding.
6- Check for laryngeal crepitus. It was absent.
7- Indirect laryngoscopy. It was not possible to perform indirect laryngoscopy
because the patient was an anxious and non-cooperative child.
8- Auscultation of the chest. There was no positive finding on auscultation
of the chest.
Investigations:
1- Plain X-ray chest and neck (AP view). It showed a radiopaque circular
foreign body impacted at the lower part of the neck (fig. 38.1). As the
foreign body was a coin and in AP view it was appearing as circular, it
means it was impacted in the hypopharynx (not in the larynx). For
localization of other foreign bodies, a plain X-ray in the lateral view is
also required to diagnose whether it is in hypopharynx or larynx.
Diagnosis:
This was a case of impacted foreign body (a coin) in the hypopharynx.
Treatment:
The patient was admitted in the hospital and planned for removal of
120

Clinical scenarios in oto-rhino-laryngology

Fig. 38.1
X-ray chest and neck (AP view)
showing a radiopaque circular foreign
body (coin) impacted in hypopharynx

foreign body through endoscopy under general anaesthesia. After endotracheal


intubation rigid anterior commissure type of direct laryngoscope was passed
and hypopharynx examined (fig. 38.2). The coin was visible, impacted at
the crico-pharyngeous. Foreign body was removed with the help of crocodile
forceps. Post-operative recovery was uneventful and the patient was discharged
from the hospital after 6 hours.

Fig. 38.2
Rigid anterior commisure type of
direct laryngoscope
with crocodile forceps

Discussion:
Foreign body impaction in the pharynx and oesophagus is quite
common. Children are most commonly the victims, as they have tendency to
put different objects in the mouth and during playing these objects get impacted
in the throat. In adults foreign body may get impacted accidentally or in
psychotic patients by themselves to commit suicide.
Commonly encountered foreign bodies in the digestive tract are coins,
food bolus (usually meat), bones, pins and dentures etc. Digestive tract foreign
bodies are likely to be impacted either at the site of anatomical narrowing or
sometimes at pathological narrowing like stricture, stenosis or neoplasia. The
commonest site of impaction of ingested foreign bodies is at or above the crico121

Clinical scenarios in oto-rhino-laryngology

pharyngeous sphincter. In most of the cases removal of the foreign body is


possible through rigid endoscopy (laryngoscopy or oesophagoscopy) depending
upon the site of impaction.

TEST YOURSELF

Read the clinical scenario given at the beginning and answers the following questions
1234-

What are the possible sites of impaction of an ingested foreign body?


What is the probable site of impaction in this patient?
How will you investigate this case?
How will you manage this case?

122

Clinical scenarios in oto-rhino-laryngology

Case 39
Clinical Scenario
A 8 years old girl presented with a swelling in floor of the mouth under the
tongue for last few months. She also complained of discomfort and difficulty in chewing
and swallowing for last one month due to the swelling. Clinical examination showed
a smooth, soft and fluctuant swelling in the floor of the mouth (fig. 39.1).

Fig. 39.1
Patient with a swelling in floor of the
mouth

Important points in history taking:


1- Detailed history regarding the swelling. According to the patient, she
noticed this swelling few months before when it was very small. Then
it started to increase in size day by day.
2- Any external swelling in the neck, sub-mental, sub-mandibular region.
There was no external swelling.
3- Any change in size of the swelling in relation to meals. There was no
such change.
4- Any history of pain in the swelling. It was painless.
5- Any history of bleeding or discharge from the swelling. There was no
such history.
6- Any history of fever. There was no history of fever.
Important points in clinical examination:
1- Inspection of the swelling. There was a single, smooth, roughly oval
shaped, slightly bluish coloured swelling of 4 cms. in maximum diameter
was present in floor of the mouth (fig. 39.1).
2- Palpation of the swelling. It was non-tender, soft, fluctuant and nonreducible swelling.
3- Bimanual palpation of floor of the mouth. Nothing significant.
4- Examination of the tongue movements. It was normal.
123

Clinical scenarios in oto-rhino-laryngology

5- Palpation of cervical lymph nodes. No palpable lymph nodes.


Differential Diagnosis:
1- Ranula
2- Haemangioma
3- Sialoadenitis
Investigations:
1- Baseline investigations for general anaesthesia. All were normal.
Diagnosis:
This was a case of simple ranula.
Treatment:
The patient was admitted and planned for marsupialization under
general anaesthesia, where roof of the ranula was removed and the inner
lining was stitched with the oral mucosa. Post-operative recovery was
uneventful.
Discussion:
Ranula is a retention cyst in the floor of the mouth arising from the
mucous gland or from sub-mandibular or sub-lingual salivary gland or its
ducts. There are two types of ranula:
1- Simple ranula: The retention cyst is limited to the floor of mouth.
2- Plunging ranula: The cyst may extend into the tissues of the neck and
presents externally in sub-mental or sub-mandibular region. It is due to
extravasation of its content into the soft tissues below mylohyoid muscle.

TEST YOURSELF

Read the clinical scenario given at the beginning and answers the following questions
12345-

What is the most likely diagnosis in this case?


What are the important points in history taking and clinical examination in this patient?
What are the differential diagnosis in this case?
How will you manage this case?
What is ranula and what are its different types?

124

Clinical scenarios in oto-rhino-laryngology

Section IV

LARYNX & TRACHEA


Case 40Case 41Case 42Case 43-

Vocal nodules
Tracheostomy
Carcinoma of the larynx
Foreign body of the bronchus

125

127
130
133
138

Clinical scenarios in oto-rhino-laryngology

Case 40
Clinical Scenario
A 38 years old male primary school teacher by profession came in OPD with
the complaints of voice change/hoarseness for the past six to seven months. Hoarseness
was continuous and increasing day by day slowly. There was no definite history of fever
or pain in the throat. He had never done smoking in his life.
Important points in history taking:
1- Occupation/vocal abuse. There was definite history of vocal abuse as he
was a primary school teacher by profession.
2- Detailed history regarding hoarseness. According to him the onset of
hoarseness was insidious. It was continuous and progressively increasing.
It was aggravated by vocal abuse and relieved to some extent by voice
rest. It was not associated with respiratory distress, stridor or any other
airway problem.
3- History of neck trauma or surgery. There was no such history.
4- Smoking/alcohol/exposure to chemicals or fumes. He was a non-smoker
and had no history of alcohol consumption or exposure to chemicals.
5- Past history of tuberculosis/ tuberculosis in the family. There was no
such history.
6- Fever. There was no history of fever.
7- Weight loss/anorexia/dysphagia. There was no such history in this case.
Important points in clinical examination:
1- General physical examination. Only positive finding on general physical
examination was hoarse voice.
2- Indirect laryngoscopy. It is very important to examine the interior of the
larynx especially vocal cords. This patient was very anxious during
indirect laryngoscopy so the vocal cords could not be seen.
3- Flexible fiber-optic laryngoscopy. This is of much help especially in
anxious persons where indirect laryngoscopy is difficult. On flexible
laryngoscopy two nodular thickenings were found at the junction of
anterior one-third and posterior two-third of the true vocal cords. Both
vocal cords were found to be fully mobile.
4- Neck nodes palpation. In this case there was no palpable cervical lymph
node.
5- Other ENT examination. It was within normal limits.
Diagnosis:
The diagnosis in this case was vocal nodules.
127

Clinical scenarios in oto-rhino-laryngology

Differential Diagnosis:
The other causes of chronic hoarseness should be considered in the
differential diagnosis like:
1- Chronic laryngitis
2- Laryngeal tuberculosis
3- Intubation granuloma
4- Benign laryngeal tumour e.g. papilloma
5- Malignant laryngeal tumour e.g. squamous cell carcinoma
6- Vocal cord paralysis e.g. after thyroid surgery, bronchogenic carcinoma
7- Vocal cord polyp
8- Laryngocoele
Investigations:
1- Baseline investigations for general anaesthesia when planned for surgery
like blood complete picture, ESR, random blood sugar, blood coagulation
studies, urine D/R and X-ray chest (PA view). All were within normal
limits.
Treatment:
After flexible laryngoscopy and baseline investigations, patient was
planned for micro-laryngoscopy and excision of the vocal nodules. Appearance
of the vocal nodules on microlaryngoscopy is shown in fig 40.1. In
microlaryngoscopy an operating microscope was used with an objective lens
of 400 mm to see and treat laryngeal pathology under magnification (fig
40.2). Complete excision of the vocal nodules were done under microscope.
Post-operative recovery of the patient was uneventful. Patient was advised
voice rest for 7 to 10 days along with a broad spectrum prophylactic antibiotic.
Patients voice became normal after that. He was also advised to avoid vocal
abuse and shouting thereafter.
Discussion:
It is a condition which occurs in persons who use their voice
excessively with straining or faulty production. It is also called singers

Fig. 40.1
Vocal nodules as seen on microlaryngoscopy

128

Clinical scenarios in oto-rhino-laryngology

Fig. 40.2
Micro-laryngoscopy procedure

node or screamers node. This condition is seen in singers, teachers,


hawkers, actors etc. It is also seen in mothers of young children who shout
a lot and persons talking to the deaf. These people sometimes squeeze their
voice. As a result of this small haematoma is formed at the point of maximum
impact of vocal cords. Due to repeated straining this haematoma may organize
and forms nodular thickening. These are always bilateral, grayish white in
colour and at the junction of anterior one-third and posterior two-third of
the true vocal cords.

TEST YOURSELF

Read the clinical scenario given at the beginning and answers the following questions
12345-

What is the most likely diagnosis in this case?


What are the differential diagnosis in this case?
How will you investigate this case?
How will you manage this case?
What is the pathology of vocal nodules?

129

Clinical scenarios in oto-rhino-laryngology

Case 41
Clinical Scenario
A call recieved from ICU for tracheostomy in a 29 years old female patient who
is on artificial respiration (ventilator) for last 4 days with endotracheal intubation (fig.
41.1). Four days back she had undergone some surgery under general anaesthesia and
developed cerebral hypoxia during anaesthesia and is on ventilator after that.

Fig. 41.1
ICU patient on artificial ventilation
with endotracheal tube

Important points in history taking and clinical examination:


1- Clinical assessment for how long the patient will remain on ventilator.
For short duration artificial ventilation, endotracheal intubation is preferable
but for prolonged artificial ventilation (more than 72 hours), tracheostomy
is better. This patient was already on endotracheal intubation for the last
4 days and still expected time period for recovery is unknown, so
tracheostomy should be done in this patient.
2- Endotracheal tube if remained for a longer time, it causes trauma to lips,
oral cavity, pharynx, vocal cords and sub-glottic area. So after performing
tracheostomy and removal of endotracheal tube, look for any injury in
these areas.
Investigations:
No special investigation was required in this case.
Treatment:
Patient was planned for elective standard tracheostomy in operation
theatre. In the presence of an anaesthetist with all monitoring and artificial
ventilation through endotracheal tube, patient was positioned on operation
table with sand bag under the shoulders and head ring under the head. Local
130

Clinical scenarios in oto-rhino-laryngology

anaesthesia was administered and a horizontal incision was given two fingers
above the supra-sternal notch from anterior border of one sterno-cleidomastoid
muscle to the other. Flap raised, strap muscles seperated in the midline,
thyroid isthmus retracted down, trachea opened and Portex tracheostomy
tube inserted (fig. 41.2 and 41.3). Before opening the trachea anaesthetist
was asked to remove endotracheal tube and after insertion of the tracheostomy
tube ventilation circuit was connected to it.

Fig. 41.2
Portex tracheostomy tube with cuff

Fig. 41.3
Patient after tracheostomy

Discussion:
In comatosed ICU patient where tracheostomy is needed, we have
two options:
1- Standard tracheostomy in operation theatre
2- Percutaneous tracheostomy (PCT) in ICU
Shifting of the patient to operation theatre is difficult as these patients
are already on ventilator. To avoid this problem we have option of percutaneous
tracheostomy in ICU. In this patient we performed tracheostomy in operation
theatre because PCT kit is expensive and was not available at that time. In
percutaneous tracheostomy (PCT), first a guide wire is introduced percutaneously into the trachea and then dilators of progressive thickness are
131

Clinical scenarios in oto-rhino-laryngology

introduced through this guide wire so as to make a bigger hole. Finally when
the hole is large enough tracheostomy tube is passed through the same guide
wire. Advantages of PCT are:
1- No need of operation theatre.
2- Hazards of shifting such patients to OT can be avoided.
3- Smaller and more aesthetic wound.
4- Less operative bleeding.
5- Less chances of wound infection.
But this procedure is hazardous in the following situations and should
not be performed:
1- Patients having short and thick neck.
2- Goitre.
3- Patients below 15 years of age.
4- Previous tracheostomy patients.
5- Previous laryngeal or neck surgery patients.

TEST YOURSELF

Read the clinical scenario given at the beginning and answers the following questions
1234-

What are the options available in this patient for performing tracheostomy?
Briefly outline the steps of tracheostomy operation.
What is percutaneous tracheostomy? and what are its advantages?
What are the contra-indications for percutaneous tracheostomy?

132

Clinical scenarios in oto-rhino-laryngology

Case 42
Clinical Scenario
A 49 years old male patient came in OPD with complaints of change of voice,
respiratory distress on exertion and sometimes noisy breathing for last 3 to 4 months.
He was a smoker for last 35 years taking about 20 to 25 cigarettes per day.
Important points in history taking:
1- Detailed history regarding his symptoms. According to the patient he
was in his usual state of health 3 to 4 months before when he noticed
change in his voice (hoarseness). Onset of hoarseness was insidious and
initially he ignored it but after sometime it started to increase for which
he consulted his family physician who gave him some medication.
Hoarseness was continuous and progressively increasing with no definite
aggravating or relieving factors. After sometime he also experienced
respiratory distress or dyspnoea on exertion and sometimes noisy breathing
as well. It was relieved by taking rest.
2- Any history of dysphagia. According to patient he had no difficulty in
swallowing but sometimes he felt that something was obstructing his
throat or something was present in his throat.
3- Any history of fever, pain or cough. There was no history of fever or
pain but he had cough on and off.
4- Any history of anorexia or weight loss. He had history of anorexia and
weight loss for last few months.
5- Any history of neck swelling. There was no such history.
6- Any history of previous surgery. There was no history of any surgery in
the past.
7- Any family history of throat diseases. His father had carcinoma of the
larynx and he had been operated for it about 20 years back.
8- Occupation of the patient and history of vocal abuse. He was an electrician
and there was no history of vocal abuse.
Important points in clinical examination:
1- General physical examination. Patient was a middle aged person of
average height and lean built, fully oriented in time, space and person.
His voice was hoarse and breathing was laboured. On number counting
he was not able to count more than 5 or 6 in one breath. His pulse was
86 per minute, respiratory rate was 24 per minute, temperature was
98.7oF and blood pressure was 140/90 mm. of Hg. There was no cyanosis,
anaemia or other positive finding on general physical examination.
2- External examination of the neck. There was widening or splaying of
the thyroid cartilages (fig. 42.1). Laryngeal crepitus was positive.
133

Clinical scenarios in oto-rhino-laryngology

Fig. 42.1
Patient showing widening or splaying
of the thyroid cartilages

3- Examination of the oral cavity and oropharynx. There was nothing


significant on it.
4- Indirect laryngoscopy. Indirect laryngoscopy was difficult with overhanging
epiglottis.
5- Fiber-optic flexible direct laryngoscopy. It showed an extensive growth
on the right side involving full length of true vocal cord and extending
both above and down to involve supra-glottic as well as sub-glottic region.
Right vocal cord was fixed and the airway was compromised due to
growth (fig. 42.5).
6- Examination of cervical lymph nodes. There was no palpable cervical
lymph node.
7- Rest of ENT and systemic examination. There was no significant finding.
Differential Diagnosis:
1- Squamous cell carcinoma
2- Other malignant tumour
3- Benign tumour
4- Other causes of chronic hoarseness
Investigations:
1- Direct laryngoscopy and biopsy. It was done under general anaesthesia
and all the findings of flexible laryngoscopy were confirmed. Punch
biopsy from multiple areas were taken and sent for histopathology which
showed poorly differentiated squamous cell carcinoma.
2- CT scan of the neck. It showed an extensive growth present in the larynx
on right side involving supra-glottic, glottic and sub-glottic region causing
marked narrowing of the airway (fig. 42.2).
3- Investigations for distant metastasis like X-ray chest (PA view),
ultrasonography of the abdomen etc. All were within normal limits.
4- Other baseline investigations including blood complete picture, coagulation
profile, liver function tests, renal function tests, serum electrolytes etc.
All were within normal limits.
134

Clinical scenarios in oto-rhino-laryngology

Fig. 42.2
CT scan of the neck (axial view)
showing growth in larynx with airway
narrowing

Diagnosis:
This was a case of squamous cell carcinoma of the larynx. Clinical
staging was T4N0M0.
Treatment:
The patient was planned and prepared for total laryngectomy under
general anaesthesia. Patient needed pre-operative tracheostomy due to
extensive growth, anaesthetist was unable to pass endotracheal tube. U
shaped modified Gluck/Sorensons incision was used (fig. 42.3). Whole of
the larynx was removed along with hyoid bone and muscles (fig. 42.4 and
42.5). Pharynx was reconstructed and tracheal opening stitched to the skin.
Drain applied and wound closed in layers. Post-operative recovery was
uneventful.
Discussion:
Squamous cell carcinoma of the larynx is common and it is among the
list of top ten commonest malignancies in Pakistan. It has a high rate of cure
and a good prognosis. Early diagnosis and correct treatment gives a long
survival. Laryngeal carcinoma has a definite male preponderance but the sex
incidence varies worldwide. In Western countries the incidence of laryngeal

Fig. 42.3
Patient with pre-operative marking for
U shaped modified Gluck/Sorensons
incision

135

Clinical scenarios in oto-rhino-laryngology

Fig. 42.4
Patients open neck after removal of
the larynx

Fig. 42.5
Specimen of the larynx after removal
showing growth on the right side

carcinoma in females is increasing, probably because of the smoking habits.


The peak age of presentation is between forty and sixty years. Laryngeal
carcinoma in childhood is a rare entity. Like most of the other malignancies,
the aetiology of laryngeal carcinoma is unknown. A number of possible
aetiological factors have been described. Smoking is considered as most
important aetiological factor. The risk is directly proportional to the number of
cigarettes smoked per day, duration of smoking and use of filter. Tobacco in
other forms like hukka, berri, tobacco chewing etc. also has the same effect.
Alcohol consumption is another important causative factor especially in
western countries. Other factors, which are associated with the increase
incidence of laryngeal carcinoma, include environmental pollution, vocal abuse,
radiation to the neck and occupational exposure to asbestos and certain other
metals. In childhood laryngeal carcinoma radiation is important, as malignant
change may occur in juvenile laryngeal papillomatosis treated with radiation.
The squamous cell carcinoma develops not only from areas normally
covered by squamous epithelium but it may also arise from areas that are
lined by ciliated columnar and transitional epithelium. According to the site
of origin and involvement, laryngeal carcinoma are classified as:
1- Supra-glottic
136

Clinical scenarios in oto-rhino-laryngology

2- Glottic
3- Sub-glottic
4- Transglottic
Most of the carcinomata develop on the vocal cords (glottic carcinoma),
about 76% of total cases. Anterior half of the vocal cord is more commonly
involved than the posterior half. Supra-glottic carcinoma arises from any
part of supraglottis and comprises about 19%. Those that arise from the subglottic region are sub-glottic carcinomata, these are rare about 1-5% of total
cases. Transglottic carcinomata are glottic lesions that invade both the supraglottic and sub-glottic regions of the larynx. These tumours have deep
extension, which cross the laryngeal ventricle vertically to involve two or
more anatomical areas.

TEST YOURSELF

Read the clinical scenario given at the beginning and answers the following questions
123456-

What are the differential diagnosis in this case?


What are the important points in history taking and clinical examination in this case?
How will you investigate this case?
How will you manage this case?
What are the aetiological factors of carcinoma of the larynx?
What is the commonest site for origin of carcinoma of the larynx?

137

Clinical scenarios in oto-rhino-laryngology

Case 43
Clinical Scenario
A 3 years old boy was referred from paediatric department for opinion. This
boy first came with the complaint of cough, fever and breathlessness about two weeks
back and diagnosed as a case of lower respiratory tract infection and treated accordingly.
The condition improved with treatment but after stopping the drugs he again developed
same condition. Plain X-ray chest (PA view) was done which showed a radiopaque
metallic foreign body (a metallic nail) impacted in the left main bronchus (fig. 43.1).

Fig. 43.1
X-ray chest (PA view) showing a
radiopaque foreign body (metallic nail)
impacted in the left main bronchus

Important points in history taking:


1- Any history of foreign body ingestion. According to his parents there
was no history of foreign body ingestion. Some construction work was
going on in their house and the child might have taken this foreign body
in their absence.
2- Any history of choking attack. There was no such history.
Important points in clinical examination:
1- General physical examination. The child was looking ill but fully conscious
and oriented. His pulse was 116 per minute, respiratory rate was 30 per
minute, temperature was 101oF. There was no cyanosis or other positive
finding on general physical examination.
2- Inspection of the chest. It showed slightly decreased movement of the
chest on left side.
3- Auscultation of the chest. Air entry was reduced on the left side with
crepts all over the chest.
4- Percussion of the chest. It has a slightly dull note on left side of the chest
as compared to right side.
138

Clinical scenarios in oto-rhino-laryngology

Investigations:
1- Plain X-ray chest (PA view). It was already done which showed a
radiopaque foreign body (metallic nail) in the left main bronchus with
prominent vascular marking (fig. 43.1).
2- Baseline investigations for general anaesthesia. All were within normal
limits.
Diagnosis:
This was a case of impacted foreign body (metallic nail) in the left
main bronchus.
Treatment:
The patient was admitted and planned for removal of foreign body
through bronchoscopy under general anaesthesia. Parenteral broad spectrum
antibiotic was started immediately with other supportive treatment. Anaesthetist
opinion was taken for fitness and planned for elective bronchoscopy next
day. Bronchoscope was passed into the trachea till opening of the left main
bronchus which showed marked oedema and granulation tissues. With some
manipulation foreign body became visible which was removed with a
crocodile forceps. Post-operative recovery was uneventful, he remained in
hospital for two days and his condition improved very much during this
period.
Discussion:
Most of the foreign bodies entering into the respiratory tract, pass
through the larynx and lodge in the trachea or mostly in bronchi. Bronchus on
the right side is involved more than the left side, because right bronchus is
wider and more in line with the trachea. Clinical features of impacted foreign
body in the respiratory tract depend upon the site of impaction, size and nature
of the foreign body. Cough and dyspnoea may be present initially at the time
of accident. Vegetative foreign bodies like chalia and peanuts produce an intense
inflammatory reaction in the mucosa. Symptoms of acute tracheo-bronchitis
may be present. Emphysema, atelactasis, lung abscess, pneumonia or even lung
collapse may occur. Non-vegetative foreign bodies like whistle, needle, metallic
pieces etc. may remain silent for sometimes because of minimal local
inflammatory reaction. Atelactasis occurs if the foreign body completely
obstructs the lumen. In partial obstruction, obstructive emphysema may
develop.
Radiopaque foreign body may be seen on plain radiograph. Radiolucent
foreign body especially vegetative may produce changes on radiology due
to inflammatory reaction or obstruction. Emphysema, pneumonia, lung
collapse and bronchitic changes may be seen.
Removal of the tracheal and bronchial foreign body is done through
rigid type of bronchoscopy. Rigid bronchoscopy is performed under general
anaesthesia where a small lumen size endotracheal tube is preferred. Patient
lies in supine position with flexion at cervical vertebrae and extension of head
139

Clinical scenarios in oto-rhino-laryngology

Fig. 43.2
Bronchoscopy procedure

at atlanto-occipital joint. This brings the laryngo-tracheal axis in line with the
oral axis (fig. 43.2). Bronchoscope is held in the right hand and introduced
through the mouth into the larynx. Bronchoscope is then advanced and whole
tracheo-bronchial tree is examined. Foreign body can be removed through the
rigid bronchoscope by holding it with crocodile forceps. If foreign body is large
and it is not possible to pass it through the bronchoscope, then bronchoscope is
also withdrawn along with the foreign body.

TEST YOURSELF

Read the clinical scenario given at the beginning and answers the following questions
12345-

What is the probable site of impaction of foreign body in this patient?


How will you manage this case?
What are the clinical features of impacted foreign body in the trachea or bronchus?
How is the foreign body removed from the trachea or bronchus?
Outline the steps of rigid bronchoscopy procedure.

140

Clinical scenarios in oto-rhino-laryngology

Section V

HEAD & NECK


Case 44Case 45Case 46Case 47Case 48Case 49Case 50-

Ludwigs angina
Branchial cyst
Retro-pharyngeal abscess
Sub-mandibular salivary calculus
Parotid gland pleomorphic adenoma
Multinodular goiter (MNG)
Tuberculous cervical lymphadenopathy

141

143
146
149
152
155
158
162

Clinical scenarios in oto-rhino-laryngology

Case 44
Clinical Scenario
A 42 years old male patient had a history of tooth extraction about 5 days back
of the left lower jaw tooth. He presented in OPD with complaint of painful swelling and
redness below his chin and lower jaw, which started three days back and increased
rapidly in last two days. External examination findings are shown in fig. 32.1.

Fig. 32.1
Patient with marked swelling and
redness over the sub-mental and
sub-mandibular region, more on the
left side

Important points in history taking:


1- Detailed history regarding swelling. According to the patient he noticed
a painful swelling in the floor of mouth and below mandible three days
back. Rapidly the swelling increased next day along with severe pain.
There was no aggravating or relieving factor for the swelling. It was
associated with pain on swallowing and excessive salivation.
2- Detailed history regarding pain. Pain started before the swelling. Initially
it was mild but soon became moderate to severe. It was continuous, sharp
cutting in nature and localized. It aggravated by swallowing and relieved
to some extent by taking analgesics.
3- Any history of fever. Patient had history of fever without rigors, around
101 to 102oF.
4- Detailed history regarding his dental problem. According to the patient
he had dental carries in multiple teeth. He developed pain and swelling
around gums in one tooth on left lower side for which he consulted a
dentist. Tooth extraction was done five days back.
5- Any history of respiratory distress, dyspnoea or stridor. There was no
such history.
Important points in clinical examination:
1- General physical examination. Patient was a middle aged person of
143

Clinical scenarios in oto-rhino-laryngology

2-

3-

456-

average height and slightly obese built, looking ill but fully oriented in
time, space and person. His pulse was 100/minute, blood pressure was
130/85 mm of Hg., respiratory rate was 22/minute and temperature was
100.8oF.
External examination of the neck. It showed a generalized swelling
involving sub-mental and sub-mandibular region on both sides but more
marked on left side (fig. 32.1). The skin over the swelling was smooth,
red and congested. The margins of the swelling were ill defined. There
was no pulsation, discharge, sinus, hyperpigmentation or reducibility.
On palpation temperature was raised over the swelling. It was tender,
firm, non-fluctuant, smooth with ill defined margins. Trans-illumination
was negative.
Examination of the oral cavity and throat. It showed restricted tongue
movement with swelling in floor of the mouth. Pooling of saliva was
also present in the oral cavity. Signs of tooth extraction were present in
first molar tooth on the left lower side with swelling around the gums.
Bi-manual palpation. It was not done because of severe pain.
Any signs of respiratory distress, dyspnoea, cyanosis or stridor. All were
absent.
Rest of ENT examination was within normal limits.

Diagnosis:
The diagnosis in this case was Ludwigs angina.
Investigations:
1- Blood complete picture. It showed increased in the total white cell count
with 80% neutrophils.
Treatment:
The patient was admitted in the hospital for further management and
observation. Parenteral antibiotics (amoxicillin with clavulanic acid and
metronidazole) were started along with analgesic and anti-inflammatory drug.
Patient showed marked improvement within 48 hours. He was discharged
on oral medications with uneventful recovery.
Discussion:
Ludwigs angina is the infection of sub-mandibular space. This space
is bounded above by the mucous membrane of the floor of mouth and tongue
and below by deep fascia which extends from the hyoid to the mandible. It
is divided into two compartments by the mylohyoid muscle:
1- Sub-lingual: It lies above the mylohyoid muscle.
2- Sub-maxillary and sub-mental: It lies below the mylohyoid muscle.
In about 80% of the cases, infection reaches to this space by extension
of dental root infection. The other causes are sub-mandibular sialadenitis,
penetrating injuries of the floor of mouth and mandibular fractures. Most of
the infections of this space are caused by mixed aerobic and anaerobic
144

Clinical scenarios in oto-rhino-laryngology

organisms. Common organisms responsible are streptococci, staphylococci,


E. coli and bacteroids.
If Ludwigs angina is not treated properly it can give rise to following
complications:
1- Laryngeal oedema
2- Mediastinitis
3- Lower respiratory tract infection
4- Parapharyngeal abscess
5- Retro-pharyngeal abscess

TEST YOURSELF

Read the clinical scenario given at the beginning and answers the following questions
1234567-

What is the diagnosis in this case?


What are the important points in history taking in this patient?
What are the important points in clinical examination of this patient?
How will you manage this case?
What is the pathology of Ludwigs angina?
What are the boundries and compartments of sub-mandibular space?
What complications can occur if Ludwigs angina is not treated early?

145

Clinical scenarios in oto-rhino-laryngology

Case 45
Clinical Scenario
A 17 years old boy presented with a swelling on lateral aspect of the neck on
left side (fig 45.1). According to him this swelling was present since childhood but
caused no problem to him as it was very small and painless, but for the last two years
it had increased in size. Two months back he had undergone incision and drainage under
local anaesthesia by his family physician but swelling reappeared soon.

Fig. 45.1
Patient with swelling on lateral aspect
of the neck on left side with redness
of overlying skin

Important points in history taking:


1- Detailed history about the swelling. According to the patient this swelling
was present since childhood and he did not know how it started. Initially
it was very small and barely visible to others and it remained small for
a long time. Two years back it started to grow very slowly and during
this period it became large to its present size. There was no aggravating
or relieving factor and it was not associated with any other complaint.
2- Any history of pain. Patient stated that initially there was no pain but as
the swelling increased in size he occasionally felt mild localized pain in
the swelling. After incision and drainage of the swelling he developed
recurrent pain in the swelling which was mild to moderate in severity.
3- Any history of fever. There was no definite history of fever but occasionally
he felt that his body was warm during last two months.
4- Detailed history about the surgical procedure. According to him he
consulted his family physician regarding the swelling who did incision
and drainage under local anaesthesia. Some thick yellowish material came
out and for some times swelling disappeared but after few days it again
started to grow and became of same size as before. He also noticed redness
of the skin over the swelling and has recurrent pain after surgery.
146

Clinical scenarios in oto-rhino-laryngology

Important points in clinical examination:


1- Examination of the swelling. A rounded, smooth, single swelling of 5
cms. diameter was present on the lateral aspect of neck on the left side
just at anterior border of sternocleidomastoid muscle near its middle (fig.
45.1). The margins of the swelling were well defined. The skin over the
swelling was reddened with a scar mark of previous surgery. There was
no pulsation, discharge, opening or sinus, reducibility, increase of size
on coughing and movement of swelling with swallowing. Temperature
over the swelling was slightly raised and tenderness was present. The
skin over the swelling was mobile and swelling itself was also mobile
in both vertical and horizontal direction. The swelling was soft to firm
in consistency and transillumination was positive.
2- Examination of the throat, nose and ears. It was within normal limits.
Differential Diagnosis:
Following conditions should be considered in differential diagnosis
of a swelling in the lateral aspect of neck for long duration:
1- Branchial cyst
2- Epidermoid cyst
3- Lymphadenopathy
4- Thyroid nodule
5- Parapharyngeal space tumour
Investigations:
1- Ultrasound of the neck. It showed presence of a thick walled cyst of 6
cms. in maximum diameter. Thyroid gland was separate from the swelling
and was normal in size and texture.
2- Fine needle aspiration cytology. Aspirate mainly consist of straw coloured
fluid which contained few lymphocytes.
2- Baseline investigations for general anaesthesia. All were within normal
limits.
Diagnosis:
The diagnosis of this case was branchial cyst.
Treatment:
The patient was planned for excision of branchial cyst under general
anaesthesia. Horizontal incision was given over the swelling and skin flaps
including platysma muscle were raised above and down. Deep fascia was
cut anterior to sternocleidomastoid muscle and muscle retracted posteriorly.
Whole cyst was separated from surrounding and removed completely. Wound
closed in layers.
Discussion:
The branchial cyst is also known as lateral cervical cyst. It is usually
present in lateral part of the neck deep to sternocleidomastoid muscle at the
147

Clinical scenarios in oto-rhino-laryngology

junction of upper one-third and lower two-third. The origin of branchial cyst
is debatable and there are four theories of origin:
1- Branchial apparatus theory
2- Cervical sinus theory
3- Thymopharyngeal duct theory
4- Inclusion theory
Branchial cyst is mostly lined by stratified squamous epithelium and
contains straw coloured fluid with cholesterol crystals. In 80% of the cases,
the wall of the cyst contains lymphoid tissues. Males are slightly more
affected than females (ratio of 3:2). The peak age incidence for presentation
of branchial cyst is third decade. Majority are present in the classical upper
lateral neck but some are also present in lower neck, parotid and posterior
triangle of the neck.

TEST YOURSELF

Read the clinical scenario given at the beginning and answers the following questions
12345-

What is the most likely diagnosis in this case?


What are the differential diagnosis in this case?
How will you manage this case?
What are the theories of origin of branchial cyst?
What is the classical position of branchial cyst in the neck?

148

Clinical scenarios in oto-rhino-laryngology

Case 46
Clinical Scenario
A 28 years old male patient came with the complaint that three days back during
lunch, a sharp fish bone got penetrated into the posterior wall of the throat, half of it
was visible in the throat. He went to his family physician who removed the foreign
body. After removal he had some discomfort in the throat which increased next day.
After two days he had severe pain in the throat along with difficulty in swallowing.
Important points in history taking:
1- Detailed history regarding pain. According to the patient he had discomfort
in the throat after removal of the foreign body and next day it increased
to moderate pain. It was continuous, localized, dull in character, not
radiating to anywhere and progressively increasing and became severe
next day.
2- Detailed history regarding difficulty in swallowing. According to him,
next day after removal of the foreign body he developed difficulty in
swallowing which increased rapidly and he was unable to eat anything
and even he was not able to swallow his own saliva.
3- Any history of fever. He had fever without rigors for last one day. He
checked his body temperature in the morning and it was 101oF.
4- Any history of neck swelling. There was no such history.
5- Any history of inability to open mouth (trismus). There was no such
history.
6- Any previous history of recurrent sore throat. He had no such problem.
7- Any history of diabetes mellitus or any other immuno-compromised state.
He was non-diabetic and otherwise healthy.
8- Any history of pain or restriction in neck movements. There was no such
complaint.
9- Any history of dyspnoea, stridor or voice change. His voice was muffled
after the start of this problem.
Important points in clinical examination:
1- General physical examination. Patient was a young male looking ill and
lethargic but fully oriented. His pulse was 108/minute, blood pressure
was 115/80 mm of Hg., temperature was 101.8oF and respiratory rate
was 20/minute. His voice was muffled.
2- Examination of the oral cavity and throat. His oral hygiene was poor
with pooling of saliva in the mouth. There was a smooth bulge in the
posterior pharyngeal wall. Tonsils, pillars, soft palate and other areas
were normal.
3- Examination of the neck. There was no swelling in the neck. Cervical
149

Clinical scenarios in oto-rhino-laryngology

lymph nodes were not palpable. Neck movements were normal.


4- Indirect laryngoscopy. It was not done due to pain in the throat.
Differential Diagnosis:
1- Retropharyngeal abscess
2- Parapharyngeal abscess
3- Quinsy
4- Acute tonsillitis/pharyngitis
5- Other causes of acute sore throat
Investigations:
1- Plain X-ray neck (lateral view). It showed widening of the prevertebral
space with pushing of the larynx and trachea anteriorly (fig. 46.1). There
was no visible air fluid level.
2- Blood complete picture. It showed marked leucocytosis of 24,000/mm3
with 85% neutrophils.

Fig. 46.1
Plain X-ray neck (lateral view) showing
marked widening of the prevertebral
space

Diagnosis:
This was a case of acute retropharyngeal abscess.
Treatment:
Patient was admitted for incision and drainage of the retropharyngeal
abscess. Patient was very anxious and was not willing to underwent this
procedure under local anaesthesia, so it was done under general anaesthesia.
Endotracheal intubation was difficult with risk of spontaneous rupture of the
abscess, tube was passed with caution. A vertical incision was given on the
posterior pharyngeal wall, thick pus came out. Pus was sent for culture and
sensitivity. Post-operatively broad spectrum parenteral antibiotic was given
(amoxicillin with clavulanic acid). Culture and sensitivity report showed
heavy growth of staphylococcus aureus, sensitive to the given antibiotic.
Further recovery was uneventful and patient was discharged from the hospital
after two days.
150

Clinical scenarios in oto-rhino-laryngology

Discussion:
Retropharyngeal space is a potential space that lies behind the pharynx
between the bucco-pharyngeal fascia covering pharyngeal muscles and the
prevertebral fascia covering prevertebral muscles. It extends from base of
the skull to the posterior mediastinum as far as the level of bifurcation of the
trachea. The space is divided by a fibrous raphe in the midline into two lateral
halves called space of Gillette. Each of this space contains retropharyngeal
lymph node, which usually disappear by the age of four years. Retropharyngeal
space communicates laterally with parapharyngeal space.
Retropharyngeal abscess is the collection of pus in this retropharyngeal
space and there are two distinct clinical types:
1- Acute retropharyngeal abscess
2- Chronic retropharyngeal abscess
Acute retropharyngeal abscess is mostly caused by suppuration in
the retropharyngeal lymph nodes. This variety is commonly seen in children
below the age of 4 years. In adults acute retropharyngeal abscess may occur
as a result of penetrating injury in the posterior pharyngeal wall or cervical
oesophagus. Rarely infection may reach here from petrous apex abscess.

TEST YOURSELF

Read the clinical scenario given at the beginning and answers the following questions
12345-

What are the differential diagnosis in this case?


How will you investigate this case?
How will you manage this case?
What is retropharyngeal space? and what are its contents?
What are different clinical types of retropharyngeal abscess?

151

Clinical scenarios in oto-rhino-laryngology

Case 47
Clinical Scenario
A 36 years old male patient came with the complaints of recurrent pain and
swelling in the sub-mandibular region on right side for last many months. Swelling
increased in size whenever he used to take meals especially sour substances. He also
had pain in the swelling and floor of the mouth during meals, which remained there for
one to two hours.
Important points in history taking:
1- Detailed history regarding the swelling. According to him he did not
know how the swelling started, but first time he noticed the swelling
because of pain. Initially it was very small but progressively it increased.
Swelling usually appeared during meal time and remained there for
sometime. In between meal time it was barely visible.
2- Detailed history regrading pain. According to him he had pain in the
sub-mandibular region and floor of the mouth during meals, which was
localized, dull, mild to moderate in nature and remained for one to two
hours after meals. Pain was associated with increase in size of the swelling.
3- Any history of fever. There was no history of fever.
Important points in clinical examination:
1- Examination of the sub-mandibular region. No distinct swelling was
present in this region at the time of examination but there was a generalized
bulge of the sub-mandibular salivary gland. On palpation gland was
palpable and appeared soft to firm in consistency.
2- Examination of the floor of mouth. By asking the patient to elevate the
tongue, a diffuse bulge was seen in the floor of the mouth on the right
side (fig. 47.1).

Fig. 47.1
A diffuse bulge in floor of the mouth
on the right side

152

Clinical scenarios in oto-rhino-laryngology

3- Bimanual palpation of the floor of mouth and sub-mandibular region. In


this patient a diffuse enlargement of the sub-mandibular salivary gland
was present and a hard substance was palpable in the sub-mandibular
duct in its posterior part, which was slightly mobile along the duct. Most
probably it was a stone in the duct.
4- Rest of ENT examination. No positive finding on it.
Differential Diagnosis:
On history and clinical examination this appears to be a case of stone
in the sub-mandibular duct causing partial obstruction. The following
conditions should be considered as differential diagnosis:
1- Stricture or stenosis of the sub-mandibular duct.
2- A small tumour of the duct or its surrounding structures causing obstruction
to the duct.
3- Enlarged sub-mandibular lymph node.
4- Neoplasia of the sub-mandibular salivary gland.
Investigations:
1- Plain X-ray floor of the mouth (occlusal view). This showed a radiopaque
shadow in the region of the sub-mandibular duct (fig 47.2).
2- Sialogram. It is not done in this case as radiopaque stone was visible on
plain X-ray. If the stone was not visible on plain X-ray or some other
pathology like stricture was suspected then sialogram is indicated.
3- Other baseline investigations for general anaesthesia including blood
complete picture, ESR, random blood sugar, blood coagulation studies,
urine D/R and X-ray chest (PA view). All were normal.
Diagnosis:
This was a case of stone in the sub-mandibular salivary gland duct
(salivary calculus).
Treatment:
Patient was admitted and planned for either removal of the stone

Fig. 47.2
Plain X-ray floor of the mouth
(occlusal view) showing a radiopaque
shadow along the sub-mandibular duct

153

Clinical scenarios in oto-rhino-laryngology

Fig. 47.3
Specimen of the sub-mandibular
salivary gland with the removed stone

intra-orally or removal of the gland with stone through external approach.


The sub-mandibular duct stone is possible to be removed intra-orally by
opening the duct if it is present in the duct anteriorly. More posteriorly lying
stone or stone in the gland itself is not possible to be removed intra-orally.
In such cases whole of the sub-mandibular salivary gland is removed along
with major part of its duct. In this case, it was not possible to remove the
stone intra-orally, so external approach was used to remove the whole gland
and its duct with the stone (fig. 47.3).
Discussion:
Stone formation can occur in the duct or within the salivary gland
as a result of calcium deposition on any epithelial debris, mucous or organic
matrix. Stone formation is much common in sub-mandibular duct (90%) as
compared to the parotid duct. There are several reasons for more incidence
of stone formation in sub-mandibular duct. Sub-mandibular secretion is
mainly mucous and thick as compared to serous and thin secretion of the
parotid gland. In addition the calcium content of the sub-mandibular secretion
is higher.

TEST YOURSELF

Read the clinical scenario given at the beginning and answers the following questions
123456-

What are the differential diagnosis in this case?


How will you investigate this patient?
How will you manage this case?
What are the surgical options for removal of sub-mandibular duct stone?
In which salivary gland, incidence of stone formation is maximum?
Why the incidence of stone formation is more in sub-mandibular gland as compared to
parotid gland?
154

Clinical scenarios in oto-rhino-laryngology

Case 48
Clinical Scenario
A 44 years old male patient presented with a painless swelling near angle of
mandible on the right side for the last many years. Initially the swelling was very small
and barely visible but over a period of many years it increased gradually to its present
size (fig 48.1).

Fig. 48.1
Patient with a swelling near angle of
mandible on the right side

Important points in history taking:


1- Detailed history regarding swelling. According to the patient, swelling
was present for last many years and he did not know how it started.
Initially it was very small and he ignored it. Very slowly and gradually
it started to increase in size till its present form (fig. 48.1). There was no
aggravating or relieving factor known to him and there was no associated
factor.
2- Any history of sudden increase in size. In this case there was no such
history and swelling was increasing very slowly and gradually.
3- Any history of pain in the swelling. He occasionally felt some discomfort
in the region of swelling and there was no history of pain.
4- Any history of facial nerve paralysis/paresis or facial asymmetry. There
was no such history.
5- Any history of dry mouth and difficulty in chewing or swallowing. There
was no such history in this case.
Important points in clinical examination:
1- Examination of the swelling. A rounded, smooth, single swelling of 8
cms. diameter was present in the parotid region and angle of mandible
on right side. The margins of the swelling were well defined. The skin
over the swelling was normal and mobile. There was no pulsation,
155

Clinical scenarios in oto-rhino-laryngology

2345-

discharge, opening or sinus, reducibility or hyper-pigmentation over the


swelling. Temperature over the swelling was equal to the surrounding
with no tenderness. The swelling was firm in consistency with no
fluctuation and transillumination was negative. The swelling was only
slightly mobile and appeared to be arising from the parotid gland.
Examination of the facial nerve. It was intact.
Examination of the oral cavity and oropharynx. It was also within normal
limits.
Examination of the ear. It was within normal limits.
Examination of the cervical lymph nodes. There was no palpable lymph
node in the neck on any side.

Investigations:
1- Fine needle aspiration cytology. There was scanty aspirate which showed
presence of mixed cells of ductal origin and myo-epithelial cells. No
features of malignancy were seen in the cells. Probable diagnosis was
pleomorphic adenoma.
2- MRI/CT scan. It was not done as the patient was non-affording.
3- Baseline investigations for general anaesthesia. All were within normal
limits.
Diagnosis:
The diagnosis in this case was pleomorphic adenoma arising from
the parotid gland.
Treatment:
The patient was admitted and planned for superficial parotidectomy
under general anaesthesia. Parotidectomy incision (Blairs incision) as shown
in fig. 48.2 was given, which started near the upper part of the pinna, ran in
the pre-auricular crease till the point where lobule was attached to the face.
It turned backwards till tip of the mastoid process, from there it curved forward
parallel to lower margin of the mandible in the neck crease about two fingers

Fig. 48.2
Surgical incision for parotidectomy

156

Clinical scenarios in oto-rhino-laryngology

below the mandible. Flap was raised and reflected anteriorly. Main trunk of
the facial nerve was identified which was entering into the parotid gland.
Whole of superficial lobe of the gland was removed and wound closed in
layers after applying drain. Post-operative recovery of the patient was uneventful
and he was followed up regularly with no recurrence.
Discussion:
Although the incidence of salivary gland tumour varies in different
geographical regions, these tumours in general are uncommon. Approximately
80% of all salivary tumours are located in the parotid, 10% in the submandibular and rest of the 10% in sub-lingual and minor salivary glands. In
the parotid gland most of the tumours (about 80%) are benign. Pleomorphic
adenoma is the commonest among benign tumours of the salivary glands.
It is characterized by slow growth and clinically a benign course. It is
essentially an epithelial tumour of complex morphology, with both epithelial
and myo-epithelial tissues. That is why it is called mixed tumour or
pleomorphic. The epithelial and myo-epithelial elements are arranged in
various patterns. It is surrounded by a false capsule, which is formed in
response to expansion of tumour. This false capsule of compressed parotid
tissue varies in thickness and tumour may extend into the capsule in a
lobulated or pseudopod pattern. That is why removal of the tumour through
its capsule may lead to recurrence thus it is treated by removal of whole
superficial lobe. It is usually seen in third and fourth decade with a slight
female preponderance.
The most important complication of the parotid gland surgery is
injury to the facial nerve. Facial nerve after emerging from the stylomastoid
foramen, enters into the parotid gland and divides into its five terminal
branches within the parotid gland. Thus facial nerve divides the parotid gland
into two parts, superficial and deep lobes.

TEST YOURSELF

Read the clinical scenario given at the beginning and answers the following questions
123456-

What is the most likely diagnosis in this case?


What are the important points in history taking in this patient?
What are the important points in clinical examination of this patient?
How will you manage this case?
What is the relation of facial nerve with the parotid gland?
Classify various tumours of the parotid gland.

157

Clinical scenarios in oto-rhino-laryngology

Case 49
Clinical Scenario
A 41 years old female patient came in OPD with the complaint of swelling
infront of her neck for last 4 years (fig. 49.1).

Fig. 49.1
Patient with swelling infront of the
neck

Important points in history taking:


1- Detailed history about the swelling. Onset of the swelling was insidious
and she did not know how it started. Then it started to increase in size
very slowly and gradually till it reached to its present form. There was
no known aggravating or relieving factor and it was not associated with
any other complaint.
2- Any sudden increase in size of the swelling. There was no history of
sudden increase in size.
3- Any history of pain in the swelling or neck. There was no such history.
4- Any symptom related with compression of either food or air passage.
There was no history of dysphagia, dyspnoea, stridor etc. but she complained
that now she could not sleep in supine position for a long time.
5- History of any symptom related with hyper or hypothyroidism like weight
loss or weight gain, palpitation, tremors, change in skin or hairs, change
in eyes, change in appetite etc. There was no such history.
6- Area where she lived. She was recently migrated to Karachi from northern
area of the country (which is endemic for goiter).
7- History of similar problem in the other family members. Many of her
close relatives and family members have similar problem.
Important points in clinical examination:
1- General physical examination. She was a middle aged women of obese
built, looking comfortable and well oriented. Her pulse rate was 78 per
158

Clinical scenarios in oto-rhino-laryngology

2-

345-

minute, blood pressure was 130/84 mm of Hg., respiratory rate was 20


per minute and temperature was 98.8oF. There was no pedal oedema,
tremors, sweating or dryness in hands etc.
Examination of the swelling. An irregular swelling was present in front
and lateral aspect of the neck, measuring 15 cms. at its maximum. The
margins of the swelling were ill defined and diffuse. The surface was
irregular with nodularity. It moved on swallowing but not on protrusion
of the tongue. The skin over the swelling was normal and mobile. It was
soft to firm in consistency with no fluctuation and transillumination was
negative. On auscultation there was no bruit.
Movement of the vocal cords. On indirect laryngoscopy both vocal cords
were fully mobile and normal.
Examination of the cervical lymph nodes. There was palpable lymph
node in the neck.
Examination of the eyes for proptosis and movements. It was within
normal limits.

Investigations:
1- Fine needle aspiration cytology of the swelling. It showed that aspirate
contained mainly colloid fluid with few clusters of follicular cells admixed
with foamy histiocytes and hemosiderin laden macrophages. The features
were suggestive of benign nodular goiter.
2- Ultrasonography of the swelling. It showed grossly enlarged thyroid gland
with multiple cysts of variable sizes in both lobes of the gland.
3- Thyroid function tests. T3, T4 and TSH were all within normal limits.
4- Serum calcium level. It was also within normal limit.
5- Baseline investigations for general anaesthesia. All were within normal
limits.
Diagnosis:
This was a case of multinodular goiter (MNG).
Treatment:
Patient was planned for thyroidectomy under general anaesthesia. A
cervical collar incision was given in a skin crease and flaps elevated. Straps
muscles were separated in the midline and retracted. Thyroid gland was
exposed and sub-total thyroidectomy was done (fig. 49.2 and 49.3) with
preservation of both inferior parathyroid glands with intact blood supply.
Drain was applied and wound closed in layers. Post-operative recovery was
uneventful.
Discussion:
Multinodular goiter (MNG) is a commonly used term to describe an
enlarged thyroid gland with multiple areas of nodularity. Worldwide MNG
is the most common endocrine disorder affecting 500 to 600 million people,
where iodine deficiency is often the causative factor. MNG is further classified
159

Clinical scenarios in oto-rhino-laryngology

Fig. 49.2
Multinodular goiter during surgery

Fig. 49.3
Multinodular goiter after sub-total
thyroidectomy

into toxic and non-toxic variety and the treatment slightly varies in these two
types. In typical non-toxic MNG surgery is done while in toxic MNG
medication is given before surgery to make the patient euthyroid. Although
the surgical technique is the same for both toxic and non-toxic MNG.
In iodine deficient areas, resultant hypothyroidism contributes to the
etiology of multinodular goiter. Through the feedback mechanism,
hypothyroidism causes an increase in TSH, which stimulates growth of
thyroid tissues. The other factors such as genetic influences also play a role
in the development of multinodular goiter. It is well established that
multinodular goiter may harbor occult malignancy, although the true incidence
has been disputed.
Patients with multinodular goiter often present with a neck swelling
producing cosmetic deformity or sometimes diagnosed by a physician during
routine examination. Large goiter may produce compressive symptoms like
dysphagia, feeling of something in the throat or dyspnoea especially on lying
supine.
Controversy remains regarding the extent of thyroid gland removal
so as to prevent recurrence in MNG and at the same time to prevent serious
complications. At one hand is the sub-total or near total thyroidectomy, where
160

Clinical scenarios in oto-rhino-laryngology

the complications are less but the chances of recurrence are high and resurgery in such cases has more complications. On the other hand is the total
thyroidectomy where there is virtually no chance of recurrence but the
complications are more than partial surgery. The rate of recurrence in cases
of MNG, treated by partial surgery is directly proportional to the volume of
remnant thyroid tissues left behind and the amount of multinodularity. In
cases of extensive, bilateral and generalized multinodularity, the recurrence
rate is higher.

TEST YOURSELF

Read the clinical scenario given at the beginning and answers the following questions
123456-

What are the important points in history taking in this patient?


What are the important points in clinical examination in this patient?
What is the most likely clinical diagnosis after history and clinical examination?
How will you investigate such a patient?
How will you treat this patient?
What is multinodular goiter (MNG)?

161

Clinical scenarios in oto-rhino-laryngology

Case 50
Clinical Scenario
A 14 years old boy came with the complaints of painless swelling on the right
side of his neck for last three months which was increasing progressively (fig. 50.1).
He was otherwise healthy and had no other symptoms referable to the head and neck
area, except he complained of low grade fever and sweating especially in the evening
for the same period.

Fig. 50.1
Patient with swelling in posterior
triangle of the neck on right side

Important points in history taking:


1- Detailed history regarding the swelling. According to the patient it started
insidiously and initially it was very small and barely visible but then it
started to increase in size progressively day by day. It was painless but
sometimes he felt discomfort in the neck. There was no aggravating or
relieving factors and no known associated symptom.
2- Detailed history regarding fever. According to the patient he had frequent
low grade fever often associated with chills and sweating especially in
the evening and night.
3- Any history of nasal obstruction, epistaxis or other complaint related with
the nose. There was no such history.
4- Any complaint related with respiratory or food passage. There was no
complaint.
5- Occupation and socio-economic condition. He was a worker with a car
mechanic and belonged to a low socio-economic class. He lived in a one
room house with eight family members.
6- Any family history of similar problem, tuberculosis or any respiratory
disease. According to him his elder brother had tuberculosis of the lungs.
7- Any history of weight loss or anorexia. There was no such history.
162

Clinical scenarios in oto-rhino-laryngology

8- Any history of smoking, pan eating or alcohol intake. There was no such
history.
Important points in clinical examination:
1- General physical examination. Patient was a young boy of lean built and
average height, fully oriented. His vital signs were within normal limits.
1- Examination of the swelling. There was an irregular swelling present in
the posterior triangle of the neck around 6x8 cms. The margins were illdefined and irregular, surface was nodular, overlying skin was normal
looking with no signs of inflammation. Swelling was not pulsatile and
not reducible. On palpation it was firm in consistency, nodular, not
fluctuant, slightly mobile over its bed, skin over the swelling was mobile.
The temperature of the swelling was same as of surrounding and it was
not tender to touch. Transillumination was negative. Clinical impression
being multiple lymph nodes matted together.
2- Examination of the nose and nasopharynx. It was within normal limits.
3- Examination of the throat and indirect laryngoscopy. It was within normal
limits.
4- Examination of the ears. There was no positive finding on it.
Differential Diagnosis:
1- Tuberculous lymphadenopathy
2- Lymphoma/leukaemia
3- Nodal metastasis from a primary in head & neck region
4- Other causes of lymphadenopathy
5- Benign or malignant tumour of mesenchymal origin
Investigations:
1- Blood complete picture and ESR. It showed increase in lymphocyte count
of 55% and raised ESR of 80 mm in first hour.
2- Plain X-ray chest (PA view). It was within normal limits.
3- Ultrasonography of the neck. It showed multiple solid swellings with
central necrosis in few of them. Impression was multiple lymph nodes
with central necrosis. Overall size was 7x8 cms.
4- Fine needle aspiration cytology (FNAC). It showed presence of granuloma
with multiple giant cell and casseous necrosis, consistent with tuberculosis.
5- Sputum for acid fast bacilli (AFB). It was negative for AFB.
6- Mountoux test (MT). It was strongly positive of 14 mm.
Diagnosis:
The diagnosis of this case was tuberculous cervical lymphadenopathy.
Treatment:
The patient was planned for antituberculous therapy. Initially four
drug regimen was given for two months including rifampicin, ethambutol,
pyrazinamide and isoniazid according to the body weight. After two months
163

Clinical scenarios in oto-rhino-laryngology

three drugs (rifampicin, ethambutol and isoniazid) were given for 6 months.
Patients recovery was uneventful with disappearance of all cervical lymph
nodes.
Discussion:
Tuberculosis is primarily a medical disease. If the diagnosis is
established in a timely and accurate manner, immediate institution of
antituberculous therapy produces dramatic resolution. In the past, surgery
has played an important role in both diagnosis as well as treatment of cervical
tuberculous lymphadenopathy. The only effective way to make definite
diagnosis in such cases is open biopsy. Increasing use of FNAC for evaluating
neck mass has revolutionized ability to make early diagnosis, institute
appropriate treatment and in many cases to avoid open biopsy.
Only 10 to 15% of the patients who present with cervical tuberculous
lymphadenopathy, will have chest X-ray findings consistent with pulmonary
tuberculosis. However despite normal chest X-ray, some of these patients
will have a positive sputum culture for acid fast bacilli (AFB). Therefore
even if the radiograph is negative, all patients with cervical tuberculous
lymphadenopathy should have obtained sputum culture for AFB.

TEST YOURSELF

Read the clinical scenario given at the beginning and answers the following questions
1234-

What is the most likely diagnosis in this case?


How will you investigate this patient?
How will you manage this case?
What is the role of open biopsy in such cases?

164

Clinical scenarios in oto-rhino-laryngology

INDEX

nose, 71, 84
Branchial cyst, 146
Bronchial foreign body, 138
Bronchoscopy, 139

Abscess,
brain abscess, 27
extra dural abscess, 27
parapharyngeal, 105, 145, 150
peri-tonsillar, 104
retropharyngeal, 105, 145, 149
septal abscess, 71
sub dural abscess, 27
Acoustic neuroma, 52
Acute,
otitis media, 18
retropharyngeal abscess, 149
suppurative otitis media, 18
tonsillitis, 150
Adenoid cystic carcinoma, 111
Adenoidectomy, 108
Adenoids, 107
Adenoid facies, 107
Adenoma, pleomorphic, 156
Allergic fungal sinusitis, 73
Allergic polyp, 73
Allergic rhinitis, 78
Angiofibroma, nasopharyngeal, 92
Angiography, 94
Antro-choanal polyp, 66, 75
Antrostomy, 91
Aspergillus, in otomycosis, 14
Attico antral, type of CSOM, 24, 29
Aural polyp, 21, 41

Caloric test, 49, 52


Canal wall down procedures, 26, 29
Canal wall up procedures, 23
Candida albicans, 15
Carcinoma,
cheek, 113
larynx, 133
tongue, 117
verrucous, 117
Carharts notch, 39
Catarrhal otitis media, 36
Cerebral hypoxia, 130
Cerumen, 9
Cervical lymphadenopathy, 162
Cholesteatoma, 24, 29
Chronic,
laryngitis, 128
mastoiditis, 26
retropharyngeal abscess, 149
sinusitis, 89
suppurative otitis media, 23, 24, 29
tonsillitis, 99
Cochlear implant, 55
Cortical mastoidectomy, 23
Cough, 138
CSOM,
attico-antral, 24, 29
tubo-tympanic, 23
CT scan,
larynx, 135
nose & PNS, 67, 68, 74, 90, 93, 94
Cyst, branchial, 146

Benign positional vertigo, 51


BERA, 54
Bimanual palpation, 153
Bleeding,
from the ear, 16
from the mouth, 102
from the nose, 62, 86, 92
Blockage, of the ear, 9
Boil,
ear, 3

Deafness, 3,9,21,24,34,38,42,45,48,54
Deviated nasal septum, 53
Direct laryngoscopy, 121, 134
165

Clinical scenarios in oto-rhino-laryngology

tongue, 117

Discharge,
blood stained, 81
from the ear, 3,14,16,21,24,28,31
from the nose, 59, 66, 73, 81
from the sinus, 12
Discomfort,
in the ear, 9
in the throat, 149
Dry perforation, of ear drum, 31
Dysphagia, 120, 149
Dyspnoea, 138

Haemangioma, 124
Haemorrhage, post-tonsillectomy, 102
Haematoma, septal, 60
Headache, 66, 89
Hearing aid, 44
Hearing impairment, 3, 9, 21, 24, 34, 38,
42, 45, 48, 54
Hoarseness, 127, 133
Hypertrophied inferior turbinate, 68
Hypopharynx, foreign body, 120
Hyposmia, 92

Earache, 3, 6, 9, 12, 16, 18, 107


Endolymphatic hydrops, 50
Epistaxis, 86
Epleys maneuver, 53
Ethmoidal polyp, 73
Ethmoidectomy, 91
Extra cranial complications, of CSOM, 26

Intracranial complications of CSOM, 27


Inverted papilloma, 68, 93
Itching, in the ear, 14

Juvenile, angiofibroma, 92

Facial asymmetry, 28
Facial nerve, 28
Fever, 18, 99, 104, 138, 162
Foreign body,
bronchial, 138
ear, 6
nose, 81
hypopharynx, 120
Fracture nasal bone, 62
Functional endoscopic sinus surgery, 68,75
Fungal sinusitis, 93
Furuncle,
ear, 3
nose, 71, 84

Kiessel Bachs plexus, 87

Labyrinthitis, 49, 52
Laryngeal,
carcinoma, 133
oedema, 145
tuberculosis, 128
Laryngectomy, 135
Laryngoscopy, 121, 134
Littles area, 87
Ludwigs angina, 143
Lymphadenopathy, 162
Lymphoma, 163

Gastro-oesophageal reflux, 99
Glue ear, 36
Goiter, multinodular, 158
Grommet, 36
Growth,
cheek, 110, 113
floor of the mouth, 123

Maggots, in ear, 16
Maggot oil, 17
Malignant tumours, of
larynx, 133
oral cavity, 113, 117
166

Clinical scenarios in oto-rhino-laryngology

neoplasia of, 110, 113


Orthopentomogram, 114
Osteoma. external auditory canal, 4
Osteomeatal complex, 90
Otitis externa, 10, 15
Otitis media,
acute suppurative, 18
catarrhal, 36
chronic suppurative, 23, 24, 29
exudative, 36
mucinous, 36
non-suppurative, 36
secretory, 36
serous, 36
with effusion, 36
Otomycosis, 10, 14
Otosclerosis, 38

Marsupialization, 124
Mass,
in the external auditory canal, 21
in the nose, 66, 73, 92
Mastoidectomy, 23
Maxillary sinusitis, 89
Mediastinitis, 145
Menieres disease, 48
Meningitis, 27
Microlaryngoscopy, 128
Modified radical mastoidectomy, 26, 29
Mouth,
breathing, 107
neoplasia of, 113, 117
Multinodular goiter, 158
Myringoplasty, 32
Myringotomy, 36

Pain,
during swallowing, 99, 104
ear, 3, 6, 9, 12, 16, 18, 107
facial, 66, 89
nose, 84
sub-mandibular region, 152
throat, 104, 120, 149
Papilloma, of cheek, 110
Paracusis Willisi, 38
Parapharyngeal abscess, 105, 145, 150
Parotid gland, 156
Parotidectomy, 156
Percutaneous travheostomy, 131
Perforation, tympanic membrane, 31
Peri-tonsillar abscess, see also quinsy, 104
Pleomorphic adenoma, 156
Plunging ranula, 124
Polyp,
antro-choanal, 66, 75
aural, 4, 21
ethmoidal, 73
Polypectomy, nasal, 68
Post-nasal dripping, 59, 89
Post-tonsillectomy haemorrhage, 102
Pre-auricular sinus, 12
Presbyacusis, 42

Nasal allergy, 78
Nasal bone fracture, 62
Nasal deformity, 62
Nasal obstruction, 59, 62, 66, 70, 73, 76,
78, 92, 107
Nasal packing, 86
Nasal polyp, 66, 73
Nasopharyngeal,
angiofibroma, 92
tonsils, 109
Nasopharynx,
neoplasia of, 92
Neck dissection, 115
Neoplasia, of
cheek, 110, 113
larynx, 133
oral cavity, 110, 113, 117
salivary gland, 156
tongue, 117
Noise induced deafness, 45
Non suppurative otitis media, 36

Odynophagia, 99, 104, 123


Oesophagoscopy, 122
Oral cavity,
167

Clinical scenarios in oto-rhino-laryngology

chin, 143
external auditory canal, 3
floor of the mouth, 123, 147
neck, 146, 158, 162
nose, 70, 84
parotid region, 155
sub-mandibular region, 147
tongue, 117
Syringing, ear, 38

Quinsy, 104

Radical mastoidectomy, 29
Ranula, 123
Retropharyngeal abscess, 105, 145, 149
Rhinorrhoea, 56, 66, 73, 78
Rhino-sinusitis, 89
Ringing, in the ear, 34, 45, 48
Roses position, 100

Tinnitus, 34, 45, 48


Tongue,
carcinoma, 117
Tonsillectomy, 100
Tonsillitis, chronic, 99
Total laryngectomy, 135
Tracheostomy, 130
Tracheostomy tube, 131
Traumatic perforation, of ear drum, 10,33
Trismus, 104, 113
Tuberculosis, of cervical lymph nodes, 162
Tubo tympanic, CSOM, 44
Tympanogram, 25
Tympanoplasty, 48

Salivary glands,
calculus, 153
neoplasia of, 155
Secretory otitis media, 36
Semonts maneuver, 53
Senile deafness, 42
Septal,
adhesions, 76
deviation, 53
haematoma, 70
surgery, 60
Septoplasty, 60
Septo-rhinoplasty, 64
Serous otitis media, 36
Sialoadenitis, 124
Sialogram, 153
Simple ranula, 124
Singers node, 128
Sinusitis, 89
Sneezing, 73, 78
Snoring, 107
Squamous cell carcinoma, 111,114,118,134
Squamous cell papilloma, 111
Stapedotomy, 40
Stridor, 133
Sub mucous diathermy, 80
Suction cleaning, of ear, 10
Superficial parotidectomy, 156
Supra-omohyoid neck dissection, 114,
Sweating, 162
Swelling,
cheek, 110

Vertigo, 48, 51
Vertigo, benign paroxysmal positional, 51
Verrucous carcinoma, 117
Vestibular neuronitis, 49, 52
Vocal nodule, 127
Voice change, 127, 133
Voice test, 19

Walshams forceps, 64
Wax, in ear, 38
Webers test, 19

X-ray,
mastoid, 22, 26
nasal bone, 63
PNS, 60, 67, 79
168

S-ar putea să vă placă și